Sbi Po 2013 Preparation Guide

169
7/30/2019 Sbi Po 2013 Preparation Guide http://slidepdf.com/reader/full/sbi-po-2013-preparation-guide 1/169 SBI PO 2013 Preparation Guide Page 1 www.jagranjosh.com Page 1 SBI PO 2013 Preparation Guide WWW.JAGRANJOSH.COM SBI PO 2013 PREPARATION GUIDE 

Transcript of Sbi Po 2013 Preparation Guide

Page 1: Sbi Po 2013 Preparation Guide

7/30/2019 Sbi Po 2013 Preparation Guide

http://slidepdf.com/reader/full/sbi-po-2013-preparation-guide 1/169

SBI PO 2013 Preparation Guide Page 1 

www.jagranjosh.com Page 1 

SBI PO 2013 Preparation Guide 

WWW.JAGRANJOSH.COM SBI PO 2013 PREPARATION

GUIDE 

Page 2: Sbi Po 2013 Preparation Guide

7/30/2019 Sbi Po 2013 Preparation Guide

http://slidepdf.com/reader/full/sbi-po-2013-preparation-guide 2/169

SBI PO 2013 Preparation Guide Page 2 

www.jagranjosh.com 

Page 2 

SBI PO 2013 Preparation Guide 

PREFACE

Jagranjosh State Bank of India Probationary Officers Recruitment Preparation Package (SBI PO

PDF Package) is an ideal platform for every candidate, who desires to achieve a good score in

the SBI Probationary Officers Recruitment. This exam which is going to be conducted on 28

April 2013.

The SBI PO Preparation Package has been prepared by experts of Jagranjosh.com. Our team of 

experts has put in its utmost efforts to bring out a perfect preparation package by blending in

all the ingredients of the bank recruitment examinations.

It is highly advisable for all those appearing for the examination to go through the package with

utmost seriousness and time their performance. This will allow them to get used to the time

limit set for the examination and accordingly manage time. Moreover the results of this exam

should be taken as a benchmark for their preparations so far, and students should try to

improve further more.

Jagranjosh SBI PO Exam PDF Package 2013

The SBI PO Preparation PDF Package includes:

  Reasoning

  Data Analysis and Interpretation

  General English

 General Awareness, Marketing and Computer(Separate Issue on 28

thMarch 2013 with latest update)

  Practice Set

  Previous Years Exam Papers

The questions from all the chapters have also been included to perfectly suit the candidates

need and to finally overcome the written examination. We have tried hard to ensure that each

and every question will help the students to brush their readings so far. Jagranjosh’s SBI PO PDF

Package is a remarkable adaptation to train the students for SBI PO Exam 2013. All the sections

of this Preparation Package have been made very reader-friendly.

The students on completion of the test will be left feeling more confident on their preparationskills and this will boost their overall performance at the time of the examination.

JagranJosh.com requests all the students to provide their valuable feedback on the package, so

we can provide better versions of preparatory packages to the candidates in the future.

Our team at Jagranjosh.com wishes all the students appearing for the examination.

All The Best.

Page 3: Sbi Po 2013 Preparation Guide

7/30/2019 Sbi Po 2013 Preparation Guide

http://slidepdf.com/reader/full/sbi-po-2013-preparation-guide 3/169

SBI PO 2013 Preparation Guide Page 3 

www.jagranjosh.com 

Page 3 

SBI PO 2013 Preparation Guide 

Contents 

Reasoning ........................................................................................................................................ 5 

Puzzles ..................................................................................................................................... 5 

Answers ................................................................................................................................... 9 

Input- Output ................................................................................................................................ 10 

Syllogism ....................................................................................................................................... 23 

Data Sufficiency ............................................................................................................................ 29 

Coding-Decoding ....................................................................................................................... 35 

Coded Relations ......................................................................................................................... 39 

Digits/Symbols & Codes ............................................................................................................. 42 

Strenghtening and Weakening Arguments ............................................................................... 45 

Courses of Action ....................................................................................................................... 50 

Cause and Effect ........................................................................................................................ 56 

Data Analysis And Interpretation ................................................................................................. 63 

Table Charts ............................................................................................................................... 63 

Bar Graphs ................................................................................................................................. 68 

Line Graphs ................................................................................................................................ 71 

Pie Charts ................................................................................................................................... 75 

Case Lets .................................................................................................................................... 79 

Permutations ............................................................................................................................. 83 

Combinations ............................................................................................................................. 91 

Probability ................................................................................................................................ 100 

Ratio & Proportion ................................................................................................................... 105 

Average .................................................................................................................................... 108 

Percentage ............................................................................................................................... 111 

English Language ......................................................................................................................... 115 

Page 4: Sbi Po 2013 Preparation Guide

7/30/2019 Sbi Po 2013 Preparation Guide

http://slidepdf.com/reader/full/sbi-po-2013-preparation-guide 4/169

SBI PO 2013 Preparation Guide Page 4 

www.jagranjosh.com 

Page 4 

SBI PO 2013 Preparation Guide 

Antonyms ................................................................................................................................. 115 

Synonyms ................................................................................................................................. 119 

Fill In the Blanks ....................................................................................................................... 123 

Error Detection ........................................................................................................................ 129 

Idioms & Phrases ..................................................................................................................... 132 

Cloze Tests ............................................................................................................................... 138 

Passages ................................................................................................................................... 145 

Para Jumbled ........................................................................................................................... 155 

Sentence Completion Tests ..................................................................................................... 163 

Page 5: Sbi Po 2013 Preparation Guide

7/30/2019 Sbi Po 2013 Preparation Guide

http://slidepdf.com/reader/full/sbi-po-2013-preparation-guide 5/169

SBI PO 2013 Preparation Guide Page 5 

www.jagranjosh.com 

Page 5 

SBI PO 2013 Preparation Guide 

REASONING

PUZZLES

Introduction: In the reasoning section of State Bank of India Probationary Exam, the questions

from puzzles are very important to get good marks in the exam. The questions of puzzles can be

asked in 2 or 3 sets and the number of questions can be 10 to 15.

Here we will do some exercises of puzzles which are very much similar to the expected

questions of SBI PO Exam 2013.

Practice Questions

Directions (Q. 1 – 5): Study the following information carefully and answer the questions given

below.

A, B, C, D, E, F and G are travelling in three different cars Alto, Punto and Vento and there are at

least two persons in any of these cars. Each of them has a favorite (likes) colour, viz black, red,

yellow, green, white, blue and pink, not necessarily in the same order.

  B likes yellow and is not travelling in Vento.

  The one who likes black is travelling in the same car in which E is travelling.

  C likes blue and is travelling in the same car in which G is travelling.  D is going in Punto only with the one who likes pink.

  G is not travelling either in Punto or Vento.

  F does not like black.

  G does not like either green or white.

  D does not like green.

  E does not like pink.

1.  Who likes Black?

(a) A

(b) B

(c)  C

(d) D

(e)  None of these

2.  Which of the following combinations is correct?

Page 6: Sbi Po 2013 Preparation Guide

7/30/2019 Sbi Po 2013 Preparation Guide

http://slidepdf.com/reader/full/sbi-po-2013-preparation-guide 6/169

SBI PO 2013 Preparation Guide Page 6 

www.jagranjosh.com 

Page 6 

SBI PO 2013 Preparation Guide 

(a)  D – Vento – White

(b) F – Alto – Pink

(c)  G – Alto – Red

(d) B – Punto – Yellow

(e) All are correct

3.  What is E’s favourite colour? 

(a)  Red

(b) Green

(c)  White

(d) Either Black or Yellow

(e)  None of these

4.  Which of the following person is travelling in Alto?

(a) CD

(b) BCE

(c)  DF

(d) BCG

(e)  None of these

5.  Who likes White?

(a)  A

(b) B

(c)  C

(d) Data inadequate

(e)  None of these

Directions (Q. 6 – 10): Study the following information carefully and answer the questions given

below.

A, B, C, D, E, F, G and H are sitting around a circular table facing the centre but not necessarily in

the same order.

  C is sitting third to the left of E.

  D and G are not the immediate neighbor C and E.

  A is second to the right of G and third to the left of H.

  F is not an immediate neighbor of E.

6.  Who is third to the left of G?

(a)  E

(b) F

(c)  H

Page 7: Sbi Po 2013 Preparation Guide

7/30/2019 Sbi Po 2013 Preparation Guide

http://slidepdf.com/reader/full/sbi-po-2013-preparation-guide 7/169

SBI PO 2013 Preparation Guide Page 7 

www.jagranjosh.com 

Page 7 

SBI PO 2013 Preparation Guide 

(d) Data inadequate

(e)  None of these

7.  What is F’s position with respect to B? 

(a)  Fourth to the right

(b) Third to the left

(c)  Third to the right

(d) Fourth to the left

(e)  None of these

8.  Who is fourth to the right of A?

(a)  E

(b) F

(c)  G

(d) H

(e)  None of these

9.  Who is second to the left of G?

(a)  A

(b) B

(c)  C

(d) D

(e)  None of these

10. Who are the immediate neighbors’ of C? (a)  AB

(b) AC

(c)  BF

(d) AF

(e)  None of these

Directions (Q. 11  – 15): Study the following information carefully and answer the questions

given below.

There are six friends; A, B, C, D, E and F sitting around a circular table. All the friends are facingthe centre. All of them are working as a Doctor, Engineer, Lawyer, Teacher, Shop keeper and

Manager but not in the same order.

  The manager and the teacher are the immediate neighbour of the shop keeper.

  A is sitting second to the left of the shop keeper and second to the right of B who is a

lawyer.

Page 8: Sbi Po 2013 Preparation Guide

7/30/2019 Sbi Po 2013 Preparation Guide

http://slidepdf.com/reader/full/sbi-po-2013-preparation-guide 8/169

SBI PO 2013 Preparation Guide Page 8 

www.jagranjosh.com 

Page 8 

SBI PO 2013 Preparation Guide 

  Teacher is not an immediate neighbor of A.

  Doctor is not the immediate neighbor of B.

  F is a manager.

  C is not an engineer.

  E is neither an engineer nor a shop keeper.

11. D is working as a/an

(a)  Doctor

(b) Engineer

(c)  Teacher

(d) Data inadequate

(e)  None of these

12. Who are the immediate neighbours of the engineer?

(a)  Shop keeper and Lawyer(b) Doctor and Manager

(c)  Teacher and Lawyer

(d) Cannot be determined

(e)  None of these

13. Who is sitting between A and C?

(a)  B

(b) D

(c)  E

(d) F

(e)  None of these

14. What is F’s position with respect to D? 

(a)  Second to the right

(b) Third to the right

(c)  Second to the left

(d) Third to the left

(e)  None of these

15. Which of the following pairs is sitting opposite to each other?

(a)  AC

(b) FE

(c)  DC

(d) BD

(e)  None of these

Page 9: Sbi Po 2013 Preparation Guide

7/30/2019 Sbi Po 2013 Preparation Guide

http://slidepdf.com/reader/full/sbi-po-2013-preparation-guide 9/169

SBI PO 2013 Preparation Guide Page 9 

www.jagranjosh.com Page 9 

SBI PO 2013 Preparation Guide 

ANSWERS

1.  (a)

2.  (c)

3.  (b)

4.  (d)

5.  (e)

6.  (c)

7.  (b)

8.  (a)

9.  (e)

10. (d)

11. (b)

12. (e)

13. (d)

14. (a)

15. (c)

Page 10: Sbi Po 2013 Preparation Guide

7/30/2019 Sbi Po 2013 Preparation Guide

http://slidepdf.com/reader/full/sbi-po-2013-preparation-guide 10/169

SBI PO 2013 Preparation Guide Page 10 

www.jagranjosh.com Page 10 

SBI PO 2013 Preparation Guide 

INPUT- OUTPUT

Introduction: The questions of Input-Output in the PO exams are very common and can be very

much scoring within few minutes. These questions are pattern based in which you have to find

the hidden pattern and then answer the given questions.

Here we will practice the questions of Input-Output

Practice Questions

Directions (Q. 1 to 5): Refer to the following data, answer the question that follows a set of 

numbers when put through a machine comes out in a particular sequence. The table is given

which has six steps. Study the table and answer the questions that follow:

Input: 94 32 54 18 09 08 17 21 68 77 84 92

Step 1: 94 32 54 18 09 17 21 68 77 84 92 08

Step 2: 94 92 32 54 18 17 21 68 77 84 09 08

Step 3: 94 92 84 32 54 18 21 68 77 17 09 08

Step 4: 94 92 84 77 32 54 21 68 18 17 09 08

Step 5: 94 92 84 77 68 32 54 21 18 17 09 08

Step 6: 94 92 84 77 68 54 32 21 18 17 09 08

1.  If two new numbers 30 and 22 are added to the sequence, then the position of number 32

with respect to 94 in the last step would be:

(a)  7th

 

(b) 9th

 

(c)  10th

 

(d) 8th

 

(e)  6th

 

2.  By putting the sequence given below in the same machine, what would be the third step in

the sequence?

Input: 24 28 37 11 78 89 96 68 48 54

(a)  96 89 78 11 24 28 37 68 54 48

(b) 96 78 37 89 68 54 48 28 24 11

Page 11: Sbi Po 2013 Preparation Guide

7/30/2019 Sbi Po 2013 Preparation Guide

http://slidepdf.com/reader/full/sbi-po-2013-preparation-guide 11/169

SBI PO 2013 Preparation Guide Page 11 

www.jagranjosh.com Page 11 

SBI PO 2013 Preparation Guide 

(c)  96 89 78 37 68 54 48 28 24 11

(d) 89 78 96 37 68 54 11 28 48 24

(e)  None of these

3.  For the sequence in above question what will be the 7th number from left?

(a)  37

(b) 54

(c)  48

(d) 24

(e)  None of these

4.  What would be the middle term of the following sequence after final step?

Input: 24 36 48 54 09 11 17 26 98

(a)  26

(b) 36

(c)  24

(d) 48

(e)  none of these

5.  For the sequence in above question, how many steps are involved to get the final

arrangement?

(a)  3

(b) 4

(c)  5

(d) 6

(e)  none of these

Directions (Q. 6 to 10): The arrangements of given input will follow a particular logic.

Understand this logic and solve the questions that follow.

Input: 45 328 877 24 159 648

Step 1: 877 328 45 24 159 648

Step 2: 877 328 45 648 159 24

Step 3: 877 648 45 328 159 24

Step 4: 877 648 159 328 45 24

Output: 877 648 328 159 45 24

6.  Which of the following will be 2nd last result of the following input:

Page 12: Sbi Po 2013 Preparation Guide

7/30/2019 Sbi Po 2013 Preparation Guide

http://slidepdf.com/reader/full/sbi-po-2013-preparation-guide 12/169

SBI PO 2013 Preparation Guide Page 12 

www.jagranjosh.com Page 12 

SBI PO 2013 Preparation Guide 

Input: 92 87 153 234 645

(a)  92 153 87 234 645

(b) 87 92 153 234 645

(c)  645 92 153 234 87

(d) None of these

(e)  Can’t be determined 

7.  If the given result is 5th and final then what will be the input?

Step 5: 334 224 114 84 64 4

(a)  224 114 84 64 4 334

(b) 114 224 84 4 64 334

(c)  334 224 4 84 64 114

(d) None of these

(e)  Can’t be determined 

8.  Which will be the last result of given input?

Input: 789 798 79 98 778

(a)  798 79 98 789 778

(b) 778 79 789 98 798

(c)  79 98 778 789 798

(d) 798 789 778 98 79

(e)  None of these

9.  Given result is first which will be the IIIrd result of input?

Step 1: 656 224 170 589 264

(a)  656 224 589 170 264

(b) 224 656 589 170 264

(c)  264 170 589 656 224

(d) None of these

(e)  Can’t be determined 

10. Following is the fourth result what will be the sixth?

Step 4: 317 314 263 219 87 85

(a)  317 314 219 263 87 85(b) 85 87 219 263 314 317

(c)  263 87 219 85 314 317

(d) None of these

(e)  Can’t be determined 

Page 13: Sbi Po 2013 Preparation Guide

7/30/2019 Sbi Po 2013 Preparation Guide

http://slidepdf.com/reader/full/sbi-po-2013-preparation-guide 13/169

SBI PO 2013 Preparation Guide Page 13 

www.jagranjosh.com Page 13 

SBI PO 2013 Preparation Guide 

Directions for questions 11 to 15: A word arrangement, when rearranged by given input

follows a particular logic behind it. Study that logic and answer the questions that follow.

Input: Blankets And Warm Quilts Help In Winter.

Step 1: Winter Blankets and Warm Quilts Help In

Step 2: Winter Warm Blankets and Quilts Help In

Step 3: Winter Warm Quilts Blankets and Help In

Step 4: Winter Warm Quilts in Blankets and Help

Output: Winter Warm Quilts in Help Blankets And

11. Which of the following will be Step 2 for the given input?

Input: “Life without success is boring” (a)  Is life boring without success

(b) Without life is boring success

(c)  Without success life is boring

(d) Data insufficient

(e)  None of these

12. In the above question, how many steps are required to get the final output?

(a)  1

(b) 2

(c)  3

(d) 4

(e)  None of these

13. If the following Step is 2nd

for a certain input then what will be the fourth Step?

Step-II: The success my is outcome of diligence.

(a)  My success is outcome of the diligence

(b) The my success is outcome of diligence

(c)  Success is the outcome of my diligence

(d) The success outcome of my is diligence(e)  None of these

14. Which of the following will be the last Step of given input?

Input: “Never break your elders trust.” 

(a)  Break never trust your elders

(b) Trust your elders never break

Page 14: Sbi Po 2013 Preparation Guide

7/30/2019 Sbi Po 2013 Preparation Guide

http://slidepdf.com/reader/full/sbi-po-2013-preparation-guide 14/169

SBI PO 2013 Preparation Guide Page 14 

www.jagranjosh.com Page 14 

SBI PO 2013 Preparation Guide 

(c)  Break your elders trust never

(d) Can’t be determined 

(e)  None of these

15. How many Steps will be required to get the final output in the above question?

(a)  1

(b) 2

(c)  3

(d) 4

(e)  None of these

Directions for questions 16 to 20: A word arrangement machine, when given an input line of 

words, rearranges them following a particular rule in each step. The following is an illustration

of input and the steps rearrangement.

Input: Guest forum to Thank Busy Easter to Access Bank

Step 1: Access Guest forum to Thank Busy Easter to Bank

Step 2: Access Bank Guest forum to Thank Busy Easter to

Step 3: Access Bank Busy Guest forum to Thank Easter to

Step 4: Access Bank Busy Easter Guest forum to thank to

Step 5: Access Bank Busy Easter forum Guest to Thank to

Step 6: Access Bank Busy Easter forum Guest Thank to to

Step 7: Access Bank Busy Easter forum Guest Thank To to

As per the rules followed in the above steps, find out in the given questions the appropriate

step for the given input.

16. Which of the following will be Step 4 for the given input?

Input: Star Fast arise on water Heater attire

(a)  arise attire Fast Heater on water star

(b) arise attire Fast Heater on star water

(c)  arise attire Fast Heater star on water

(d) arise attire Heater Fast star on water

(e)  None of these

17. Which of the following steps will be the last but one?

Page 15: Sbi Po 2013 Preparation Guide

7/30/2019 Sbi Po 2013 Preparation Guide

http://slidepdf.com/reader/full/sbi-po-2013-preparation-guide 15/169

SBI PO 2013 Preparation Guide Page 15 

www.jagranjosh.com Page 15 

SBI PO 2013 Preparation Guide 

Input: The other stations on first target sorted

(a)  2

(b) 3

(c)  4

(d) 5

(e)  None of these

18. The Step II of an input is as follows: “are Dare persons to over that” 

Which of the following would definitely be the input?

(a)  Dare over persons to are that

(b) Dare persons to are that over

(c)  Dare are persons to over that

(d) Cannot be determined

(e)  None of these

19. Which of the following steps would be: “an and every for peer to”? 

Input: every and peer to an for

(a)  2

(b) 3

(c)  4

(d) 5

(e)  None of these

20. Which step number will be the last step of the above input?

Input: Over Go Forum the at once

(a)  3

(b) 5

(c)  6

(d) 7

(e)  None of these

Answers:-

1.  (e)

2.  (e)

3.  (b)

4.  (a)

5.  (b)

6.  (c)

7.  (e)

Page 16: Sbi Po 2013 Preparation Guide

7/30/2019 Sbi Po 2013 Preparation Guide

http://slidepdf.com/reader/full/sbi-po-2013-preparation-guide 16/169

SBI PO 2013 Preparation Guide Page 16 

www.jagranjosh.com Page 16 

SBI PO 2013 Preparation Guide 

8.  (d)

9.  (d)

10. (e)

11. (c)

12. (b)

13. (d)

14. (e)

15. (c)

16. (c)

17. (d)

18. (e)

19. (b)

20. (e)

Answer Explanations

Solutions to questions 1 to 5:

The above questions can be solved using the following rules:

Input: 94 32 54 18 09 08 17 21 68 77 84 92

Step 1: The smallest is found out and is put at the end. At the same time, the largest is found

out and placed at the beginning of the series.

94 32 54 18 09 17 21 68 77 84 92 08

Step 2: leaving the ones that have been sent at the end of the sequence, step 1 is repeated till

the series is in decreasing order.

94 92 32 54 18 17 21 68 77 84 09 08

Step 3: 94 92 84 32 54 18 21 68 77 17 09 08

Step 4: 94 92 84 77 32 54 21 68 18 17 09 08

Step 5: 94 92 84 77 68 32 54 21 18 17 09 08

Step 6: 94 92 84 77 68 54 32 21 18 17 09 08

Q1: Ans. (e)

Final step would be the series in descending order. There is no need to go through all the steps.

Page 17: Sbi Po 2013 Preparation Guide

7/30/2019 Sbi Po 2013 Preparation Guide

http://slidepdf.com/reader/full/sbi-po-2013-preparation-guide 17/169

SBI PO 2013 Preparation Guide Page 17 

www.jagranjosh.com Page 17 

SBI PO 2013 Preparation Guide 

The last step after re-arranging will be:

94 92 84 77 68 54 32 30 22 21 18 17 09 08

Q2: Ans. (e)

Input: 24 28 37 11 78 89 96 68 48 54

Step 1: 96 24 28 37 78 89 68 48 54 11

Step 2: 96 89 28 37 78 68 48 54 24 11

Step 3: 96 89 78 37 68 48 54 28 24 11

Q3: Ans. (b)

From above

Q4: Ans. (a)

Final step would be the series in descending order. There is no need to go through all the steps.

The last step after re-arranging will be:

98 54 48 36 26 24 17 11 09

Q5: Ans. (b)

Input: 24 36 48 54 09 11 17 26 98

Step 1: 98 24 36 48 54 11 17 26 09

Step 2: 98 54 24 36 48 17 26 11 09

Step 3: 98 54 48 24 36 26 17 11 09

Step 4: 98 54 48 36 26 24 17 11 09

Solutions to questions 6 to 10:

The above questions can be solved using the following rules:

Input: 45 328 877 24 159 648

Step 1: The largest interchanges its position with the first one. 87745

877 328 45 24 159 648

Page 18: Sbi Po 2013 Preparation Guide

7/30/2019 Sbi Po 2013 Preparation Guide

http://slidepdf.com/reader/full/sbi-po-2013-preparation-guide 18/169

SBI PO 2013 Preparation Guide Page 18 

www.jagranjosh.com Page 18 

SBI PO 2013 Preparation Guide 

Step 2: The smallest interchanges its position with the last one. 24648

877 328 45 648 159 24

Step 3: Now the first and the last are left-out and steps 1 and 2 are repeated till the

sequence is arranged in descending order.

648328

877 648 45 328 159 24

Step 4: 45159

877 648 159 328 45 24

Step 5: 328159

877 648 328 159 45 24

(This is final output, as the series is now arranged in descending order.)

Q6: Ans. (c)

Input: 92 87 153 234 645

Step 1: 645 87 153 234 92

Step 2: 645 92 153 234 87

Step 3: 645 234 153 92 87

So the 2nd

last will be Result 2.

Q7: Ans. (e)

Previous steps can’t be determined as we can’t assume the position of the number in previous

results.

Q8: Ans. (d)

The answer can be easily found out by re-arranging the series in descending order. There is no

need to go through all the steps.

Q9: Ans (d)

Step 1: 656 224 170 589 264

Page 19: Sbi Po 2013 Preparation Guide

7/30/2019 Sbi Po 2013 Preparation Guide

http://slidepdf.com/reader/full/sbi-po-2013-preparation-guide 19/169

SBI PO 2013 Preparation Guide Page 19 

www.jagranjosh.com Page 19 

SBI PO 2013 Preparation Guide 

Step 2: 656 224 264 589 170

Step 3: 656 589 264 224 170

Q10: Ans. (e)

Because 317 314 263 219 87 85 is already arranged in descending order.

Solutions to Questions 11 to 15: These questions are based on alphabetical reverse order like Z

Y X W V U.... In every step the lowest in alphabetical order is found out and put at the

beginning.

Input: Blankets And Warm Quilts Help In Winter.

Step 1: Winter Blankets and Warm Quilts Help In

Step 2: Winter Warm Blankets and Quilts Help In

Step 3: Winter Warm Quilts Blankets and Help In

Step 4: Winter Warm Quilts In Blankets and Help

Step 5: Winter Warm Quilts In Help Blankets and (This is the final step)

Q11: Ans. (c)

Input: Life without success is boring

Step 1: Without life success is boring

Step 2: Without success life is boring (This is the final step)

Q12: Ans. (b)

Q13: Ans. (d)

Step 2: The success my is outcome of diligence.

Step 3: The success outcome my is of diligence.

Step 4: The success outcome of my is diligence.

Q14: Ans. (e)

In such type of problems direct rule follows that is to arrange the input in descending order.

Last Step will be “Your trust never elders break” 

Page 20: Sbi Po 2013 Preparation Guide

7/30/2019 Sbi Po 2013 Preparation Guide

http://slidepdf.com/reader/full/sbi-po-2013-preparation-guide 20/169

SBI PO 2013 Preparation Guide Page 20 

www.jagranjosh.com Page 20 

SBI PO 2013 Preparation Guide 

Q15: Ans (c)

Input: Never break your elders trust.

Step 1: Your never break elders trust

Step 2: Your trust never break elders

Step 3: Your trust never elders break

Solution for questions 16 to 20

In the given arrangement, the words have been arranged alphabetically in a sequence, altering

the position of only one word in each step.

Input: Guest forum to Thank Busy Easter To Access Bank

Step 1: Access Guest forum to Thank Busy Easter To Bank

Step 2: Access Bank Guest forum to Thank Busy Easter To

Step 3: Access Bank Busy Guest forum to Thank Easter To

Step 4: Access Bank Busy Easter Guest forum to Thank To

Step 5: Access Bank Busy Easter forum Guest to Thank To

Step 6: Access Bank Busy Easter forum Guest Thank to To

Step 7: Access Bank Busy Easter forum Guest Thank To to

Q16: Ans. (c)

Input: star Fast arise on water Heater at

Step 1: arise star Fast on water Heater at

Step 2: arise at star Fast on water Heater

Step 3: arise at Fast star on water Heater

Step 4: arise at Fast Heater star on water

Q17: Ans. (d)

Input: The other stations on first target sorted

Page 21: Sbi Po 2013 Preparation Guide

7/30/2019 Sbi Po 2013 Preparation Guide

http://slidepdf.com/reader/full/sbi-po-2013-preparation-guide 21/169

SBI PO 2013 Preparation Guide Page 21 

www.jagranjosh.com Page 21 

SBI PO 2013 Preparation Guide 

Step 1: first The other stations on target sorted

Step 2: first on The other stations target sorted

Step 3: first on other The stations target sorted

Step 4: first on other stations The target sorted

Step 5: first on other stations sorted The target

Step 6: first on other stations sorted target The

Clearly, Step 6 is the last step and 5 is the last but one (second last).

Q18: Ans. (e)

Clearly, none of the given input gives the desired output at Step 2 on rearrangement.

Q19: Ans. (b)

Input: every and peer to an for

Step 1: an every and peer to for

Step 2: an and every peer to for

Step 3: an and every for peer to

Q20: Ans. (e)

Input: Over Go Forum the at once

Step 1: at Over Go Forum the once

Step 2: at Forum Over Go the once

Step 3: at Forum Go Over the once

Step 4: at Forum Go once Over the

Since, all the words in the given input have been arranged alphabetically in Step 4, so it is the

last step.

Page 22: Sbi Po 2013 Preparation Guide

7/30/2019 Sbi Po 2013 Preparation Guide

http://slidepdf.com/reader/full/sbi-po-2013-preparation-guide 22/169

SBI PO 2013 Preparation Guide Page 22 

www.jagranjosh.com Page 22 

SBI PO 2013 Preparation Guide 

Page 23: Sbi Po 2013 Preparation Guide

7/30/2019 Sbi Po 2013 Preparation Guide

http://slidepdf.com/reader/full/sbi-po-2013-preparation-guide 23/169

SBI PO 2013 Preparation Guide Page 23 

www.jagranjosh.com Page 23 

SBI PO 2013 Preparation Guide 

SYLLOGISM

Introduction: In this unit we will study some useful concepts of Syllogism and questions which

are generally asked in the SBI PO Exams. After analyzing previous year papers, it is clear that in

the exam you can get 5 to 7 questions from Syllogism.

Syllogism: The literal meaning of syllogism is ‘Conclusion’ or ‘inference’. The questions in the

syllogism are in the form of statements (premises) followed by Conclusion (proposition) and the

students are asked to find the correct conclusion on the basis of the statements.

The general form of statements and conclusions in the syllogism is;

  All as are Bs. 

  Some As are Bs. 

  No A is a B. 

The graphical representation of the statements

1.  All As are Bs.

2.  Some As are Bs.

3.  No A is a B.

Page 24: Sbi Po 2013 Preparation Guide

7/30/2019 Sbi Po 2013 Preparation Guide

http://slidepdf.com/reader/full/sbi-po-2013-preparation-guide 24/169

SBI PO 2013 Preparation Guide Page 24 

www.jagranjosh.com Page 24 

SBI PO 2013 Preparation Guide 

Practice Examples

Direction (Q. 1 – 10): In each questions below there are two/three statements followed by two

conclusions I and II. Assuming both the statements true, you have to decide which of the two

conclusions logically follows the statements. Give your answer

(a)  If only conclusion I follows.

(b)  If only conclusion II follows.

(c)  If either conclusion I or conclusion II follows.

(d)  If neither conclusion I nor conclusion II follows.

(e)  If both conclusion I and conclusion II follow.

1.  Statements: Some pens are pencils.

Some pencils are books.

Conclusions: I. some pens are books.

II. All books are pens.

2.  Statements: All bottles are bags.

All bags are buses.

Conclusions: I. Some bags are bottles.

II. All bags are bottles.

3.  Statements: All pens are pencils.

No pencil is a book.

Conclusions: I. Some books are pens.

II. Some pencils are pens.

4.  Statements: Some cups are plates.

No plates are bottle.

Conclusions: I. Some bottles are cups

II. No bottle is a cup.

5.  Statements: All cups are plates.

All Plates are bottles.

Conclusions: I. Some bottles are cups.

II. Some plates are bottles.

6.  Statements: No plane is a river.

No river is a mountain.

Conclusions: I. Some mountains are planes.

II. No mountain is a plane.

Page 25: Sbi Po 2013 Preparation Guide

7/30/2019 Sbi Po 2013 Preparation Guide

http://slidepdf.com/reader/full/sbi-po-2013-preparation-guide 25/169

SBI PO 2013 Preparation Guide Page 25 

www.jagranjosh.com Page 25 

SBI PO 2013 Preparation Guide 

7.  Statements: No plane is a river. 

All rivers are mountains.

All planes are cities.

Conclusions: I. No plane is a mountain.

II. Some rivers are cities.

8.  Statements: Some planes are rivers. 

Some rivers are mountains.

Some mountains are cities.

Conclusions: I. Some planes are mountains.

II. Some rivers are cities.

9.  Statements: All planes are rivers. 

All rivers are mountains.

No mountain is a city.Conclusions: I. Some cities are rivers.

II. Some mountains are planes.

10.  Statements: Some planes are rivers. 

Some rivers are mountains.

No mountain is a city.

Conclusions: I. Some cities are planes.

II. No city is a plane. 

11. Statement: Some planes are rivers. No river is a mountain.

No mountain is a city.

Conclusion: I. Some cities are rivers.

II. No city is a river.

Answers:-

1.  (d)

2.  (a)

3.  (b)4.  (c)

5.  (e)

6.  (c)

7.  (d)

8.  (d)9.  (b)

10. (c)

Page 26: Sbi Po 2013 Preparation Guide

7/30/2019 Sbi Po 2013 Preparation Guide

http://slidepdf.com/reader/full/sbi-po-2013-preparation-guide 26/169

SBI PO 2013 Preparation Guide Page 26 

www.jagranjosh.com Page 26 

SBI PO 2013 Preparation Guide 

Answer Hints:-

1.  The diagram;

2.  The diagram;

3.  The diagram;

4.  The diagram;

6.  The diagram;

Page 27: Sbi Po 2013 Preparation Guide

7/30/2019 Sbi Po 2013 Preparation Guide

http://slidepdf.com/reader/full/sbi-po-2013-preparation-guide 27/169

SBI PO 2013 Preparation Guide Page 27 

www.jagranjosh.com Page 27 

SBI PO 2013 Preparation Guide 

7.  The diagram can be;

10. The diagram;

Page 28: Sbi Po 2013 Preparation Guide

7/30/2019 Sbi Po 2013 Preparation Guide

http://slidepdf.com/reader/full/sbi-po-2013-preparation-guide 28/169

SBI PO 2013 Preparation Guide Page 28 

www.jagranjosh.com Page 28 

SBI PO 2013 Preparation Guide 

Page 29: Sbi Po 2013 Preparation Guide

7/30/2019 Sbi Po 2013 Preparation Guide

http://slidepdf.com/reader/full/sbi-po-2013-preparation-guide 29/169

SBI PO 2013 Preparation Guide Page 29 

www.jagranjosh.com Page 29 

SBI PO 2013 Preparation Guide 

DATA SUFFICIENCY

Introduction: In the SBI PO exam you can get 5 to 7 questions from data sufficiency test. In thistype of questions the candidates are required to find out whether a questions can be answered

or not with the help of the given data.

Here, we will practice few questions of Data Sufficiency Test

Practice Questions

Directions: Each question is followed by two statements 'I' and 'II'. Mark your answer as

(a)  If the question can be answered with the help of statement 'I' alone.

(b)  If the question can be answered with the help of statement `II' alone.(c)  If the question can be answered with the help of both the statements together but not

with the help of either statement alone.

(d)  If the question cannot be answered even with the help of both the statements taken

together or separately.

(e)  If the question can be answered with the help of either the statement ‘I’ or the

statement ‘II’. 

1.  How far is town A from town B? Town C is 15 km west of town A.

I.  It is 10 km from town B to town C.

II.  There is a canal between town A and town B.

2.  Adi had an average score of 85 in three tests. What was the Adi's lowest score?

I.  Adi's highest score was 95.

II.  The average of Adi's two highest scores was 92.

3.  A shopkeeper sold an article for Rs. 100. How much profit did he earn?

I.  20% profit would have been earned if it were sold for Rs.90.

II.  The profit was one-third of the purchase price.

4.  What is Suman’s age today?

I.  Today, Suman’s age is five times her son’s present age. 

II.  Two years hence, Suman’s age will be three times her daughter’s age at that time. 

5.  Which is costlier – a cup of sweetened corn or a bottle of beer?

I.  Sweetened corns are sold at 4 cups for a Rs. 100.

Page 30: Sbi Po 2013 Preparation Guide

7/30/2019 Sbi Po 2013 Preparation Guide

http://slidepdf.com/reader/full/sbi-po-2013-preparation-guide 30/169

SBI PO 2013 Preparation Guide Page 30 

www.jagranjosh.com Page 30 

SBI PO 2013 Preparation Guide 

II.  Two bottles of beer can be exchanged for 4 cups of corn.

6.  Esha opened a savings account that gives simple annual interest by depositing Rs. 750.

What was the annual rate of interest?

I.  The bank does not charge for service and she made no transactions for the first 3

years.II.  Three years later her savings were Rs. 999

7.  A city doubles its population every 5 years. How long will it take to increase its population

by 3,600 from its present size?

I.  In 1972 the population was 475.

II.  The present population is 900.

8.  How many students appeared for the examination?

I.  Only 50% of students passed the examination.

II.  If 10 more students passed, the pass percentage would have been 51%.

9.  The ratio of milk to water in a mixture is 5 : 1. How many litres of water should be added to

make the ratio 5 : 3?

I.  The amount of water to be added is one third of the original mixture.

II.  The ratio of half the amount of milk to the amount of water in the original mixture is

5 : 2.

10. What is the speed of the boat in still water?

I.  It takes 2 hours to travel from A to B downstream.

II.  It takes 4 hours to travel B to A upstream.

11. Sandhya’s house is 40km away from Delhi. How long does the round trip from home to

delhi and back?

I.  Sandhya travelled at a uniform rate of 30km/hr for the whole trip.

II.  If Sandhya travelled 10km/hr faster, then it would have taken her three fourths of 

the time for the whole trip.

Answers:-

1. 

(d)

2.  (b)

3.  (c)

4.  (d)

Page 31: Sbi Po 2013 Preparation Guide

7/30/2019 Sbi Po 2013 Preparation Guide

http://slidepdf.com/reader/full/sbi-po-2013-preparation-guide 31/169

SBI PO 2013 Preparation Guide Page 31 

www.jagranjosh.com Page 31 

SBI PO 2013 Preparation Guide 

5.  (b)

6.  (c)

7.  (b)

8.  (c)

9.  (d)

10. (d)

11. (c)

Answer Explanations

1.  Ans.

Given:

From the first statement, we realise that the distance between town B and town C is 10km;

but the direction is not known.

From second statement nothing is known about the direction.

B

10 km

AC 15 km

AC 15 km

10 km B

10 AC 15B

AC 15

Page 32: Sbi Po 2013 Preparation Guide

7/30/2019 Sbi Po 2013 Preparation Guide

http://slidepdf.com/reader/full/sbi-po-2013-preparation-guide 32/169

SBI PO 2013 Preparation Guide Page 32 

www.jagranjosh.com Page 32 

SBI PO 2013 Preparation Guide 

Thus the question cannot be answered using the given statements. Option (d)

2.  Ans.

Let the lowest score be , the highest be , and the middle one be

Then,

Statement I:

Thus statement I alone is not sufficient to answer the question

Statement II:

Also,

Thus,

Thus Statement II alone is sufficient to answer the question.

Option (b)

3.  Ans.

S.P. = Rs. 100.

Statement I:

When S.P. = Rs. 90, Gain = 20%.

C.P. = Rs.100

90120

= Rs. 75.

Now, (C.P. = Rs. 75 and S.P. = Rs. 100) Profit = Rs. 25.

Thus, statement I alone gives the answer.

Statement II:

Let the C.P. be Rs. x. Then, gain = Rs.3

 x .

S.P. = Rs.3

 x  x 

= Rs.

4

3

 x .

Page 33: Sbi Po 2013 Preparation Guide

7/30/2019 Sbi Po 2013 Preparation Guide

http://slidepdf.com/reader/full/sbi-po-2013-preparation-guide 33/169

SBI PO 2013 Preparation Guide Page 33 

www.jagranjosh.com Page 33 

SBI PO 2013 Preparation Guide 

Thus,4

3

 x = 100  

3 100

4 x 

= 75 ; so C.P. = Rs. 75.

Thus, statement II alone also gives the answer. Option (c)

4.  Ans.

Statement I:

Today,

Suman’s age = 5 × (Her son’s age). 

Thus statement I alone is not sufficient to answer the question

Statement II:

Two years hence,

Suman’s age = 3 x (her daughter’s age) 

Thus statement II alone is not sufficient to answer the question

Clearly, data even in I and II is not sufficient to get Suman’s present age.

Option (d)

5.  Ans.

Statement I:

A cup of corn costs Rs. 100/4 = Rs. 25.

The rate of a bottle of beer is unknown so comparison cannot be made.

Thus statement I alone is not sufficient to answer the question

Statement II:

2 x (bottle of beer) = 4 x (Cup of corn)Bottle of beer = 2 x (Cup of corn)

Thus a bottle of beer is costlier than a cup of corn.

Thus statement II alone is sufficient to answer the question

Option (b)

6.  Ans. 

Statement I:

Statement I indicates that the amount deposited initially remained unchanged for 3 years. It

does not talk about anything else.

Thus statement I alone is not sufficient to answer the question

Statement II:

It indicates the amount accumulated at the end of 3 years.

Here we are not sure if any transaction was made during the three years or not.

Thus statement II alone is not sufficient to answer the question.

Page 34: Sbi Po 2013 Preparation Guide

7/30/2019 Sbi Po 2013 Preparation Guide

http://slidepdf.com/reader/full/sbi-po-2013-preparation-guide 34/169

SBI PO 2013 Preparation Guide Page 34 

www.jagranjosh.com Page 34 

SBI PO 2013 Preparation Guide 

Clearly, data in I and II is sufficient to answer the question as shown below:

Total interest in three years = 999-750 = 249

Interest per year = 249/3 = 83

Annual Rate of interest = 83/750 = 11.06%

(It should be noted here that the actual rate need not be calculated to answer this question)Option (c)

7.  Ans. 

Statement I:

Statement I is of no use since present year/population is not mentioned.

Thus statement I alone is not sufficient to answer the question

Statement II:

Present population is 900

Population after 5 years = 1800

Population after 10 years = 3600

Thus, the answer is 10 years.

Thus statement II alone is sufficient to answer the question.

Option (b)

8.  Ans. 

Let the total number of students that appeared in the examination be x.

Statement I:

No of passed students = 50% of x = 0.5x

Thus, statement I alone is not sufficient to answer the questionStatement II:

Let the passed number of students be p.

P + 10 = 51% of x

Thus statement II alone is not sufficient to answer the question.

Statement I and II together:

0.5x + 10 = 0.51x

0.01x = 10

X = 1000

Option (c)

9.  Ans.

Since the volume is not given in any of the statements, the question cannot be solved using

both the information together.

Option (d)

Page 35: Sbi Po 2013 Preparation Guide

7/30/2019 Sbi Po 2013 Preparation Guide

http://slidepdf.com/reader/full/sbi-po-2013-preparation-guide 35/169

SBI PO 2013 Preparation Guide Page 35 

www.jagranjosh.com Page 35 

SBI PO 2013 Preparation Guide 

10. Ans. 

Let the distance between A and B be D km and the speed of the boat and current in still

water be x km/hr and y km/hr, respectively.

Statement I:

D = (x –y) 2Thus, statement I alone is not sufficient to answer the question

Statement II:

D = (x –y) 4

Thus statement II alone is not sufficient to answer the question.

Statement I and II together:

Even if we combine both statements, we cannot find out the answer, because we have two

equations and three variables.

Option (d)

11. Ans.

Let the speed of Sandhya be x km/hr.

Time for round trip = 80/x

Statement I:

x = 30km/hr

Time = 80/30 hrs.

Thus, statement I alone is sufficient to answer the question

Statement II:

If speed = x + 10

Then, time = ¾ of 80/x = 60/x

Also, distance = 40km

i.e. (x + 10) (60/x) = 40

From this equation x can be found out, and so can be time.

Thus statement II alone is sufficient to answer the question.

Option (c)

CODING-DECODING

Introduction: In this unit, we will study about the different types of coding-decoding problems.In the SBI PO Exam this chapter consists of 5 to 7 questions. So this unit needs good practice so

that the students can be familiar with each and every type of questions which can be asked.

Coding: Coding is a process by which certain information is written into another form of 

information on the basis of certain principles. 

Page 36: Sbi Po 2013 Preparation Guide

7/30/2019 Sbi Po 2013 Preparation Guide

http://slidepdf.com/reader/full/sbi-po-2013-preparation-guide 36/169

SBI PO 2013 Preparation Guide Page 36 

www.jagranjosh.com Page 36 

SBI PO 2013 Preparation Guide 

Decoding: Decoding is use to infer the right information from the coded information.

For Example, In a certain code language, BOOK is written as 2151511. In the same language

how PEN be written?

Answer: - In the above example the letters of  BOOK is coded on the basis of their position inthe English alphabet. The letters B=2, O = 15, K = 11. In the same way we can encode PEN as

16514.

Practice Examples

1. If in a certain language, PAPER is written as OZODQ. Which word would be written as

RZKD?

(a)  SEAL

(b) SALE

(c) 

SELL(d) SKIP

(e)  None of these

2. If in a certain language, MANIPULATION is written as NOITALUPINAM. Which word would

be written as ERUTCURTS?

(a)  STRUCTURE

(b) FRACTURE

(c)  MANUFECTURE

(d) LECTURE

(e)  None of these

3. If in a certain language, TRIANGLE is written as SSHBMHKF. In the same language,

COUNTRY is written as:

(a)  BPVOSSX

(b) DNVMUQZ

(c)  BPTOSSX

(d) DNVNVQ 

(e)  None of these

4. In a certain language, TRY is coded as ABC and SHAPE is coded as DEFGH. How can THREE

be coded in the same language?(a)  ABEHH

(b) BAEFH

(c)  AECHF

(d) AEBHH

(e)  None of these

Page 37: Sbi Po 2013 Preparation Guide

7/30/2019 Sbi Po 2013 Preparation Guide

http://slidepdf.com/reader/full/sbi-po-2013-preparation-guide 37/169

SBI PO 2013 Preparation Guide Page 37 

www.jagranjosh.com Page 37 

SBI PO 2013 Preparation Guide 

5. In a certain code language, ANGLE is coded as IJKMN and STRIKE is coded as OPQRSN. How

can TRIANGLE be coded in the same language?

(a)  PQRQJJMN

(b) PQMNROJJ

(c)  POMNROJJ

(d) PQRIJKMN

(e)  None of these

6. In a certain code language, BRAIN is coded as ZYXWV and CREATE is coded as UYTXST. How

can CERTAIN be written in the same language?

(a)  UTYSXWV

(b) UTYSVWX

(c)  UTYSXVW

(d) UTYVWXS

(e)  None of these

7. If in a certain code language, TEACHER is coded as klmnolp. How can EARTH be coded in

the same language?

(a)  lmpok

(b)  lmpko

(c)  lmkop

(d)  lmokp

(e)  None of these

8. If OUGHT is coded as ABCDE in a certain language. How can TOUGH be coded in the same

language?

(a)  ABCDE

(b) BCDEA

(c)  CDEAB

(d) EABCD

(e)  None of these

9. If BITER is coded as KLMNO in a certain language. How can TRIBE be coded in the same

language?

(a)  MOLNK

(b) MOLKN

(c)  MOKLN

(d) MOKNL

(e)  None of these

10. If GROWN is coded as ABCDE in a certain code language. How can WRONG be coded in

the same language?

(a)  DBACE

Page 38: Sbi Po 2013 Preparation Guide

7/30/2019 Sbi Po 2013 Preparation Guide

http://slidepdf.com/reader/full/sbi-po-2013-preparation-guide 38/169

SBI PO 2013 Preparation Guide Page 38 

www.jagranjosh.com Page 38 

SBI PO 2013 Preparation Guide 

(b) DBAEC

(c)  DBCAE

(d) DBCEA

(e)  None of these

11. If F = 6, CAT = 24, then WORD =?(a)  59

(b) 60

(c)  61

(d) 62

(e)  None of these

12. If DOG is coded as 4157, then how would BLUE be coded?

(a)  212215

(b) 213215

(c)  212125

(d) 212521

(e)  None of these

13. If EDUCATION is coded as 2F5D1V34P, then how can EQUAL be coded?

(a)  2R15M

(b) 2RM15

(c)  2R51M

(d) 2R5M1

(e)  None of these

14. If N = 16, FAN = 27, then FRANCE =?(a) 56

(b) 57

(c) 58

(d) 59

(e) None of these

Answers:-

1.  (b)

2.  (a)

3.  (c)

4.  (d)

5.  (d)

6.  (a)

7.  (b)

Page 39: Sbi Po 2013 Preparation Guide

7/30/2019 Sbi Po 2013 Preparation Guide

http://slidepdf.com/reader/full/sbi-po-2013-preparation-guide 39/169

SBI PO 2013 Preparation Guide Page 39 

www.jagranjosh.com Page 39 

SBI PO 2013 Preparation Guide 

8.  (d)

9.  (b)

10. (d)

11. (b)

12. (a)

13. (c)

14. (d)

Answer Hints:-

1. (b) The every letters has been decreased by one in the code according to the English

alphabet.

2. (a) The letters are written in the reversed order.

3. (c) In the code, the letters at odd places is decreased by one and the letters at even placesis increased by one according to the English alphabet.

4. (d) Here, the code for T = A, R = B, Y = C, S = D, H = E, A = F, P = G, and E = H. therefore the

code for THREE is AEBHH.

5. (d) Here, the code for A = I, N = J, G = K, L = M, E = N, S = O, T = P, R = Q, I = R and K = S.

Therefore the code for TRIANGLE is PQRIJKMN.

6. (a) Here, the code for B = Z, R = Y, A = X, I = W, N = V, C = U and T = S. Therefore the code for

CERTAIN is UTYSXWV.

7. (b) Here, the code for T = k, E = l, A = m, C = n, H = o and R = p. Therefore the code for

EARTH is lmpko.

11. (b) Here, F = 6, CAT = 3 + 1 + 20 = 24. Therefore, WORD = 23 + 15 + 18 + 4 = 60.

14. (d) Here, N =(14 + 2), FAN = (6 + 2) +(1 + 2) + (14 + 2),

Therefore, FRANCE = (6 + 2) + (18 + 2) + (1 + 2) + (14 + 2) + (3 + 2) + (5 + 2) = 59.

CODED RELATIONS

Introduction: In this unit we will study about the questions of coded relation. In the SBI PO

Exam, there can be 5 questions from this section. With practice the candidates can solve all the

five questions from this section very easily.

Page 40: Sbi Po 2013 Preparation Guide

7/30/2019 Sbi Po 2013 Preparation Guide

http://slidepdf.com/reader/full/sbi-po-2013-preparation-guide 40/169

SBI PO 2013 Preparation Guide Page 40 

www.jagranjosh.com Page 40 

SBI PO 2013 Preparation Guide 

Solving the questions from Coded-Relation involves these steps

  Convert the coded relation value in the mathematical relations. Generally these relations

are;

‘<’ 

‘≤’ ‘>’ 

‘≥’ 

‘=’ 

  Place the mathematical relation in the statement of each statement and conclusion.

  And answer the conclusion which follows the respective statement. 

Practice Questions

Directions (Q. 1 – 10): in the following questions, the symbols @, #, $, © and % are used with

different meanings as follows:

  P @ Q means “P is not smaller than Q”. 

  P # Q means “P is not greater than Q”. 

  P $ Q means “P is neither greater than nor equal to Q”. 

  P © Q means “P is neither smaller than nor equal to Q”.  

  P % Q means “P is neither greater than nor smaller than Q”.

Now in each of the following questions assuming the given statements to be true, find which of 

the two conclusions I and II given below them is/are definitely true.

Give answer

(a)  If only conclusion I is true.

(b) If only conclusion II is true.

(c)  If either conclusion I or II is true.

(d) If neither conclusion I nor II is true.

(e)  If conclusions I and II both are true.

1.  Statement: A @ B, B © C, C % D

Conclusions: I. A @ DII. D % B

2.  Statement: A # B, B © C, C @ D

Conclusions: I. A @ C

II. B © C

Page 41: Sbi Po 2013 Preparation Guide

7/30/2019 Sbi Po 2013 Preparation Guide

http://slidepdf.com/reader/full/sbi-po-2013-preparation-guide 41/169

SBI PO 2013 Preparation Guide Page 41 

www.jagranjosh.com Page 41 

SBI PO 2013 Preparation Guide 

3.  Statement: A @ B, B @ C, C @ D

Conclusions: I. D # B

II. C % A

4.  Statement: A @ B, B © C, C # D

Conclusions: I. A © C

II. B % D

5.  Statement: A © B, B % C, C $ D

Conclusions: I. A © C

II. B $ D

6.  Statement: A @ B, B @ C, C # D

Conclusions: I. A © C

II. A % C

7.  Statement: A $ B, B # C, C % D

Conclusions: I. D % B

II. A # C

8.  Statements: A $ B, B @ C, C # D

Conclusions: I. C © A

II. B % D

9.  Statement: A # B, B $ C, C @ D 

Conclusions: I. B % D

II. C © A

10. Statement: A $ B, B # C, C $ D

Conclusions: I. C © A

II. B $ D

Answer :

1.  (d)

2.  (b)

3.  (a)

4.  (a)

5.  (e)

6.  (c)

7.  (d)

Page 42: Sbi Po 2013 Preparation Guide

7/30/2019 Sbi Po 2013 Preparation Guide

http://slidepdf.com/reader/full/sbi-po-2013-preparation-guide 42/169

SBI PO 2013 Preparation Guide Page 42 

www.jagranjosh.com Page 42 

SBI PO 2013 Preparation Guide 

8.  (d)

9.  (b)

10. (e)

Answer Hints:

1.  (d) The coded relation of A, B, C and D in the statement is; A ≥ B > C = D. 

2.  (b) The coded relation of A, B, C and D in the statement is; A ≤ B > C ≥ D. 

3.  (a) The coded relation of A, B, C and D in the statement is; A ≥ B ≥ C ≥ D.  

4.  (a) The coded relation of A, B, C and D in the statement is; A ≥ B > C ≤ D. 

5.  (e) The coded relation of A, B, C and D in the statement is; A > B = C < D.

6.  (c) The coded relation of A, B, C and D in the statement is; A ≥ B ≥ C ≤ D.  

7.  (d) The coded relation of A, B, C and D in the statement is; A < B ≤ C = D. 

8.  (d) The coded relation of A, B, C and D in the statement is; A < B ≥ C ≤ D. 

9.  (b) The coded relation of A, B, C and D in the statement is; A ≤ B < C ≥ D. 

10. (e) The coded relation of A, B, C and D in the statement is; A < B ≤ C < D.  

DIGITS/SYMBOLS & CODES

Introduction: When you will go through the questions of previous exams conducted by State

Bank of India you will find that the examining authorities have asked five questions from

Digits/Symbols & Codes.

These questions are relatively easy and require less time to be solved but at the same time

these questions require very good concentration otherwise you can mark a wrong answer

because the answer options are very close to the right answer.

Here, we will do few exercise of solving these questions.

Practice Questions

Directions (Q. 1  – 10): In each question below, a group of digits/symbols is given followed by

four combinations of letters options (a), (b), (c) and (d). You have to find out which of the

Page 43: Sbi Po 2013 Preparation Guide

7/30/2019 Sbi Po 2013 Preparation Guide

http://slidepdf.com/reader/full/sbi-po-2013-preparation-guide 43/169

SBI PO 2013 Preparation Guide Page 43 

www.jagranjosh.com Page 43 

SBI PO 2013 Preparation Guide 

combination is the correct code on the following coding system and the conditions that follow.

If none of the four correctly represents the group of digits/symbols, mark (e), i.e. “None of 

these” as the answer. 

Digits/Symbols 7 9 5 # 4 & 3 * 1 8 $ 2 0 @ 6 %

Letter Codes K L O P W R T E X M N B Z Y S D

Conditions:

i.  If the first unit in the group is an even digit and the last unit is a symbol both these are

to be coded as the code for the symbol.

ii.  If the first unit in the group is an odd digit and the last unit is even digit, there codes are

to be interchanged.

iii.  If both the first and the last units are symbols both these are to be coded as ‘A’. 

1.  53&8%(a)  OTRDM

(b) OTRMD

(c)  DTRMO

(d) DTROM

(e)  None of these

2.  84#7*

(a)  MWPKE

(b) MWPEK

(c)  EWPKM

(d) EWPKE

(e) None of these 

3.  #560&

(a)  POSZR

(b) ROSZP

(c)  AOSZA

(d) AOSZR

(e)  None of these

4.  2@9%3

(a)  BYLDT

(b) TYLDB

(c)  AYLDA

Page 44: Sbi Po 2013 Preparation Guide

7/30/2019 Sbi Po 2013 Preparation Guide

http://slidepdf.com/reader/full/sbi-po-2013-preparation-guide 44/169

SBI PO 2013 Preparation Guide Page 44 

www.jagranjosh.com Page 44 

SBI PO 2013 Preparation Guide 

(d) TYLDT

(e)  None of these

5.  1$7&8

(a)  XNKRM

(b) ANKRA

(c)  MNKRM

(d) XNKRX

(e)  None of these

6.  9#*%5

(a)  LPEDO

(b) OPEDO

(c)  OPEDL

(d) APEDA(e)  None of these

7.  &4%58

(a)  MWDOM

(b) RWDOM

(c)  AWDOM

(d) AWDOA

(e) None of these 

8.  7#2$6(a)  KPBNS

(b) SPBNS

(c)  SPBNK

(d) APBNA

(e)  None of these

9.  4#78*

(a)  WPKME

(b) EPKMW

(c)  EPKWM

(d) APKWA

(e)  None of these

10. 2%0@6

(a)  SDZYB

Page 45: Sbi Po 2013 Preparation Guide

7/30/2019 Sbi Po 2013 Preparation Guide

http://slidepdf.com/reader/full/sbi-po-2013-preparation-guide 45/169

SBI PO 2013 Preparation Guide Page 45 

www.jagranjosh.com Page 45 

SBI PO 2013 Preparation Guide 

(b) SDZYS

(c)  ADZYA

(d) BDZYS

(e)  None of these 

Answers:-

1.  (b)

2.  (d)

3.  (c)

4.  (a)

5.  (e)

6.  (a)

7.  (b)

8.  (c)

9.  (e)

10. (d)

STRENGHTENING AND WEAKENING ARGUMENTS

Introduction: In the State Bank of India Probationary Exam you can find questions which will be

dealing with statements and arguments which can be strengthening or weakening in their

nature. The numbers of questions of these types are generally 5.

Here, we will practice few questions which are very similar to the questions which can be asked

in the exam.

Practice questions

Directions (Q. 1 – 20): Each of the following questions consists of a statement followed by two

arguments I and II. Give answer as

(a)  If only argument I is strong;

(b) If only argument II is strong;

(c)  If either I or II is strong;

(d)  If neither I nor II is strong and

(e)  If both I and II are strong.

1.  Statement: Should foreign films be banned in India?

Arguments:

I.  Yes. They depict a culture which adversely affects the Indian value system.

II.  No. Foreign films are of a high artistic standard.

2.  Statement: Should the government jobs be offered only to the wards of governmentemployees?

Arguments:

I.  No. It denies opportunity to many deserving individuals and government may lose in the

long run.

II.  No. It is against the principle of equality of all.

Page 46: Sbi Po 2013 Preparation Guide

7/30/2019 Sbi Po 2013 Preparation Guide

http://slidepdf.com/reader/full/sbi-po-2013-preparation-guide 46/169

SBI PO 2013 Preparation Guide Page 46 

www.jagranjosh.com Page 46 

SBI PO 2013 Preparation Guide 

3.  Statement: Should young entrepreneurs be encouraged?

Arguments:

I.  Yes. They will help in industrial development of the country.

II.  Yes. They will reduce the burden on employment market.

4.  Statement: Should the sex determination test during pregnancy be completely banned

Arguments:

I.  Yes. This leads to indiscriminate female foeticide.

II.  No. People have a right to know about their unborn child.

5.  Statement: Should government jobs in rural areas have more incentives?

Arguments:

I.  Yes. Incentives are essential for attracting government servants there.

II.  No. Rural areas are already cheaper, healthier and less complex than big cities. So, why

offer extra incentives!

6.  Statement: Should there be only one rate of interest for term deposits of varying durations

in banks?

Arguments:

I.  No. People will refrain from keeping money for longer duration resulting into reduction

of liquidity level of banks.

II.  Yes. This will be much simple for the common people and they may be encouraged to

keep more money in banks.

7.  Statement: Should government stop spending money on international sports?Arguments:

I.  Yes. This money can be utilized for upliftment of the poor.

II.  No. Sports persons will be frustrated and will not get international exposure.

8.  Statement: Should officers accepting bribe be punished?

Arguments:

I.  No. Certain circumstances may have compelled them to take bribe.

II.  Yes. They should honestly do their duty.

9.  Statement: Should India become a permanent member of UN's Security Council?Arguments:

I.  Yes. India has emerged as a country which loves peace and amity.

II.  No. Let us first solve problems of our own people like poverty, malnutrition and then

think of other nations.

Page 47: Sbi Po 2013 Preparation Guide

7/30/2019 Sbi Po 2013 Preparation Guide

http://slidepdf.com/reader/full/sbi-po-2013-preparation-guide 47/169

SBI PO 2013 Preparation Guide Page 47 

www.jagranjosh.com Page 47 

SBI PO 2013 Preparation Guide 

10. Statement: Should income tax be abolished in India?

Arguments:

I.  Yes. It is an unnecessary burden on the wage earners.

II.  No. It is a good source of revenue for the government.

11. Statement: Should fashionable dresses be banned?

Arguments:

I.  Yes. Fashions keep changing and hence consumption of cloth increases.

II.  No. Fashionable clothes are a person's self expression and therefore his/her

fundamental right.

12. Statement: Should cottage industries be encouraged in rural areas?

Arguments:

I.  Yes. Rural people are creative.

II.  Yes. This would help to solve the problem of unemployment to some extent.

13. Statement: Should a health warning essentially appear on cigarette packs?

Arguments:

I.  Yes. It is a sort of brainwash to make the smokers realize that they are inhaling

poisonous stuff.

II.  No. It hampers the enjoyment of smoking.

14. Statement: Is paying ransom or agreeing to the conditions of kidnappers of political figures,

a proper course of action?

Arguments:I.  Yes. The victims must be saved at all cost.

II.  No. It encourages the kidnappers to continue their sinister activities.

15. Statement: Should the vehicles that have been running for more than 15 years be rejected

in India?

Arguments:

I.  Yes. This is a significant step to lower down the pollution level.

II.  No. It will be very difficult for vehicle owners to shift to other parts of world because

they will not get suitable job for their very I existence.

16. Statement: Should children be legally made responsible to take care of their parents during

their old age?

Arguments:

I.  Yes. Such matter can only be solved by legal means.

II.  Yes. Only this will bring some relief to poor parents.

Page 48: Sbi Po 2013 Preparation Guide

7/30/2019 Sbi Po 2013 Preparation Guide

http://slidepdf.com/reader/full/sbi-po-2013-preparation-guide 48/169

SBI PO 2013 Preparation Guide Page 48 

www.jagranjosh.com Page 48 

SBI PO 2013 Preparation Guide 

17. Statement: Should India support all the international policies of United States of America?

Arguments:

I.  No. Many other powerful countries do not support the same.

II.  Yes. This is the only way to gain access to USA developmental funds. 

18. Statement: Should all the drugs patented and manufactured in Western, countries be first

tried out on sample basis before giving license for sale to general public in India?

Arguments:

I.  Yes. Many such drugs require different doses and duration for Indian population and

hence it is necessary.

II.  No. This is not feasible and hence cannot be implemented.

19. Statement: Should so much money be spent on advertisements?

Arguments:

I.  Yes. It is essential in a capitalist economy.II.  No. It leads to wastage of resources.

20. Statement: Should jobs be linked with academic degrees and diplomas?

Arguments:

I.  No. A very large number of persons with average academic qualifications will apply.

II.  No. Importance of higher education will be diminished.

Answers:-

1.  (d)

2.  (e)

3.  (e)

4.  (a)

5.  (a)

6.  (a)

7.  (b)

8.  (b)

9.  (a)

10. (b)

11. (b)

12. (b)

13. (a)

14. (e)

15. (a)

16. (d)

17. (d)

18. (a)

19. (a)

20. (b)

Answer Explanation:

1.  Ans. (d) clearly, foreign films depict the alien culture but this only helps in learning more.

So, argument I does not hold. Also, the reason stated in argument II is not strong enough in

contradicting the ban. So, it also does not hold.

2.  Ans. (e) Merit, fair selection and equal opportunities for all - these three factors, if taken

care of, can help government recruit competent officials and also fulfill the objectives of the

Constitution. Thus, both the arguments hold strong.

Page 49: Sbi Po 2013 Preparation Guide

7/30/2019 Sbi Po 2013 Preparation Guide

http://slidepdf.com/reader/full/sbi-po-2013-preparation-guide 49/169

SBI PO 2013 Preparation Guide Page 49 

www.jagranjosh.com Page 49 

SBI PO 2013 Preparation Guide 

3.  Ans. (e) clearly, encouraging the young entrepreneurs will open up the field for the

establishment of new industries. Thus, it shall help in industrial development and not only

employ the entrepreneurs but create more job opportunities for others as well. So, both the

arguments hold strong.

4.  Ans. (a) Parents indulging in sex determination of their unborn child generally do so as they

want to keep only a boy child and do away with a girl child. So, argument I hold. Also,

people have a right, to know only about the health, development and general well-being of 

the child before its birth, and not the sex. So, argument II does not hold strong.

5.  Ans. (a) clearly, government jobs in rural areas are underlined with several difficulties. In

lieu of these, extra incentives are needed. So, only argument I holds strong.

6.  Ans. (a) clearly, the proposed scheme would discourage people from keeping deposits for

longer durations (the rate of interest being the same for short durations) and not draw in

more funds. So, only argument I holds.

7.  Ans. (b) clearly, spending money on sports cannot be avoided merely because it can be

spent on socio-economic problems. So, argument I does not hold. Also, if the expenses on

sports are curtailed, the sports persons would face lack of facilities and training and our

country will lag behind in the international sports competitions. So, II holds.

8.  Ans. (b) clearly, officers are paid duly for the jobs they do. So, they must do it honestly.

Thus, argument II alone holds.

9.  Ans. (a) a peace-loving nation like India can well join an international forum which seeks to

bring different nations on friendly terms with each other. So, argument I holds strong.

Argument II highlights a different aspect. The internal problems\of a nation should not

debar it from strengthening international ties. So, argument II is vague.

10. Ans. (b) Income -tax is levied so that every citizen can contribute a share of his earning

towards the infrastructural development of the nation. So, argument I seems to be vague.

However, income-tax is no doubt a good source of revenue for the government. Hence,

argument II holds strong.

11. Ans. (b) clearly, imposing ban on fashionable dresses will be a restriction on the personal

choice and hence the right to freedom of an individual. So, only argument II is strong.

12. Ans. (b) clearly, cottage industries need to be promoted to create more job opportunities

for rural people in the villages themselves. The reason that rural people are creative is

vague. So, only argument Il holds.

Page 50: Sbi Po 2013 Preparation Guide

7/30/2019 Sbi Po 2013 Preparation Guide

http://slidepdf.com/reader/full/sbi-po-2013-preparation-guide 50/169

SBI PO 2013 Preparation Guide Page 50 

www.jagranjosh.com Page 50 

SBI PO 2013 Preparation Guide 

13. Ans. (a) clearly, such words on cigarette packs would warn the smokers beforehand of its

adverse effects. So, argument I holds strong. However, smoking is a bad habit with' long-

term health hazards and is no means of enjoyment. So, argument II is vague.

14. Ans. (e) both the arguments are strong enough. The conditions have to be agreed to, in

order to save the life of the victims, though actually they ought not to be agreed to, as they

encourage the sinister activities of the kidnappers.

15. Ans. (a) clearly, 15 year old vehicles are not Euro-compliant and hence cause much more

pollution than the recent ones. So, argument I holds. Argument II is vague since owners of 

these vehicles need not shift themselves. They might sell off their vehicles and buy new

ones — a small price which every citizen can afford for a healthy environment.

16. Ans. (d) Taking care of the parents is a moral duty of the children and cannot be thrust upon

them legally, nor can such a compulsion ensure good care of the old people. So, none of the

arguments holds strong.

17. Ans. (d) our country cannot support USA's policies blindly without analysis, just to gain

monetary help. Also, we should not withdraw our support without considering the policies,

 just because some other nations have done so. So, none of the arguments holds strong.

18. Ans. (a) clearly, health of the citizens is an issue of major concern for the Government. So, a

product like drugs must be first studied and tested in the Indian context before giving

license for its sale. So, only argument I holds strong.

19. Ans. (a) clearly, the advertisements are the means to introduce people with the product

and its advantages. So, argument I holds strong. But argument II is vague because

advertisements are an investment for better gain and not wastage.

20. Ans. (b) clearly, delinking jobs with degrees will diminish the need for higher education as

many of them pursue such education for jobs. So, only argument II is strong.

COURSES OF ACTION

Introduction: In the SBI PO exam, the questions on Courses of Action are asked to judge the

Applicable knowledge of a candidate. These questions are related to decision making situations

in which you have to suggest the most suitable and correct action which will be required in a

given situation.

Here, we will practice few questions from the topic.

Page 51: Sbi Po 2013 Preparation Guide

7/30/2019 Sbi Po 2013 Preparation Guide

http://slidepdf.com/reader/full/sbi-po-2013-preparation-guide 51/169

SBI PO 2013 Preparation Guide Page 51 

www.jagranjosh.com Page 51 

SBI PO 2013 Preparation Guide 

Practice Questions

Directions (Q. 1 to 20): In each question below given is a statement followed by two courses of 

action numbered I and II. Assume everything in the statement to be true and on the basis of the

information given in the statement, decide which of the suggested courses of action logically

follow.

Mark your answer as:

(a)  if only I follows;

(b)  if only II follows,

(c)  if either I or II follows;

(d)  if neither I nor II follows and

(e)  if both I and II follow.

1.  Most of the development plans in the country develop in papers only.

I.  The in-charges should be instructed to supervise the field-work regularly.

II.  The supply of, paper to such departments should be cut short.

2.  The Central Bureau of Investigation receives the complaint of an officer taking bribe to do

the duty he is supposed to.

I.  CBI should try to catch the officer red-handed and then take a strict action against him.

II.  CBI should wait for some more complaints about the officer to be sure about the

matter.

3.  Due to substantial reduction in fares by different airline services, large numbers of 

passengers so far travelling by upper classes in trains have switched over to airline services.

I.  The railways should immediately reduce the fare structure of the upper classes

substantially to retain its passengers.

II.  The railways should reduce the capacity of upper classes in all the trains to avoid loss.

4.  The availability of imported fruits has increased in the indigenous market and so the

demand for indigenous fruits has been decreased.

I.  To help the indigenous producers of fruits, the Government should impose high import

duty on these fruits, even if these are not of good quality.

II.  The fruit vendors should stop selling imported fruits so that the demand for indigenous

fruits would be increased.

Page 52: Sbi Po 2013 Preparation Guide

7/30/2019 Sbi Po 2013 Preparation Guide

http://slidepdf.com/reader/full/sbi-po-2013-preparation-guide 52/169

SBI PO 2013 Preparation Guide Page 52 

www.jagranjosh.com Page 52 

SBI PO 2013 Preparation Guide 

5.  A large number of engineering graduates in the country are not in a position to have gainful

employment at present and the number of such engineers is likely to grow in the future.

I.  The government should launch attractive employment generation schemes and

encourage these graduates to opt for such schemes to use their expertise and

knowledge effectively.

II.  This happened due to proliferation of engineering colleges in the country and thereby

lowered the quality of the engineering graduates. Those colleges which are not

equipped to impart quality education should be closed down.

6.  A large number of people in ward X of the city are diagnosed to be suffering from a fatal

malaria type.

I.  The city municipal authority should take immediate steps to carry out extensive

fumigation in ward X.

II.  The people in the area should be advised to take steps to avoid mosquito bites.

7.  There have been many instances of derailment of trains due to landslide in the hilly areas

which caused loss of many lives.

I.  The railway authority should arrange to deploy pilot engines before the movement of 

passenger trains in the hilly areas to alert the trains in case of any landslide.

II.  The railway authority should strengthen the hill slopes by putting iron meshes so that

the loose boulders do not fall on the track.

8.  Footpaths of a busy road are crowded with vendors selling cheap items.

I.  The help of police should be sought to drive them away.

II.  Some space should be provided to them where they can earn their bread without

blocking footpaths.

9.  It is necessary to adopt suitable measures to prevent repetition of bad debts by learning

from the past experiences of mounting non- performing assets of banks.

I.  Before granting loan to customers their eligibility for loan should be evaluated strictly.

II.  To ensure the payment of installments of loan, the work, for which loan was granted,should be supervised minutely on regular basis.

10.  The sale of a particular product has gone down considerably causing great concern to the

company.

I.  The company should make a proper study of rival products in the market.

Page 53: Sbi Po 2013 Preparation Guide

7/30/2019 Sbi Po 2013 Preparation Guide

http://slidepdf.com/reader/full/sbi-po-2013-preparation-guide 53/169

SBI PO 2013 Preparation Guide Page 53 

www.jagranjosh.com Page 53 

SBI PO 2013 Preparation Guide 

II.  The price of the product should be reduced and quality improved.

11.  Exporters in the capital are alleging that commercial banks are violating a Reserve Bank of 

India directive to operate a post shipment export credit denominated in foreign currency at

international rates from January this year.

I.  The officers concerned in the commercial banks are to be suspended.

II.  The RBI should be asked to stop giving such directives to commercial banks.

12. The police department has come under a cloud with recent revelations that at least two

senior police officials are suspected to have been involved in the illegal sale of a large

quantity of weapons from the state police armory.

I.  A thorough investigation should be ordered by the State Government to bring out all

those who are involved into the illegal sale of arms.

II.  State police armoury should be kept under Central Government's control.

13. Every year large number of devotees die due to severe cold on their way to the shrine

located at the top of the mountain range.

I.  The devotees should be discouraged to visit the shrine without having proper warm

clothes and other amenities.

II.  The government should provide warm clothes and shelter to all the devotees visiting the

shrine.

14. It is reported that though Vitamin E present in fresh fruits and vegetables is beneficial forhuman body, capsuled Vitamin E does not have the same effect on human body.

I.  The sale of capsuled Vitamin E should be banned.

II.  People should be encouraged to take fresh fruits and vegetables to meet the body's

requirement of Vitamin E.

15. A large number of people visiting India from country X have been tested positive for

carrying viruses of a killer disease.

I.  The government of India should immediately put a complete ban on people coming to

India from country X including those Indians who are settled in country X.

II.  The government of India should immediately set up detection centres at all its airports

and seaports to identify and quarantine those who are tested positive.

16. The Minister said that the teachers are still not familiarized with the need, importance and

meaning of population education in the higher education system. They are not even clearly

aware about their role and responsibilities in the population education program.

Page 54: Sbi Po 2013 Preparation Guide

7/30/2019 Sbi Po 2013 Preparation Guide

http://slidepdf.com/reader/full/sbi-po-2013-preparation-guide 54/169

SBI PO 2013 Preparation Guide Page 54 

www.jagranjosh.com Page 54 

SBI PO 2013 Preparation Guide 

I.  Population education programme should be included in the college curriculum.

II.  Orientation programme should be conducted for teachers on population education.

17. Mr. X, an active member of the Union, often insults his superiors in the office with his rude

behavior.

I.  He should be transferred to some other department.

II.  The matter should be referred to the Union.

18. As stated in the recent census report the female to male ratio is alarmingly low.

I.  The government should conduct another census to verify the results.

II.  The government should immediately issue orders to all the departments to encourage

people to improve the ratio.

19. There has been large number of cases of internet hacking in the recent months creating

panic among the internet users.

I.  The government machinery should make an all out effort to nab those who are

responsible and put them behind bars.

II.  The internet users should be advised to stay away from using internet till the culprits are

caught.

20. Some serious blunders were detected in the Accounts section of a factory.

I.  An efficient team of auditors should be appointed to check the Accounts.

II.  A show cause notice should be issued to all the employees involved in the irregularity.

Answers:-

1.  (a)

2.  (a)

3.  (a)

4.  (d)

5.  (b)

6.  (e)

7.  (c)

8.  (e)

9.  (e)

10. (a)

11. (d)

12. (a)

13. (a)

14. (b)

15. (b)

16. (b)

17. (d)

18. (b)

19. (a)

20. (e)

Answer Explanations:

1.  Ans. (a) clearly, proper supervision alone can see the development in practice. So, only

course I follows.

Page 55: Sbi Po 2013 Preparation Guide

7/30/2019 Sbi Po 2013 Preparation Guide

http://slidepdf.com/reader/full/sbi-po-2013-preparation-guide 55/169

SBI PO 2013 Preparation Guide Page 55 

www.jagranjosh.com Page 55 

SBI PO 2013 Preparation Guide 

2.  Ans. (a) clearly, one complaint is enough for a wrong doing. This should be confirmed by

catching the guilty red-handed and then strict action taken against him. So, only course I

follows.

3.  Ans. (a) Airlines, being convenient and faster means of transport, people would surely

prefer it to the railways if there is a marginal difference between the fares. Hence, a

considerable gap between the two fares is a must for the railways. So, course I follows.

Following course II would reduce the volume of passengers. Hence, II does not follow.

4.  Ans. (d) the ideas suggested in both I and II represent unfair means to cut competition. The

correct way would be to devise methods and techniques such that the indigenous

producers could produce better quality fruits and make them available in the market at

prices comparable with those of the imported ones. Hence, neither I nor II follows.

5.  Ans. (b) the emphasis should be not on the Government putting all the engineering

graduates to jobs but on the colleges producing not 'degree-holders' but real technical

minds which could compete well for gainful employment. So, only course II follows.

6.  Ans. (e) clearly, prevention from mosquitoes and elimination of mosquitoes are two ways to

prevent malaria. So, both the courses follow.

7.  Ans. (c) clearly, either something should be done to alert the trains well in advance in case

of a landslide or some means should be adopted to prevent blockage of tracks during

landslides. Thus, either I or II follows.

8.  Ans. (e) crowding on footpaths is a great inconvenience for walkers. So, stern action needs

to be taken to remove the vendors. But at the same time these people ought to be provided

alternative means of livelihood. So, both the courses follow.

9.  Ans. (e) to ensure that debts taken are repaid promptly, the customers' requirements and

future prospects ought to be studied and their work constantly checked. Thus, both the

courses follow.

10. Ans. (a) clearly, a study of rival products in the market will help assess the cause for the

lowering down of sales and then a suitable action can be taken. Thus, only I follows.

11. Ans. (d) the statement mentions that the commercial banks violate a directive issued by the

RBI. The remedy is only to make the banks implement the Act. So, none of the courses

follows.

12. Ans. (a) clearly, the situation demands finding out the real culprits first. So, only I follows.

Page 56: Sbi Po 2013 Preparation Guide

7/30/2019 Sbi Po 2013 Preparation Guide

http://slidepdf.com/reader/full/sbi-po-2013-preparation-guide 56/169

SBI PO 2013 Preparation Guide Page 56 

www.jagranjosh.com Page 56 

SBI PO 2013 Preparation Guide 

13. Ans. (a) clearly, the problem can be solved by warning the devotees of the excessive cold at

the shrine. So, only I follow.

14. Ans. (b) the statement implies that capsule Vitamin E does not function so effectively as

natural Vitamin E. Since no negative effect of capsule Vitamin E is mentioned, so I does not

follow. Hence, only II follows.

15. Ans. (b) clearly, the non-infected persons should not be debarred from visiting India. So,

only course II follows.

16. Ans. (b) clearly, the statement stresses on teachers' lack of awareness and knowledge in

population education and as such the best remedy would be to guide them in this field

through orientation programs. So, only course II follows.

17. Ans. (d) clearly, the only remedy is to somehow attempt to change the habit. If transferred,

the habit will create problem elsewhere. Also, it is no legal complaint to be referred to theUnion. So, neither of the courses follow.

18. Ans. (b) a census is always conducted with the utmost precision, leaving chances of only

negligible differences. So, I does not follow. Further, the ratio can be improved by creating

awareness among the masses and abolishing female foeticide. Thus, only course II follows.

19. Ans. (a) clearly, internet users should not suffer on account of certain individuals who

indulge in internet hacking. However, such wrong-doers ought to be penalised so that there

are no hassles in the use of internet. So, only course I follows.

20. Ans. (e) clearly, the situation demands that the faults in Accounts be properly worked out

and the persons involved be interrogated about the matter. So, both the courses follow.

CAUSE AND EFFECT

Introduction: In the SBI PO exam, there can be questions based on the Cause and Effect

relationship and the candidates are asked to identify the relationship. These type of questions

are asked to test the candidate’s analytical skill. 

Here, we will practice few questions on the Cause and Effect relationship.

Practice Questions

Directions: Below in each question are given two statements (I) and (II). These statements may

be either independent causes or may be effects of independent causes or a common cause.

One of these statements may be the effect of the other statement. Read both the statements

Page 57: Sbi Po 2013 Preparation Guide

7/30/2019 Sbi Po 2013 Preparation Guide

http://slidepdf.com/reader/full/sbi-po-2013-preparation-guide 57/169

SBI PO 2013 Preparation Guide Page 57 

www.jagranjosh.com Page 57 

SBI PO 2013 Preparation Guide 

and decide which of the following answer choice correctly depicts the relationship between

these two statements.

Mark (A) If statement I is the cause and statement II is its effect.

Mark (B) If statement II is the cause and statement I is its effect.

Mark (C) If both the statements I and II are independent causes.

Mark (D) If both the statements I and II are effects of independent causes.

Mark (E) If both the statements I and II are effects of some common causes. 1. 

I.  Many schools in the district are closed down this year.

II.  Severe draught situation gripped the state resulting into acute shortage of drinking

water.

2. I.  A ban on the use of plastic products has been imposed.

II.  Small scale units producing plastic products are on the verge of closure.

3. I.  Police had launched a crackdown on all the criminal activities in the locality last

month.

II.  There has been a significant decline in the cases of criminal activities in the locality.

4. I.  A large number of devotees visited the shrine on Sunday.

II.  Every Sunday special prayers are offered.

5. I.  The government has empowered the Village Panchayats to settle cases of land

disputes in the villages.

II.  There has been significant reduction in the number of criminal cases in the district

court.

6. I. 

The average day temperature of Delhi has increased by about 2°C in the current yearover the average of last 10 years.

II.  More people living in rural areas of the state have started migrating to the urban

areas in comparison to earlier years.

7. 

Page 58: Sbi Po 2013 Preparation Guide

7/30/2019 Sbi Po 2013 Preparation Guide

http://slidepdf.com/reader/full/sbi-po-2013-preparation-guide 58/169

SBI PO 2013 Preparation Guide Page 58 

www.jagranjosh.com Page 58 

SBI PO 2013 Preparation Guide 

I.  Many shopkeepers have closed their shops for the second continuous day.

II.  Two groups of people have been fighting with each other with bricks and stones and

forcing people to stay indoors.

8. I.  The Govt has decided to increase the prices of LPG gas cylinders with immediate

effect.

II.  The govt has decided to increase the price of petrol and diesel with immediate

effect.

9. I.  A cyclone is expected to hit the coastal areas within next 24 hours.

II.  Warning has been issued that heavy rains are expected in the coastal areas in the

next 24 hours.

10. I.  A majority of the first year students failed in Mathematics.

II.  The management has terminated the service of the contractual mathematics

teacher.

11. I.  Taliban is now trying to capture other areas, after its success in Swat valley.

II.  Pakistan government is decided to fight against the Taliban.

12. I.  There was a huge rush of people to the temple on 15

th

and 16th

of the month.II.  The temple authority had decided to close down the temple for repairs from 17th of 

the month.

13. I.  Large number of people living in the low lying areas has been evacuated during the

last few days to safer places.

II.  The Government has rushed in relief supplies to the people living in low lying areas.

14. I.  The Reserve Bank of India has recently put restrictions on few small banks in the

country.

II.  The small banks in the private and co-operative sector in India are not in a position

to withstand the competitions of the bigger banks in the public sector.

15. 

Page 59: Sbi Po 2013 Preparation Guide

7/30/2019 Sbi Po 2013 Preparation Guide

http://slidepdf.com/reader/full/sbi-po-2013-preparation-guide 59/169

SBI PO 2013 Preparation Guide Page 59 

www.jagranjosh.com Page 59 

SBI PO 2013 Preparation Guide 

I.  The prices of petrol and diesel in the domestic market have remained unchanged for

the past few months.

II.  The crude oil prices in the International market have gone up substantially in the last

few months.

16. 

I.  The performance of Indian sports persons in the recent Olympics could not reach

the level of expectation the country had on them.

II.  The performance of Indian sports person in the last Asian games was far better than

any previous games.

17. I.  The university officers have decided to conduct last examination every year in

March/April in order to announce the result at proper time.

II.  In past, the result was declared late by the University due to the lack of number of examiners.

18. I.  The State Government has announced special tax package for the new industries to

be set-up in the State.

II.  Last year the State Government had hiked the taxes for all industrial activities in the

State.

19. I.  The government has allowed private airline companies in India to operate to

overseas destinations.

II.  The national air carrier has increased its flights to overseas destinations.

20. I.  The Government has imported large quantities of groundnut from other countries.

II.  The price of groundnut in the domestic market has fallen sharply.

Answers:-

1.  (d)

2.  (a)

3.  (a)

4.  (b)

5.  (d)

6.  (d)

7.  (b)

8.  (e)

9.  (a)

10. (a)

11. (a)

12. (b)

13. (e)

14. (b)

15. (c)

16. (e)

17. (b)

18. (b)

19. (a)

20. (a)

Answer Explanations:-

Page 60: Sbi Po 2013 Preparation Guide

7/30/2019 Sbi Po 2013 Preparation Guide

http://slidepdf.com/reader/full/sbi-po-2013-preparation-guide 60/169

SBI PO 2013 Preparation Guide Page 60 

www.jagranjosh.com Page 60 

SBI PO 2013 Preparation Guide 

1.  Ans. (d) there is no relation between schools and draught and drinking water. Both the

statements are the effects of independent causes.

2.  Ans. (a) clearly, statement I is the cause and statement II is its effect.

3.  Ans. (a) clearly, decline in criminal activity is the result of Police activity.

4.  Ans. (b) devotees visited shrine on Sunday because special prayers are offered every

Sunday.

5.  Ans. (d) there cases of land are civil cases, and village panchayats have been empowered to

settle civil cases; and there is no link between civil and criminal cases. Both statements are

independent effects.

6.  Ans. (d) the increase in temp is due to some reason say climate change or pollution but that

has no link to migration of people. They might be migrating because of n number of reasons

poverty, jobs, social issues, disease… So clearly I & II are effects whose causes are

independent.

7.  Ans. (b) clearly because of chaos the shopkeeper have been forced to stay indoors and

cannot open shops. So the cause is the conflict and effect is closing of shops.

8.  Ans. (e) Govt has decided to raise the price of LPG, Diesel and Petrol also but reason is not

given here but surely the reason will be same because they all belong to same category  – 

i.e. fuels.

9.  Ans. (a) Cyclone is expected to hit the coastal region so warning has been issued that there

might be heavy rains.

10. Ans. (a) since all the students failed, the Management no longer wanted this teacher, so it

terminated its services. Again Because of statement I(cause), statement II was taken into

action (effect).

11. Ans. (a) clearly, Pakistan’s action (Effect) is a result of Taliban’s activity (Cause). 

12. Ans. (b) since the temple authority had decided to close down the temple for repairs from

17th of the month, hence, there was a huge rush of people to the temple on 15 th & 16th of the month.

13. Ans. (e) clearly, both the statements are effects of some common cause.

Page 61: Sbi Po 2013 Preparation Guide

7/30/2019 Sbi Po 2013 Preparation Guide

http://slidepdf.com/reader/full/sbi-po-2013-preparation-guide 61/169

SBI PO 2013 Preparation Guide Page 61 

www.jagranjosh.com Page 61 

SBI PO 2013 Preparation Guide 

14. Ans. (b) as, the small banks in the private and co-operation sector in India are not in a

position to withstand the competitions of the bigger banks in the public sector, therefore,

the Reserve Bank of India has recently put restrictions on few small banks in the country.

15. Ans. (c) clearly, both are independent causes.

16. Ans. (e) both the statements are effects of some common cause as both are talking of 

performance of Indian Sports Persons.

17. Ans. (b) since in the past the result was declared late by university, it has decided to

conduct the examination in March/April in order to announce the result at proper time.

18. Ans. (b) due to hike of taxes last year, the State Government has announced special tax

package the new industries to be set up in the State.

19. Ans. (a) when private airlines will operate overseas, the frequency of national flights to

overseas destinations will increase.

20. Ans. (a) since the Government has imported large quantities of groundnut, its price has

fallen.

Page 62: Sbi Po 2013 Preparation Guide

7/30/2019 Sbi Po 2013 Preparation Guide

http://slidepdf.com/reader/full/sbi-po-2013-preparation-guide 62/169

SBI PO 2013 Preparation Guide Page 62 

www.jagranjosh.com Page 62 

SBI PO 2013 Preparation Guide 

Page 63: Sbi Po 2013 Preparation Guide

7/30/2019 Sbi Po 2013 Preparation Guide

http://slidepdf.com/reader/full/sbi-po-2013-preparation-guide 63/169

SBI PO 2013 Preparation Guide Page 63 

www.jagranjosh.com Page 63 

SBI PO 2013 Preparation Guide 

DATA ANALYSIS AND INTERPRETATION

TABLE CHARTS

Introduction: The questions on table charts in the State Bank of India (SBI) Probationary

Officers exams have been asked frequently. After the study of the previous year question

papers we can say the data interpretation section will consist of 2  – 3 table charts with 10  – 15

questions. It is advisable to every student to understand the concept of table chart so that they

can earn good marks in the section.

Table Charts: These are the charts of facts and figures shown in horizontal rows and vertical

columns. A table chart can contain facts and figures related to cities, states, countries, time

periods, individual names, companies and miscellaneous information.

The questions on the basis of the table charts are generally asked to find the relation among

the facts and the figures. The question can be asked to find average, ratio or percentage of 

some facts and figures related to some other facts and figures.

Solved Example;

Directions (Q. 1 – 5): Study the following table carefully to answer the questions that follow:

Numbers (N) of five types of FMCG products sold by six different stores in a month and the

price per product (P) (price in Rs 000) charged by each store.

Store A B C D E

Product N P N P N P N P N P

P 56 125 58 122 60 124 61 128 42 126

Q  73 3.5 43 4.8 55 5.6 59 3.9 57 4.5

R 58 13 57 19 52 13 57 12 64 15

S 52 55 55 52 51 54 54 59 49 57

T 62 75 63 76 58 82 55 88 49 86

1.  The number of P type products sold by Store E is what per cent of the number of the same

type of products sold by Store C?

(a)  80

(b) 75

(c)  70

Page 64: Sbi Po 2013 Preparation Guide

7/30/2019 Sbi Po 2013 Preparation Guide

http://slidepdf.com/reader/full/sbi-po-2013-preparation-guide 64/169

SBI PO 2013 Preparation Guide Page 64 

www.jagranjosh.com Page 64 

SBI PO 2013 Preparation Guide 

(d) 125

(e)  None of these

Answer: (c) here, type P from Store E is 42 and from Store C is 60. Therefore,

2.  What is the ratio of the total number of P and Q type products together sold by Store C and

that of R and T type of products sold by Store B?

(a) 4 : 5

(b) 5 : 4

(c) 24 : 23

(d) 23 : 24

(e) None of these

Answer: (d) the type P and Q product from store C are 60 and 55 respectively and the type R

and T from store B are 57 and 63. Therefore, the required ratio is 115 : 120 = 23 : 24. 

3.  What is the average price charged by all the Stores together for the product T?

(a)  Rs. 81,400

(b) Rs. 80,600

(c)  Rs. 81,600

(d) Rs. 80,400

(e)  None of these

Answer:  (a) the total price for product T from all the stores is Rs. 4,07,000. Therefore the

average =

4.  What is the difference in the amount earned by Store A through sale of P type products and

that earned by Store C through the sale of S type products?

(a)  Rs. 4.246 lakhs

(b) Rs. 42.46 lakhs

(c)  Rs. 424.6 lakhs

(d) Rs. 0.4246 lakhs

(e)  None of these

Answer: (b) the amount earned on P from store A = 56 × (Rs. 1,25,000) = Rs. 70,00,000 and the

amount earned on S from store C = 51 × (Rs. 54,000) = Rs. 27,54,000.

Therefore the difference = Rs. 70,00,000 – Rs. 27,54,000 = Rs. 42,46,000. 

5.  What is the total amount earned by Store D through the sale of S and T type products

together?

(a)  Rs. 8,026 Lakhs

Page 65: Sbi Po 2013 Preparation Guide

7/30/2019 Sbi Po 2013 Preparation Guide

http://slidepdf.com/reader/full/sbi-po-2013-preparation-guide 65/169

SBI PO 2013 Preparation Guide Page 65 

www.jagranjosh.com Page 65 

SBI PO 2013 Preparation Guide 

(b) Rs. 802.6 Lakhs

(c)  Rs. 8.026 Lakhs

(d) Rs. 0.8026 Lakhs

(e)  None of these

Answer: (e) the amount earned by Store D through the sale of S and T type products is;54 × (Rs. 59,000) + 55 × (Rs. 88,000) = Rs. 31,86,000 + Rs. 48,40,000 = Rs. 80,26,000. 

Practice Questions

Direction (Q. 1 – 5): Study the given table carefully and answer the questions that follow:

Percentage of marks obtained by five students in five different subjects in a school

SUBJECT English Hindi Science Mathematics Social Studies

STUDENT (100) (50) (125) (150) (75)

Abhinav 59 76 92 90 64Brajesh 78 80 72 70 84

Chandan 75 74 80 86 80

Dhruv 71 76 68 78 68

Eeshita 73 72 88 92 80

Figures in brackets indicate maximum marks for a particular subject.

1.  What is the average mark obtained by all the students together in Science?

(a)  92

(b) 98

(c)  100

(d) 102

(e)  105

2.  What is Chandan’s overall percentage in the examination? 

(a)  80.0

(b) 80.2

(c)  80.4

(d) 80.6(e)  80.8

3.  What is the ratio of the total marks obtained by Chandan in Science to the marks obtained

by Abhinav and Eeshita in Social Science?

(a)  25 : 24

Page 66: Sbi Po 2013 Preparation Guide

7/30/2019 Sbi Po 2013 Preparation Guide

http://slidepdf.com/reader/full/sbi-po-2013-preparation-guide 66/169

SBI PO 2013 Preparation Guide Page 66 

www.jagranjosh.com Page 66 

SBI PO 2013 Preparation Guide 

(b) 24 : 25

(c)  27 : 25

(d) 25 : 27

(e)  None of these

4.  If in order to pass the exam a minimum of 102 marks is needed in Science, how many

students pass the exam?

(a)  Three

(b) One

(c)  Two

(d) Zero

(e)  More than three

5.  Who got the maximum marks among these five students?

(a)  Eeshita

(b) Dhruv

(c)  Chandan

(d) Brajesh

(e)  Abhinav

Direction (Q. 6 – 10): Study the given table carefully and answer the questions that follow:

A survey of online shopping habits of city dwellers from 5 cities A, B, C, D and E is given below.

The first column gives the percentage of people who do online purchasing less than two times a

month. The second column gives the total number of people who do online purchasing two or

more times per month.

City I II

A 65% 1,75,000

B 30% 3,85,000

C 86% 1,40,000

D 72% 2,38,000

E 56% 3,30,000

1.  How many people in city D do online purchasing loss than times in a month?

(a)  6,04,000

(b) 6,00,000

(c)  6,12,000

(d) 6,55,000

Page 67: Sbi Po 2013 Preparation Guide

7/30/2019 Sbi Po 2013 Preparation Guide

http://slidepdf.com/reader/full/sbi-po-2013-preparation-guide 67/169

SBI PO 2013 Preparation Guide Page 67 

www.jagranjosh.com Page 67 

SBI PO 2013 Preparation Guide 

(e)  None of these

2.  The city with the lowest number of people who do online shopping less than two times in a

month is

(a)  A

(b) B

(c)  C

(d) D

(e)  None of these

3.  The highest number of people who purchase online less than two times in a month from

any given city (in the survey) is

(a)  8,60,000

(b) 6,12,000

(c)  4,20,000(d) 3,25,000

(e)  None of these

4.  What are the total number of people who purchase online less than two times in a month

from A and B?

(a)  3,25,000

(b) 1,65,000

(c)  8,65,000

(d) 6,12,000

(e)  None of these

10. What is the average number of people who purchase online less than two times from all the

five the cities?

(a)  4,75,000

(b) 4,75,300

(c)  4,76,300

(d) 4,76,400

(e)  None of these

Answers:-

1.  (c)

2.  (b)

3.  (d)

4.  (a)

5.  (e)

6.  (c)

7.  (b)

8.  (a)

9.  (e)

10. (d)

Page 68: Sbi Po 2013 Preparation Guide

7/30/2019 Sbi Po 2013 Preparation Guide

http://slidepdf.com/reader/full/sbi-po-2013-preparation-guide 68/169

SBI PO 2013 Preparation Guide Page 68 

www.jagranjosh.com Page 68 

SBI PO 2013 Preparation Guide 

BAR GRAPHS

Introduction: The questions on Bar Graphs in the State Bank of India (SBI) Probationary Officers

exams have been asked frequently. After the study of the previous year question papers we can

say the data interpretation section will consist of 2 – 3 Bar Graphs with 10  – 15 questions. It isadvisable to every student to understand the concepts of Bar Graphs so that they can earn

good marks in the section.

Bar Graphs: These graphs are one of the oldest methods to present a data. The bar graphs

consist of bars, each of which is a thick box. The value of the reading the bar is determined by

the height of the bar. The bar graphs are generally presented with different colours, shades,

dots, dashes, etc.

The questions on the basis of the Bar Graphs are generally asked to find the relation among

facts and the figures. The question can be asked to find average, ratio or percentage of somefacts and figures related to some other facts and figures.

Solved Example;

Direction (Q. 1 – 5): Study the given graph carefully and answer the questions that follow:

Number of people (in thousands) using three different types of Bank Account P, Q and R over

the years.

1.  What is the average number of people using Bank Account R over the six years together?

(a)  18,000

(b) 20,500

(c)  22,000

Page 69: Sbi Po 2013 Preparation Guide

7/30/2019 Sbi Po 2013 Preparation Guide

http://slidepdf.com/reader/full/sbi-po-2013-preparation-guide 69/169

SBI PO 2013 Preparation Guide Page 69 

www.jagranjosh.com Page 69 

SBI PO 2013 Preparation Guide 

(d) 22,500

(e)  None of these

Answer:- (d)

2.  The total number of people using all the three bank accounts in the year 2008 is

approximately what per cent of the total number of people using all the three bank

accounts in the year 2006?

(a)  180

(b) 175

(c)  154

(d) 150

(e)  None of these

Answer:- (c)

3.  The number of people using bank account Q in the year 2007 forms approximately what percent of the number of people using bank account P in the year 2009?

(a)  55

(b) 58

(c)  60

(d) 65

(e)  None of these

Answer:- (b)

4.  What is the ratio of the number of people using bank account R in the year 2006 to the

number of people using bank account P in the year 2009?

(a)  3 : 4

(b) 4 : 3

(c)  1 : 2

(d) 2 : 1

(e)  None of these

Answer:- (e)

5.  What is the total number of people using bank account P over the six years together?

(a)  1.19 lakhs(b) 11.9 lakhs

(c)  119 lakhs

(d) 0.119 lakhs

(e)  None of these

Answer:- (a)

Page 70: Sbi Po 2013 Preparation Guide

7/30/2019 Sbi Po 2013 Preparation Guide

http://slidepdf.com/reader/full/sbi-po-2013-preparation-guide 70/169

SBI PO 2013 Preparation Guide Page 70 

www.jagranjosh.com Page 70 

SBI PO 2013 Preparation Guide 

Practice Questions

Directions (Q. 1 – 5): study the graphs carefully to answer the questions that follow:

Total number of children in 5 different schools and the percentage of boys in them

1.  Approximately what percentage of girls is in the schools A and F together?

(a)  40

(b) 44

(c)  46

(d) 42

(e)  48

2.  What is the total number of girls in School C and E together?

(a)  200

(b) 225

(c)  210

(d) 217

(e)  219

3.  The number of girls in school B is what percent of the number of girls in school D?

(a)  45

(b) 35

(c)  40(d) 55

(e)  54

4.  What is the average number of boys in the schools C and F?

(a)  183

Page 71: Sbi Po 2013 Preparation Guide

7/30/2019 Sbi Po 2013 Preparation Guide

http://slidepdf.com/reader/full/sbi-po-2013-preparation-guide 71/169

SBI PO 2013 Preparation Guide Page 71 

www.jagranjosh.com Page 71 

SBI PO 2013 Preparation Guide 

(b) 180

(c)  182

(d) 192

(e)  186

5.  What is the ratio of the number of boys in schools D to the number of boys in school E?

(a) 4 : 3

(b) 6 : 5

(c) 36 : 25

(d) 32 : 29

(e) None of these

Answers-

1.  (b) 2.  (d) 3.  (c) 4.  (a) 5.  (e)

LINE GRAPHS

Introduction: The questions on Line graphs or X  – Y Graphs in the State Bank of India (SBI)

Probationary Officers exams have been asked frequently. After the study of the previous year

question papers we can say the data interpretation section will consist 2  – 3 Line Graphs with

10 – 15 questions. It is advisable to every student to understand the concepts of Line Graphs so

that they can earn good marks in the section.

Line Graphs: These graphs are one of the oldest methods to present a data related to time

series and frequency distribution. This type of representation is widely used by news papers,

televisions, government reports, magazines and research papers.

The Line graphs can be;

  Single dependent Variable Graph 

  More than One variable Graph 

  Graphs with two Scale (two continuous variable) 

  Range Graph 

  Band Graph 

  Speed time Graph 

Page 72: Sbi Po 2013 Preparation Guide

7/30/2019 Sbi Po 2013 Preparation Guide

http://slidepdf.com/reader/full/sbi-po-2013-preparation-guide 72/169

SBI PO 2013 Preparation Guide Page 72 

www.jagranjosh.com Page 72 

SBI PO 2013 Preparation Guide 

The questions on the basis of the Line Graphs are generally asked to find the relation among

the facts and the figures. The question can be asked to find average, ratio or percentage of 

some facts and figures related to some other facts and figures.

Solved Example;

Direction (Q. 1 – 5): Study the graph carefully and answer the questions that follow:

Per cent profit made by two companies over the years

1.  If in the year 2006 the expenditures incurred by company P and Q were same, what was the

ratio of the income of company Q to that of company P in that year?

(a)  26 : 27

(b) 27 : 26

(c)  24 : 25

(d) 25 : 24

(e)  None of these

Answer: (c)

2.  If the amount of profit earned by company Q in the year 2007 was Rs. 2.4 lakhs, what was

its expenditure in that year?

(a)  Rs. 13 lakhs

(b) Rs. 15 lakhs

(c)  Rs. 24 lakhs

Page 73: Sbi Po 2013 Preparation Guide

7/30/2019 Sbi Po 2013 Preparation Guide

http://slidepdf.com/reader/full/sbi-po-2013-preparation-guide 73/169

SBI PO 2013 Preparation Guide Page 73 

www.jagranjosh.com Page 73 

SBI PO 2013 Preparation Guide 

(d) Rs. 16 lakhs

(e)  Rs. 20 lakhs

Answer: (d) 

3.  What is the average per cent profit earned by company P over all the years together?

(a)  30

(b) 25

(c)  40

(d) 33

(e)  None of these

Answer: (b)

4.  If in the year 2009, the incomes of both the companies P and Q were same, what was the

ratio of the expenditure of company P to the expenditure of company Q in the same year?

(a)  26 : 23

(b) 23 : 26

(c)  24 : 25

(d) 25 : 24

(e)  None of these

Answer: (a)

5.  What is the ratio of the amount of profit earned by company A to that by company B in the

year 2010?

(a)  27 : 24(b) 24 : 27

(c)  23 : 24

(d) 24 : 23

(e)  None of these Cannot be determined

Answer: (e)

Practice Questions

Direction (Q. 1 – 5): Study the given graph carefully and answer the questions that follow:

The line diagram shows the cost of production and profit of six companies for the year 2011-12.

(The figures are in 'Lakhs').

Revenue = Cost of Production + Profit.

Page 74: Sbi Po 2013 Preparation Guide

7/30/2019 Sbi Po 2013 Preparation Guide

http://slidepdf.com/reader/full/sbi-po-2013-preparation-guide 74/169

SBI PO 2013 Preparation Guide Page 74 

www.jagranjosh.com Page 74 

SBI PO 2013 Preparation Guide 

1.  The ratio of profits of company B and D to the profits of A and E is:

(a)  2 : 3

(b) 10 : 9

(c)  3 : 2

(d) 10 : 7

(e)  None of these

2.  The profit of company C is what percentage of the revenue of company F?

(a)  20%

(b) 25%

(c)  30%

(d) 35%

(e)  None of these

3.  The revenue of company C is how many times of company E's profit?

(a)  5.5

(b) 5.25

(c)  5.75

(d) 5

(e)  None of these

4.  Which company has the maximum percentage of profit?

(a)  C

(b) D

(c)  E

(d) F

(e)  None of these

5.  What is the average profit of the last five companies (B, C, D, E and F)?

(a)  Rs. 500

Page 75: Sbi Po 2013 Preparation Guide

7/30/2019 Sbi Po 2013 Preparation Guide

http://slidepdf.com/reader/full/sbi-po-2013-preparation-guide 75/169

SBI PO 2013 Preparation Guide Page 75 

www.jagranjosh.com Page 75 

SBI PO 2013 Preparation Guide 

(b) Rs. 5,000

(c)  Rs. 50,000

(d) Rs. 4,66,667

(e)  None of these 

Answers:-

1.  (d) 2.  (a) 3.  (c) 4.  (b) 5.  (e)

PIE CHARTS

Introduction: The questions on Pie Chart Graphs in the State Bank of India (SBI) Probationary

Officers exams have been asked frequently. After the study of the previous year question

papers we can say the data interpretation section will consist of 2  – 3 Pie Charts with 10  – 15

questions. It is advisable for every student to understand the concepts of Pie Charts so that

they can earn good marks in the section.

Pie Charts: These charts are specific types of data presentation technique where the data is

represented in the form of a circle. In a Pie Chart, a circle is divided into various sections or

segments such that each sector or segment represents a certain type of data in a certain

percentage, proportion or angle. The total of the angles in a Pie Chart is equal to 360°.

The questions on the basis of the Pie Charts are generally asked to find the relation among the

facts and the figures. The question can be asked to find ratio or percentage of some facts and

figures related to some other facts and figures.

Solved Example;

Direction (Q. 1 – 5): Study the given pie-charts carefully and answer the questions that follow:

Discipline-wise breakup of the number of candidates appeared in Interview and Discipline-wise

breakup of the candidates selected by and organisation.

Total number of candidates appeared in the interview = 6,100 and total number of candidates

selected after interview = 2,100.

Page 76: Sbi Po 2013 Preparation Guide

7/30/2019 Sbi Po 2013 Preparation Guide

http://slidepdf.com/reader/full/sbi-po-2013-preparation-guide 76/169

SBI PO 2013 Preparation Guide Page 76 

www.jagranjosh.com Page 76 

SBI PO 2013 Preparation Guide 

1.  What was the ratio of the number of candidates appeared in interview from Art discipline

and the number of candidates selected from Science disciplines?

(a)  56 : 55

(b) 61 : 56

(c)  56 : 61

(d) 55 : 56

(e)  None of these

Answer: (b)

2.  The total number of candidates appeared in interview from Management and Others

disciplines was what per cent of the number of candidates from Engineering and Art

disciplines?

(a)  66.67

Page 77: Sbi Po 2013 Preparation Guide

7/30/2019 Sbi Po 2013 Preparation Guide

http://slidepdf.com/reader/full/sbi-po-2013-preparation-guide 77/169

SBI PO 2013 Preparation Guide Page 77 

www.jagranjosh.com Page 77 

SBI PO 2013 Preparation Guide 

(b) 75

(c)  80

(d) 133.33

(e)  150

Answer: (a)

3.  What was the difference between the number of candidates selected from Art discipline

and the number of candidates selected from others discipline?

(a)  21

(b) 210

(c)  2100

(d) 84

(e)  None of these

Answer: (d)

4.  From which discipline was the difference in number of candidates selected to number of 

candidates appeared in interview the maximum?

(a)  Management

(b) Engineering

(c)  Science

(d) Art

(e)  Commerce

Answer: (c) 

5.  What was the total number of candidates selected from Commerce and Art discipline

together?

(a)  1,000

(b) 840

(c)  1,050

(d) 630

(e)  924

Answer: (e)

Practice Questions

Direction (Q. 1 – 5): Study the given pie-charts carefully and answer the questions that follow:

Discipline-wise breakup of the number of candidates appeared in Interview and Discipline-wise

breakup of the candidates selected by and organisation.

Page 78: Sbi Po 2013 Preparation Guide

7/30/2019 Sbi Po 2013 Preparation Guide

http://slidepdf.com/reader/full/sbi-po-2013-preparation-guide 78/169

SBI PO 2013 Preparation Guide Page 78 

www.jagranjosh.com Page 78 

SBI PO 2013 Preparation Guide 

Total number of candidates appeared in the interview = 25,600 and total number of candidates

selected after interview = 7,500.

1.  What was the ratio of the number of candidates appeared in interview from other

disciplines and the number of candidates selected from art disciplines?

(a)  256 : 125

(b) 125 : 256

(c) 

125 : 216(d) Cannot be determined

(e)  None of these

2.  The total number of candidates appeared in interview from Management and Art disciplines

was what per cent of the number of candidates from Engineering discipline?

(a)  66.67

(b) 75

(c)  80

(d) 120

(e)  150

3.  What was the difference between the number of candidates selected from Science

discipline and the number of candidates selected from Commerce discipline?

(a)  1,000

(b) 1,100

Page 79: Sbi Po 2013 Preparation Guide

7/30/2019 Sbi Po 2013 Preparation Guide

http://slidepdf.com/reader/full/sbi-po-2013-preparation-guide 79/169

SBI PO 2013 Preparation Guide Page 79 

www.jagranjosh.com Page 79 

SBI PO 2013 Preparation Guide 

(c)  1,200

(d) 1,250

(e)  None of these

4.  From which discipline was the difference in number of candidates selected to number of 

candidates appeared in interview the maximum?

(a)  Management

(b) Engineering

(c)  Commerce

(d) Science

(e)  Art

5.  What was the total number of candidates selected from Commerce and Art discipline

together?

(a)  1,800(b) 1,950

(c)  2,100

(d) 2,250

(e)  2,400

Answers:-

1.  (a) 2.  (e) 3.  (c) 4.  (d) 5.  (b)

CASE LETS

Introduction: The questions on Case based Data in the State Bank of India (SBI) Probationary

Officers exams have been asked frequently. After the study of the previous year question

papers we can say the data interpretation section will consist of 2  – 3 Cases with 10  – 15

questions. It is advisable to every student to understand the concept of Case base data so that

they can earn good marks in the section.

Case Based Data: These types of data are presented in the competitive exams to test the

candidate’s interest in numbers, for variables and their inter -relationship and ability to convert

the data into a more meaningful form of information.

These things should be kept in mind when you are solving questions of Cases.

  First read and indentify the variables in the data and respective inter-relationship.

  Chart out the data and fill the data in a more meaningful format.

Page 80: Sbi Po 2013 Preparation Guide

7/30/2019 Sbi Po 2013 Preparation Guide

http://slidepdf.com/reader/full/sbi-po-2013-preparation-guide 80/169

SBI PO 2013 Preparation Guide Page 80 

www.jagranjosh.com Page 80 

SBI PO 2013 Preparation Guide 

  And now, try to solve the questions on the Case based data.

The questions on the basis of the case based data are generally asked to find the relation

among the facts and the figures. The question can be asked to find average, ratio or

percentage of some facts and figures related to some other facts and figures.

Solved Example;

Directions (Q. 6  – 10): Study the given information carefully to answer the questions that

follow:

An organization consists of 3500 employees working in different departments, viz HR,

Marketing, IT, Production and Accounts. The ratio of male to female employees in the

organisation is 3 : 2. 8% of the males work in the HR department. 22% of the female work in the

account department. The ratio of males to females working in the HR department is 3 : 5. One-

seventh of the females work in the IT department. 46% of the males work in the Productiondepartment. The number of females is one-sixth of the males working in the same. The

remaining females work in the Marketing department. The total number of employees working

in the IT department is 375. 22% of the males work in the Marketing department and remaining

work in the Account department.

1.  The number of males working in the Account department forms approximately what per

cent of the total number of males in the organisation?

(a)  6

(b) 8

(c)  10

(d) 11

(e)  12

Answer: (a)

2.  How many females work in Production department?

(a)  140

(b) 200

(c)  180

(d) 160(e)  None of these

Answer: (e)

3.  The total number of employees working in the Account department forms approximately

what per cent of the total number of female employees in the organisation?

Page 81: Sbi Po 2013 Preparation Guide

7/30/2019 Sbi Po 2013 Preparation Guide

http://slidepdf.com/reader/full/sbi-po-2013-preparation-guide 81/169

SBI PO 2013 Preparation Guide Page 81 

www.jagranjosh.com Page 81 

SBI PO 2013 Preparation Guide 

(a) 28

(b) 32

(c) 29

(d) 31

(e) 30

Answer: (d)

4.  The ratio of the numbers of females working in IT department to the numbers of males

working in the same department is

(a)  15 : 8

(b) 1 : 2

(c)  8 : 15

(d) 2 : 1

(e)  7 : 11

Answer: (c)

5.  What is the total number of employees working in the Marketing and Production

departments together?

(a)  1900

(b) 2040

(c)  2020

(d) 2031

(e)  2042

Answer: (b)

Practice Questions

Directions (Q. 1 – 5): Study the given information carefully to answer the questions that follow:

An organization consists of 3200 employees working in different departments, viz HR,

Marketing, IT, Production and Accounts. The ratio of male to female employees in the

organisation is 5 : 3. 12% of the males work in the HR department. 24% of the female work in

the account department. The ratio of males to females working in the HR department is 6 : 11.

One-tenth of the females work in the IT department. 42% of the males work in the Production

department. The number of females 5% of the males working in the same. The remaining

females work in the Marketing department. The total number of employees working in the IT

department is 300. 20% of the males work in the Marketing department and remaining work in

the Account department.

Page 82: Sbi Po 2013 Preparation Guide

7/30/2019 Sbi Po 2013 Preparation Guide

http://slidepdf.com/reader/full/sbi-po-2013-preparation-guide 82/169

SBI PO 2013 Preparation Guide Page 82 

www.jagranjosh.com Page 82 

SBI PO 2013 Preparation Guide 

1.  The number of males working in the IT department forms what per cent of the total number

of males in the organisation?

(a)  5

(b) 10

(c) 

12(d) 15

(e)  18

2.  How many males work in Accounts department?

(a)  840

(b) 400

(c)  220

(d) 240

(e)  None of these

3.  The total number of employees working in the HR department forms what per cent of the

total number of employees in the organisation?

(a) 28

(b) 28.75

(c) 29

(d) 29.75

(e) 30

4.  The number of females working in the Production department forms what per cent of the

total number of males in the organisation?

(a)  21

(b) 12

(c)  5

(d) 3.1

(e)  2.1

5.  What is the total number of employees working in the Marketing and IT departments

together?

(a)  190(b) 200

(c)  220

(d) 240

(e)  42

Page 83: Sbi Po 2013 Preparation Guide

7/30/2019 Sbi Po 2013 Preparation Guide

http://slidepdf.com/reader/full/sbi-po-2013-preparation-guide 83/169

SBI PO 2013 Preparation Guide Page 83 

www.jagranjosh.com Page 83 

SBI PO 2013 Preparation Guide 

Answers:-

1.  (d)  2.  (c)  3.  (b)  4.  (e)  5.  (a) 

PERMUTATIONS

Introduction: In the SBI PO exams the questions based on Permutation havr been asked few

times and from the exam point of view it is very important to understand the basic concepts of 

permutation to solve those questions.

Factorial Notation: The product of n consecutive positive integers beginning with one is

denoted by n! And read as factorial n.

∴ n! = 1 × 2 × 3 ×……. × (n – 1) × (n)

For example, 6! = 1 × 2 × 3 × 4 × 5 × 6 = 720.

0! = 1! = 1.

Permutation: The number of different arrangements which can be made by taking some or all

of the given things or objects at a time is called as permutation.

The symbol for permutation of n different things taking r at a time =n

P r or n P r =

For example,5

P 3 = = .

Here, we will practice few questions to understand more about permutation.

Practice Questions

Directions (Q. 1 – 20): Consider the arrangements of the letters of the word RAINBOW

1.  In how many ways can the letters be arranged?

(a)  5040

(b) 4050

(c)  3040

(d) 8040

(e)  None of these

2.  How many words begin with R?

(a)  720

(b) 360

Page 84: Sbi Po 2013 Preparation Guide

7/30/2019 Sbi Po 2013 Preparation Guide

http://slidepdf.com/reader/full/sbi-po-2013-preparation-guide 84/169

SBI PO 2013 Preparation Guide Page 84 

www.jagranjosh.com Page 84 

SBI PO 2013 Preparation Guide 

(c)  1440

(d) 1080

(e)  None of these

3.  How many words begin with R and ends with W?

(a)  120

(b) 240

(c)  180

(d) 360

(e)  None of these

4.  How many words are there in which R and W are at the end positions?

(a)  120

(b) 180

(c)  210(d) 240

(e)  None of these

5.  How many words are there in which N and B are together?

(a)  720

(b) 360

(c)  540

(d) 1440

(e)  None of these

6.  How many words are there in which I and O are never together?

(a)  1440

(b) 720

(c)  3600

(d) 1440

(e)  None of these

7.  How many words are there in which vowels are never together?

(a)  720

(b) 1440(c)  360

(d) 3600

(e)  None of these

8.  How many words are there in which A is always before I and I is always before O?

Page 85: Sbi Po 2013 Preparation Guide

7/30/2019 Sbi Po 2013 Preparation Guide

http://slidepdf.com/reader/full/sbi-po-2013-preparation-guide 85/169

SBI PO 2013 Preparation Guide Page 85 

www.jagranjosh.com Page 85 

SBI PO 2013 Preparation Guide 

(a)  840

(b) 420

(c)  720

(d) 1080

(e) None of these

9.  How many words are there in which vowels are always before the consonants?

(a)  72

(b) 144

(c)  96

(d) 120

(e)  None of these

10. How many words are there in which first and last letters are vowels?

(a)  360(b) 720

(c)  1440

(d) 2880

(e)  None of these

11. If all the words formed are arranged in a dictionary form, then what is the position of the

word RAINBOW in that dictionary?

(a)  3136

(b) 3631

(c)  3361

(d) 1363

(e)  None of these

12. If no two consonants are together then in how many ways can the letters be arranged?

(a)  441

(b) 420

(c)  360

(d) 144

(e)  None of these

13. How many words can be formed so that the vowels may occupy only even positions?

(a)  567

(b) 144

(c)  576

Page 86: Sbi Po 2013 Preparation Guide

7/30/2019 Sbi Po 2013 Preparation Guide

http://slidepdf.com/reader/full/sbi-po-2013-preparation-guide 86/169

SBI PO 2013 Preparation Guide Page 86 

www.jagranjosh.com Page 86 

SBI PO 2013 Preparation Guide 

(d) 625

(e)  None of these

14. How many different words can be formed so that the vowels occupy odd places?

(a)  676

(b) 625

(c)  343

(d) 576

(e)  None of these

15. In how many ways can the letters be arranged so that only two vowels always remain

together?

(a)  2880

(b) 1440

(c)  3200(d) 4096

(e)  None of these

16. There are 6 books on Physics, 3 on Chemistry and 4 on Biology. In how many ways can these

be placed on a shelf if the books on the same subject are to be together?

(a)  622080

(b) 888000

(c)  222000

(d) 413080

(e)  None of these

17. In how many ways can the letters of the word TAMANNA be arranged?

(a)  120

(b) 420

(c)  840

(d) 720

(e)  None of these

18. There are 3 red. 4 green and 5 pink marbles in a bag. They are drawn one by one and

arranged in a row. Assuming that all the 12 marbles are drawn determine the number of 

different arrangements.

(a)  22770

(b) 27720

(c)  22077

Page 87: Sbi Po 2013 Preparation Guide

7/30/2019 Sbi Po 2013 Preparation Guide

http://slidepdf.com/reader/full/sbi-po-2013-preparation-guide 87/169

SBI PO 2013 Preparation Guide Page 87 

www.jagranjosh.com Page 87 

SBI PO 2013 Preparation Guide 

(d) 27270

(e)  None of these

19. How many numbers of 5 digits can be formed with the digits 0, 2, 3, 4 and 5 if the digits

many repeat?

(a)  2500

(b) 250

(c)  120

(d) 2400

(e)  None of these

20. How many numbers each lying between 9 and 1000 can be formed with the digits 0, 1, 2, 3,

7, 8 (numbers can be repeated)?

(a)  30

(b) 120(c)  210

(d) 90

(e)  None of these

Answers:-

1.  (a)

2.  (a)

3.  (a)

4.  (d)

5.  (d)

6.  (c)

7.  (b)

8.  (a)

9.  (b)

10. (b)

11. (b)

12. (d)

13. (b)

14. (d)

15. (a)

16. (a)

17. (b)

18. (b)

19. (a)

20. (c)

Answer Explanations:

1.  There are 7 letters in the word RAINBOW and each letter is used only once. So all the 7

letters can be arranged in 7! Ways.

7! = 7×6×5×4×3×2×1 = 5040.

2.  If we fix R as the initial letter, then we have to arrange only 6 remaining letters.

Hence required number of permutations = 6!

= 6 × 5 × 4 × 3 × 2 × 1 = 720

3.  If we fix R and W as the first and last letters then we have to arrange only 5 remaining

letters which can be arranged in 5! = 120 ways.

Page 88: Sbi Po 2013 Preparation Guide

7/30/2019 Sbi Po 2013 Preparation Guide

http://slidepdf.com/reader/full/sbi-po-2013-preparation-guide 88/169

SBI PO 2013 Preparation Guide Page 88 

www.jagranjosh.com Page 88 

SBI PO 2013 Preparation Guide 

4.  When R and W are the first and last letters of all the words then we can arrange them in 5!

Ways. Similarly when W and R are the first and last letters of the words then the remaining

letters can be arrange in 5! Ways.

Thus the total number of permutations = 2 × 5!

= 2 × 120 = 240

Alternatively: Except to R and W all the remaining 5 letters can be arranged in 5! Ways and

R and W can be arranged in 2! Ways at the end positions.

The total number of permutations = 2! × 5! = 2×120 = 240

5.  Assume N and B as a single letter, then we have only 6 letters for the arrangement

i.e., A, I, R, O, W, BN

which can be arranged in 6! Ways.

Now B and N can also be mutually arranged in 2! WaysTherefore total number of arrangements = 2! × 6!

= 2 × 720 = 1140

6.  Number of permutations when I and O are together

= 2! × 6! = 1440

(Very similar to the previous question)

Total number of permutations = 7! = 5040

Number of permutations when I and O are not together

= 5040 – 1440 = 3600

7.  There are three vowels viz., A, I and O

First of all arrange all the four consonants which can be done in 4! Ways.

Now we have 5 places available for the three vowels to be filled up, which can be done in5P3 ways.

Thus the total number of arrangements = 5

3

5!4! 24

2!P   

=

8.  First of all we arrange all the 4 consonants in 4! Ways.

Now we have 5 places for the 3 vowels as 1. C1 2. C2 3. C3 4. C4 5.

I.  When all the three vowels AIO are together then 5 places can be filled in 5 ways.

II.  When A and I are together and O is separated from “AI” then the 5 places can be

filled up in 10 ways.

Page 89: Sbi Po 2013 Preparation Guide

7/30/2019 Sbi Po 2013 Preparation Guide

http://slidepdf.com/reader/full/sbi-po-2013-preparation-guide 89/169

SBI PO 2013 Preparation Guide Page 89 

www.jagranjosh.com Page 89 

SBI PO 2013 Preparation Guide 

III.  When I and O are together and A is separated from “IO” then the 5 places can be

filled up in 10 ways.

IV.  When A, I and O are separated from each other then the 5 places can be filled up in

10 ways.

Thus there are total 5 + 10 + 10 + 10 = 35 ways in which 5 places can be filled up such that A

is before I and I is before O. Hence total number of ways in which all the 7 letters of 

RAINBOW can be arranged in which A is before I and I is before O = 4! × 35 = 24 × 35 = 840.

Alternatively: All the 7 letters of the word RAINBOW can be arranged in 7! Ways.

And 3 particular letters (A, I, O) can be arranged in 3! Ways. But the given condition is

satisfied by only 1 cut of 6 ways, hence required number of arrangements.

7!7 6 5 4 840

3!

 

9.  Let all the vowels be in a single packet and all the consonants be in the other packet.

Now all the vowels can be arranged in 3! Ways and all the consonants can be arranged in 4!

Ways.

Hence total number of arrangements = 3! × 4! = 6 × 24 = 144

10. The first and last letters can be arranged in3P2 ways and the remaining letters can be

arranged in 5! Ways.

Hence required number of permutations

=3P2 × 5! = 6 × 120 = 720

11. The correct order of the letters is as follows:

A, B, I, N, O, R, W

Number of words begin with A = 6!

Number of words begin with B = 6!

Number of words begin with I = 6!

Number of words begin with N = 6!

Number of words begin with O = 6!

Number of words begin with RAB = 4!Number of words begin with RAIB = 3!

Now the next word is RAINBOW (it is the first word which begins with RAIN)

So the ranking of the word RAINBOW

= 5 × 6! + 4! + 3! + 1 = 3631

Page 90: Sbi Po 2013 Preparation Guide

7/30/2019 Sbi Po 2013 Preparation Guide

http://slidepdf.com/reader/full/sbi-po-2013-preparation-guide 90/169

SBI PO 2013 Preparation Guide Page 90 

www.jagranjosh.com Page 90 

SBI PO 2013 Preparation Guide 

12. First arrange the 3 vowels which can be done in 3! Ways. Now there are 4 places created by

the 3 vowels which can be filled up by 4 consonants in4P4 ways.

Therefore total number of required permutations

= 3! ×4P4 = 6 × 24 = 144

13. 1 2 3 4 5 6 7  

In order that vowels may occupy even places arrange all the 3 vowels, then arrange all the

four consonants in four places. Thus the required number of arrangements = 3! × 4! = 144

14. 1 2 3 4 5 6 7  

First or all arrange any 3 consonants at even places in 4P3 ways. Now the newly created

four odd places can be filled by the remaining letters which includes 3 vowels and 1

consonants, which can be done in4P4 ways. Hence the required number of permutation

=4P3 ×

4P4 = 24 × 24 = 576

15. First of all arrange all the four consonants R, N, B, W in 4! Ways.

Then there are 5 places to be filled up by the vowels. But any two vowels are always

together then we assume that there are only two vowels which can be filled in 5 places in5P2 ways. But we have to take any two vowels together out of 4 vowels then this can be

done in3P2 ways.

Hence the total number of permutation =5P2 ×

3P2 × 4!

= 20 × 6 × 24 = 2880

16. All the 6 books on Physics can be mutually arranged in 6! Ways.

Similarly 3 books on Chemistry and 4 books on Biology can be arranged in 3! And 4! Ways

respectively.

Besides it the three sets of Physics, Chemistry and Biology also can be arranged in 3! Ways

Required number of permutations = 3! × 6! × 3! × 4! = 622080

17. There are total 7 letters of which A occurs 3 times and N occurs 2 times.

Hence, the required number of ways =7!

4203! .2!

 

18. There are total 3 + 4 + 5 = 12 marbles of which 3 are red (alike), 4 green (alike) and 3 are

pink (alike).

The required number of arrangements12 !

3! 4! 5!

= 27720

Page 91: Sbi Po 2013 Preparation Guide

7/30/2019 Sbi Po 2013 Preparation Guide

http://slidepdf.com/reader/full/sbi-po-2013-preparation-guide 91/169

SBI PO 2013 Preparation Guide Page 91 

www.jagranjosh.com Page 91 

SBI PO 2013 Preparation Guide 

19. Ten thousands place can assume only non-zero digits hence ten thousands place can be

filled up in 4 ways and thousands place can be filled up in 5 ways since repetition is allowed

(and 0 can be filled up in this place). Similarly hundreds, tens and unit place be filled up in 5

ways each.

The required number of numbers = 4 × 5 × 5 × 5 × 5 = 2500

20. The number lying between 9 and 1000 consist of 2 or 3 digits in which repetitions are

allowed.

Case 1. (For two digit numbers)

As the tens place can be filled by only non-zero digits and unit digit can be filled up in 6

ways.

The number of 2 digit numbers = 5 × 6 = 30

Case 2. (For three digit numbers)

Hundreds place can be filled up by only non-zero digits hence it can be done in 5 ways andthe rest of the places i.e., tens and unit places can be filled up in 6 ways each.

The number of 3 digit numbers = 5 × 6 × 6 = 180

Total number of numbers = 30 + 180 = 210

COMBINATIONS

Introduction: In the SBI PO exams the questions based on Combination having been asked few

times and from the exam point of view it is very important to understand the basic concepts of 

Combination to solve these questions.

Combination: The number of different selections or groups which can be made by taking some

or all of a number of given things or objects at a time is called combination.

The number of combinations of n different things taken r at a time =n

C r or n C r .

Here,n

C r =

For example,6

C 3 =

Few important results,n

C 0 =n

C n = 1.

nC 1 =

nC (n – 1) = n.

Here, we will practice few questions to understand more about combination.

Page 92: Sbi Po 2013 Preparation Guide

7/30/2019 Sbi Po 2013 Preparation Guide

http://slidepdf.com/reader/full/sbi-po-2013-preparation-guide 92/169

SBI PO 2013 Preparation Guide Page 92 

www.jagranjosh.com Page 92 

SBI PO 2013 Preparation Guide 

Practice Questions

1.  In a test paper there are total 10 questions. In how many different ways can you choose 6

questions to answer?

(a)  210

(b) 540

(c)  336

(d) 120

(e)  None of these

2.  In the above question if the question number 1 is compulsory in how many ways can you

choose to answer 6 questions in all?

(a)  53

(b) 63

(c)  126(d) 210

(e)  None of these

3.  Deepti has 5 friends. In how many ways can she invite one or more of them to a dinner?

(a)  31

(b) 55 

(c)  13

(d) 25 

(e)  None of these

4.  In how many ways can a committee of 6 members be formed from 7 men and 6 ladies

consisting of 4 men and 2 ladies?

(a)  252

(b) 525

(c)  625

(d) 256

(e)  None of these

5.  A committee of 5 persons is to be formed from a group of 6 gentlemen and 4 ladies. In how

many ways can this be done if the committee is to be included at least one lady?

(a)  123

(b) 113

(c)  246

(d) 945

Page 93: Sbi Po 2013 Preparation Guide

7/30/2019 Sbi Po 2013 Preparation Guide

http://slidepdf.com/reader/full/sbi-po-2013-preparation-guide 93/169

SBI PO 2013 Preparation Guide Page 93 

www.jagranjosh.com Page 93 

SBI PO 2013 Preparation Guide 

(e)  None of these

6.  A committee of 7 persons is to be chosen from 13 persons of whom 6 are Americans and 7

are Indians. In how many ways can the selection be made so as to retain a majority of 

Indians?

(a)  945

(b) 1057

(c)  923

(d) 1056

(e)  None of these

7.  In how many ways can a committee of 4 men and 3 women be appointed from 6 men and 8

women?

(a)  480

(b) 408(c)  420

(d) 840

(e)  None of these

8.  In the previous question what will be the number of committees in which Miss A refuses to

serve if Mr. B is a member?

(a)  210

(b) 420

(c)  630

(d) 720

(e)  None of these

9.  A committee of 3 experts is to be selected out of a panel of 7 persons, three of them are

engineers, three of them are managers and one is both engineer and manager. In how

many ways can the committee be selected if it must have at least an engineer and a

manager?

(a)  33

(b) 22

(c)  11(d) 66

(e)  None of these

10. A committee of 5 persons is to be formed out of 6 gents and 4 ladies. In how many ways

this can be done, when at most two ladies are included?

Page 94: Sbi Po 2013 Preparation Guide

7/30/2019 Sbi Po 2013 Preparation Guide

http://slidepdf.com/reader/full/sbi-po-2013-preparation-guide 94/169

SBI PO 2013 Preparation Guide Page 94 

www.jagranjosh.com Page 94 

SBI PO 2013 Preparation Guide 

(a)  186

(b) 168

(c)  136

(d) 169

(e) None of these

11. Mr. Adi has 18 acquaintances of whom 13 are relatives. In how many ways he may invite 10

guests so that 8 of whom are relatives?

(a)  12870

(b) 22022

(c)  20222

(d) 12780

(e)  None of these

12. If there are 11 players in a cricket team, all of whom shake hands with each other, howmany shake hands take place in the team?

(a)  36

(b) 55

(c)  66

(d) 96

(e)  None of these

13. Amit has a list of 24 friends. He wishes to invite some of them in such a manner that he can

enjoy maximum number of parties, but in each party the number of friends (i.e., invitees)

be same and each party must have different set of persons. Then how many parties can

Amitabh enjoy?

(a)  2704156

(b) 357600

(c)  235763

(d) 270156

(e)  None of these

14. An ice-cream parlor offers only family packs of ice-creams with 11 different flavors. If each

member of a family loves different flavors, the maximum how many such families canpurchase the ice-cream if each family contains equal number of persons?

(a)  246

(b) 462

(c)  123

(d) 11C2 

Page 95: Sbi Po 2013 Preparation Guide

7/30/2019 Sbi Po 2013 Preparation Guide

http://slidepdf.com/reader/full/sbi-po-2013-preparation-guide 95/169

SBI PO 2013 Preparation Guide Page 95 

www.jagranjosh.com Page 95 

SBI PO 2013 Preparation Guide 

(e)  None of these

15. In the previous question what is the maximum possible number of member in a family?

(a)  4

(b) 5

(c)  6

(d) 8

(e)  None of these

16. In a meeting everyone had shaken hands with everyone else; it was found that 66

handshakes were exchanged. How many members were present in the meeting?

(a)  10

(b) 14

(c)  12

(d) 8(e)  None of these

17. A cricket team of 11 players is to be formed from 16 players including 4 bowlers and 2

wicket keepers. In how many different ways can a team be formed so that the team has at

least 3 bowlers and at least one wicket keeper?

(a)  2472

(b) 2274

(c)  2472

(d) 1236

(e)  None of these

18. A cricket team of 11 players is to be formed from 20 players including 6 bowlers and 3

wicket keepers. In how many different ways can a team be formed so that the team contain

exactly 2 wicket keepers and at least 4 bowlers?

(a)  22725

(b) 27225

(c)  22275

(d) 22775

(e)  None of these

19. An urn contains 5 different red and 6 different green balls. In how many ways can 6 balls be

selected so that there are at least two balls of each colour?

(a)  425

(b) 245

Page 96: Sbi Po 2013 Preparation Guide

7/30/2019 Sbi Po 2013 Preparation Guide

http://slidepdf.com/reader/full/sbi-po-2013-preparation-guide 96/169

SBI PO 2013 Preparation Guide Page 96 

www.jagranjosh.com Page 96 

SBI PO 2013 Preparation Guide 

(c)  125

(d) 625

(e)  None of these

20. Find the number of straight lines formed by joining 6 different points on a plane, no three of 

them being collinear.

(a)  21

(b) 15

(c)  16

(d) 24

(e)  None of these

Answers:-

1.  (a)

2.  (c)3.  (a)

4.  (b)

5.  (c)

6.  (b)7.  (d)

8.  (c)

9.  (a)

10. (a)11. (a)

12. (b)

13. (a)

14. (b)15. (c)

16. (c)

17. (a)

18. (b)19. (a)

20. (b)

Answer Explanations:-

1.  Out of 10 questions, 6 questions can be selected in10

C6 ways

 10

C6 = 210

2.  Since question number 1 is compulsory so we have to choose only 5 questions from rest of 

the 9 questions, which can be done in9C5 ways

 9C5 = 126

3.  She may invite one or more friends by selecting either 1 or 2 or 3 or 4 or 5 friends out of 5

friends.

1 friend can be selected out of 5 in5C1 ways

2 friends can be selected out of 5 in5C2 ways

3 friends can be selected out of 5 in5C3 ways

4 friends can be selected out of 5 in5C4 ways

5 friends can be selected out of 5 in5C5 ways

Hence the required number of ways

=5C1 +

5C2 +

5C3 +

5C4 +

5C5 

= 5 + 10 + 10 + 5 + 1 = 31

Alternatively: 5C1 +

5C2 +

5C3 +

5C4 +

5C5 = 2

5  – 1 = 3 !

Page 97: Sbi Po 2013 Preparation Guide

7/30/2019 Sbi Po 2013 Preparation Guide

http://slidepdf.com/reader/full/sbi-po-2013-preparation-guide 97/169

SBI PO 2013 Preparation Guide Page 97 

www.jagranjosh.com Page 97 

SBI PO 2013 Preparation Guide 

Since,nC1 +

nC2 +

nC3 + …. +

nCn = 2

n  –1

4.  A men can be selected out of 7 men in7C4 ways

And ladies can be selected out of 6 ladies in6C2 ways

Hence, the required number of ways =7C4 ×

6C2 = 35 × 15 = 525

5.  A committee of 5 persons is to be formed from 6 gentlemen and 4 ladies by taking.

a.  1 lady out of 4 and 4 gentlemen out of 6.

b.  2 ladies out of 4 and 3 gentlemen out of 6.

c.  3 ladies out of 4 and 2 gentlemen out of 6.

d.  4 ladies out of 4 and 1 gentlemen out of 6.

In case I the number of ways =4C1 ×

6C4 = 4×15 = 60

In case II the number of ways =4C

6C

3= 6×20 = 120

In case III the number of ways =4C3 ×

6C2 = 4×15 = 60

In case IV the number of ways =4C4 ×

6C1 = 1×6 = 6

Hence, the required number of ways = 60 + 120 + 60 + 6 = 246

6.  A committee of 7 persons retaining a majority of Indians can be made from 6 Americans

and 7 Indians by taking.

a.  1 American out of 6 and 6 Indians out of 7.

b.  2 Americans out of 6 and 5 Indians out of 7.

c.  3 Americans out of 6 and4 Indians out of 7.

In case I the number of ways =6C1 ×

7C6 = 6 × 7 = 42

In case II the number of ways =6C2 ×

7C5 = 15 ×21 = 315

In case III the number of ways =6C3 ×

7C4 = 20 × 35 = 300

Hence, the required number of ways = 700 + 315 + 42 = 1057

7.  Required number of ways =6C4 ×

8C3 = 15 × 56 = 840

8.  The number of committees in which both Miss. A and Mr. B are always members. We can

select rest 3 men from the remaining 5 in5C3 ways and rest 2 women from the remaining 7

in7

C2 ways.

The required number of ways in which both Miss. A and Mr. B are always included

=5C3 ×

7C2 = 10 × 21 = 210

Page 98: Sbi Po 2013 Preparation Guide

7/30/2019 Sbi Po 2013 Preparation Guide

http://slidepdf.com/reader/full/sbi-po-2013-preparation-guide 98/169

SBI PO 2013 Preparation Guide Page 98 

www.jagranjosh.com Page 98 

SBI PO 2013 Preparation Guide 

Hence the required number of ways in which Miss. A and Mr. B do not serve together =

840 –210 = 630

9.  3 experts including at least an engineer and a manager can be selected by taking.

a.  2 managers out of 3 and 1 engineer out of 3.

b.  1 manager out of 3 and 2 engineer out of 3.

c.  2 persons out of 6 (3 managers and 3 engineers) and 1 person out of one who is both

engineer and manager.

In case I, the number of ways =3C2 ×

3C1 = 9

In case II, the number of ways =3C1 ×

3C2 = 9

In case III, the number of ways =6C2 ×

1C1 = 15

Hence, the required number of ways = 9 + 9 + 15 = 33.

10. A committee of 5 persons, consisting of at most two ladies, can be formed in the following

ways.

a.  Selecting 5 gents only out of 6.

b.  Selecting 4 gents only out of 6 and one lady out of 4.

c.  Selecting 3 gents only out of 6 and two ladies out of 4.

In case I, the number of ways =6C5 

In case II, the number of ways =6C4 ×

4C1 

In case III, the number of ways =6C3 ×

4C2 

Required number of ways =6C5 +

6C4 ×

4C1 +

6C3 ×

4C2 

= 6 + 60 + 120 = 186

11. Required number of ways =13

C8 ×5C2 = 1287 × 10 = 12870

12. One hand shake requires two different persons.

Number of hand shakes

= number of ways of selecting two persons out of 11 persons

=11

C2 = 55

13. If n is even,nCr is maximum when r =

2

Number of invitees in a party 24 122

 

And maximum possible number of parties =24

C12 = 2704156

14. If n is odd,nCr is maximum when

 – 1

2

nr  or

1

2

nr 

 

Page 99: Sbi Po 2013 Preparation Guide

7/30/2019 Sbi Po 2013 Preparation Guide

http://slidepdf.com/reader/full/sbi-po-2013-preparation-guide 99/169

SBI PO 2013 Preparation Guide Page 99 

www.jagranjosh.com Page 99 

SBI PO 2013 Preparation Guide 

 11– 1

52

r  or11 1

62

 

 11

C5 =11

C6 = 462

15. Possible number of members in a family is either 5 or 6.

So, the maximum possible number of family members = 6

16. Let there were n persons in the meeting, then number of handshakes =nC2 

 nC2 = 66

 !

662!( – 2)!

n

n

 

 ( – 1)

662

n n  

n (n –1) = 132

n = 12

Alternatively: Go through options.

17. Total number of bowlers = 4

Total number of wickets keepers = 2

Rest (normal) players = 10

Possible Combinations:

Bowlers Wicket

Keepers

Normal

Players

3 1 73 2 6

4 1 6

4 2 5

Required number of ways

= (4C3 ×

2C1 ×

10C7) + (

4C3 ×

2C2 ×

10C6) + (

4C4 ×

2C1 ×

10C6) + (

4C4 ×

2C2 ×

10C5)

= (3×1×7) + (3×2×6) + (4×1×6)+ (4×2×5)

= 960 + 840 + 420 + 252 = 2472

18. Total number of bowles = 6Total number of wicket keepers = 3

Total number of normal players = 11 [20 –(6+3)]

Possible combinations:

Page 100: Sbi Po 2013 Preparation Guide

7/30/2019 Sbi Po 2013 Preparation Guide

http://slidepdf.com/reader/full/sbi-po-2013-preparation-guide 100/169

SBI PO 2013 Preparation Guide Page 100 

www.jagranjosh.com Page

100 

SBI PO 2013 Preparation Guide 

Bowlers Wicket

Keepers

Normal

Players

4 2 5

5 2 4

6 2 3

Required number of ways

= (6C4 ×

3C2 ×

11C5) + (

6C5 ×

3C2 ×

11C4) + (

6C6 ×

3C2 ×

11C3)

= 20790 + 5940 + 495 = 27225

19. Total number of red balls = 5

Total number of green balls = 6

Possible Combinations are:

Red Green2 4

3 3

4 2

Required number of selections

= (5C2 ×

6C4) + (

5C3 ×

6C3) + (

5C4 ×

6C2) = 425

20. A line required 2 end points for its formation.

Hence required number of lines

= 2 points selected from 6 points=

6C2 = 15

Thus out of total 6 non-collinear point 15 lines can be drawn.

PROBABILITY

Introduction: With the changing pattern of bank PO exams the State Bank of India has also

included few questions from probability to access the quantitative aptitude of a candidate. The

questions of probability in the exam are also based on a given data but in the next exam there

may be some changes in the questioning pattern. Therefore, we have included this chapter for

the candidates to increase their possibility of being selected in the exam. Here, we will study

the basic concepts of probability and also solve few questions.

Probability: It is a mathematical measurement of uncertainty of an event. The value of 

Probability varies from 0 to 1. The 0 value of an event’s probability indicates that the event

cannot happen. On the other side the value 1 for any event indicates that the event is certain.

Page 101: Sbi Po 2013 Preparation Guide

7/30/2019 Sbi Po 2013 Preparation Guide

http://slidepdf.com/reader/full/sbi-po-2013-preparation-guide 101/169

SBI PO 2013 Preparation Guide Page 101 

www.jagranjosh.com Page

101 

SBI PO 2013 Preparation Guide 

Some Important Term

1.  Experiment: It is an operation which produces some well defined outcomes. 

2.  Random Experiment: It is an experiment in which all the possible outcomes are known but

the exact output cannot be predicted in advance. 

For example; Rolling an unbiased dice, Tossing a fair coin etc.

3.  Sample Space: It is a set of all possible outcomes of a random experiment.

For Example; if we toss a coin the possible outcomes are head (H) and tail (T). Therefore the

sample space for tossing a coin = {H, T}.

4.  Event: It is a subset of a sample space. It can be understood as an element or a group of 

elements from the sample space set. 

For Example; if we toss two coins, the possible outcomes are {HH, HT, TH, TT}. Here the

events can be; only H = {HH}, only one H = {HT, TH} or no H = {TT}.

5.  Probability of an Event:

Let S be the sample space and E be and Event in the given sample space.

Then, E S.

∴  .

Some Important Results

1.  P(S) = 1,

2.  0 P(E) 1

3.  P( ) = 0 

4.  For event A and B, P (A ∪ B) = P (A) + P (B) – P (A ∩ B)

5.  If   A is (not A), then P (  A ) = 1 – P (A).

Some Important Formulae

1.  n ! = n (n – 1) (n – 2) ......... 3.2.1

2.  0 ! = 1 = 1 !

3.  Selection of r things out of n things =n

C r = =

4.  nC n =

nC 0 = 1

5.  n C r = n C (n – r) 

Practice Questions

1.  Three unbiased coins are tossed. The probability of getting two heads is

Page 102: Sbi Po 2013 Preparation Guide

7/30/2019 Sbi Po 2013 Preparation Guide

http://slidepdf.com/reader/full/sbi-po-2013-preparation-guide 102/169

SBI PO 2013 Preparation Guide Page 102 

www.jagranjosh.com Page

102 

SBI PO 2013 Preparation Guide 

(a) (b) (c) (d) (e)  None of these

2.  Three unbiased coins are tossed. The probability of getting one head is

(a) (b) (c) (d) (e)  None of these

3.  Three unbiased coins are tossed. The probability of getting at least one head is

(a) (b) (c) (d) (e)  None of these

4.  Three unbiased coins are tossed. The probability of getting at most one head is

(a) (b) (c) 

Page 103: Sbi Po 2013 Preparation Guide

7/30/2019 Sbi Po 2013 Preparation Guide

http://slidepdf.com/reader/full/sbi-po-2013-preparation-guide 103/169

SBI PO 2013 Preparation Guide Page 103 

www.jagranjosh.com Page

103 

SBI PO 2013 Preparation Guide 

(d) (e)  None of these

5.  Two dice are thrown simultaneously. What is the probability of obtaining a total score of 

six?

(a) (b) (c) (d) (e)  None of these

6.  Two dice are thrown simultaneously. What is the probability of getting same digits on both

the dice?

(a) (b) (c) (d) (e)  None of these

7.  Two dice are thrown simultaneously. What is the probability of getting at least total score of 

seven?

(a) (b) (c) (d) (e)  None of these

Page 104: Sbi Po 2013 Preparation Guide

7/30/2019 Sbi Po 2013 Preparation Guide

http://slidepdf.com/reader/full/sbi-po-2013-preparation-guide 104/169

SBI PO 2013 Preparation Guide Page 104 

www.jagranjosh.com Page

104 

SBI PO 2013 Preparation Guide 

8.  Two dice are thrown simultaneously. What is the probability that the obtained value on the

both faces is not same?

(a) (b) (c) (d) (e)  None of these

9.  A bag contains 6 red and 4 blue balls. If 4 balls are drawn at random. The probability that 2

are red and 2 are blue is

(a) (b) (c) (d) (e)  None of these

10. A bag contains 7 red and 8 blue balls. If 5 balls are drawn at random. The probability that atleast 4 blue balls are drawn is

(a) (b) (c) (d) (e)  None of these

Answers:-

1.  (b) 2.  (b) 3.  (d) 4.  (a) 5.  (c)

Page 105: Sbi Po 2013 Preparation Guide

7/30/2019 Sbi Po 2013 Preparation Guide

http://slidepdf.com/reader/full/sbi-po-2013-preparation-guide 105/169

SBI PO 2013 Preparation Guide Page 105 

www.jagranjosh.com Page

105 

SBI PO 2013 Preparation Guide 

6.  (d) 7.  (a) 8.  (c) 9.  (c) 10. (c)

Answer Hints:

1.  All the possible outcomes when three coins are tossed;

{hhh, hht, hth, thh, htt, tht, tth, ttt}

3.  The favourable events are {hhh, hht, hth, thh, htt, tht, tth}

5.  All the possible outcomes when two dice are throw

{(1, 1), (1, 2), (1, 3), (1, 4), (1, 5), (1, 6),

(2, 1), (2, 2), (2, 3), (2, 4), (2, 5), (2, 6),

(3, 1), (3, 2), (3, 3), (3, 4), (3, 5), (3, 6),

(4, 1), (4, 2), (4, 3), (4, 4), (4, 5), (4, 6),

(5, 1), (5, 2), (5, 3), (5, 4), (5, 5), (5, 6),

(5, 1), (5, 2), (5, 3), (5, 4), (5, 5), (5, 6)}

9.  Here the number of sample space means selection of 4 balls out of 10 balls =10

C4 =

= 201.

Now the favourable events of selecting 2 red balls out of 6 red balls and 2 blue balls out of 4

blue balls =4

C 2  6

C 2 = = 90.

Therefore the required probability = = .

RATIO & PROPORTION

Introduction: The concept of Ratio and Proportion is very useful to solve questions on DataInterpretation. In this Unit we will learn the concepts of the topic and solve few questions.

Ratio: A ratio is a comparison of two numbers (quantity in the same unit). It is written as, a: b =

= a ÷b, where a and b are two number (quantity).

Page 106: Sbi Po 2013 Preparation Guide

7/30/2019 Sbi Po 2013 Preparation Guide

http://slidepdf.com/reader/full/sbi-po-2013-preparation-guide 106/169

SBI PO 2013 Preparation Guide Page 106 

www.jagranjosh.com Page

106 

SBI PO 2013 Preparation Guide 

In a ratio a : b, a and b are the terms of the ratio; ‘a’ is called the antecedent and ‘b’ is called

the consequent.

The word ‘antecedent’ literally means ‘that which goes before’ and the word consequent

literally means ‘that which goes after’. 

Compound Ratio: Compounded ratio is a product of two or more ratios.

Example: Find the ratio compounded of the three ratios:

2 : 3, 3 : 4 and 7 : 11

Solution: the required ratio is = .

Inverse Ratio: If a : b is a given ratio, then or b : a is called its inverse ratio or reciprocalratio.

If the antecedent (a) = the consequent (b), the ratio is called the ratio of equality, such as 2 : 2.

If the antecedent (a) > the consequent (b), the ratio is called the ratio of  greater inequality,

such as 3: 2.

If the antecedent (a) < the consequent (b), the ratio is called the ratio of less inequality, such as

2 : 3.

Proportion: A proportion expresses the equality of two ratios. e.g.

Or a : b = c : d or a : b :: c : d.

In a proportion in the form of a : b :: c : d the first and the last terms are called the extremes

and the second and the third terms or the middle terms are called as the mean terms. When

four quantities are in proportion, we can write it in the mathematical form as a : b :: c : d 

ad = bc 

Practice Questions

1.  If 2A = 3B =4C, then A : B : C is ;

(a)  2 : 3 : 4

(b) 4 : 3 : 2

(c)  6 : 4 : 3

Page 107: Sbi Po 2013 Preparation Guide

7/30/2019 Sbi Po 2013 Preparation Guide

http://slidepdf.com/reader/full/sbi-po-2013-preparation-guide 107/169

SBI PO 2013 Preparation Guide Page 107 

www.jagranjosh.com Page

107 

SBI PO 2013 Preparation Guide 

(d) 3 : 4 : 3

(e)  None of these

2.  If  , then A : B : C is ;

(a)  4 : 3 : 5(b) 3 : 4 : 5

(c)  5 : 4 : 3

(d) 5 : 3 : 4

(e)  None of these

3.  The ratio of 43.5

: 25

is same as :

(a)  2 : 1

(b) 4 : 1

(c)  8 : 1

(d) 16 : 1

(e)  None of these

4.  If then the value of x is :

(a)  1.5

(b) 2

(c)  2.5

(d) 3.5

(e)  None of these

5.  If x : y = 5 : 2, the (8x+9y) : (8x + 3y) is :

(a)  23 : 29

(b) 26 : 61

(c)  31 : 21

(d) 29 : 23

(e)  None of these

6.  If 15% of x = 20% of y, then x : y is :

(a)  4 : 3

(b) 3 : 4

(c)  7 : 3

(d) 7 : 4

(e)  None of these

7.  If x : y =3 : 2, then (x2  – y

2) : (x

2+ y

2) is :

Page 108: Sbi Po 2013 Preparation Guide

7/30/2019 Sbi Po 2013 Preparation Guide

http://slidepdf.com/reader/full/sbi-po-2013-preparation-guide 108/169

SBI PO 2013 Preparation Guide Page 108 

www.jagranjosh.com Page

108 

SBI PO 2013 Preparation Guide 

(a)  5 : 13

(b) 13 : 5

(c)  9 : 4

(d) 4 : 9

(e) None of these

8.  If  , then (x + 4) : (y + 5) is equal to :

(a)  4 : 9

(b) 9 : 4

(c)  5 : 4

(d) 4 : 5

(e)  None of these

9.  If , then is :

(a)  12 : 5

(b) 13 : 5

(c)  13 : 6

(d) 5 : 12

(e)  None of these

10.  If Rs. 483 be divided into three parts, proportional to , then the first part is :

(a)  155

(b) 126(c)  137

(d) 115

(e)  None of these

Answers

1.  (c )

2.  (b)

3.  (b)

4.  (c)

5.  (d)

6.  (a)

7.  (a)

8.  (d)

9.  (a)

10. (b)

AVERAGE

Introduction: In this unit, we will learn the concept of Average and solve the questions based

on it. The concept of Average is very helpful in solving various kinds of questions on Data

Interpretation and Analysis.

Page 109: Sbi Po 2013 Preparation Guide

7/30/2019 Sbi Po 2013 Preparation Guide

http://slidepdf.com/reader/full/sbi-po-2013-preparation-guide 109/169

SBI PO 2013 Preparation Guide Page 109 

www.jagranjosh.com Page

109 

SBI PO 2013 Preparation Guide 

In this unit, we will learn the average, which is in more accurate meaning called as an arithmetic

mean.

Average is defined as the sum of n different numerical values divided by n.

Average =

Average Speed =

If a person covers half of his journey at a speed of x km/h and the next half at the speed of y

km/h, then the average speed during the whole journey is .

Weighted Average of x1, x2 where weight is w1, w2 = .

Average of different groups

If the average of a group of n items is a and the average of another group of m items is m,

then the combined average = .

Practice Example

1.  The average of a non-zero number and its square is 5 times the number. The number is:(a)  9

(b) 17

(c)  29

(d) 295

(e)  None of these

2.  The average age of the boys in a class is 16 and that of the girls is 15 years. The average age

for the whole class is :

(a)  15 years

(b) 15.5 years

(c)  16 years

(d) Cannot be determined

(e)  None of these

Page 110: Sbi Po 2013 Preparation Guide

7/30/2019 Sbi Po 2013 Preparation Guide

http://slidepdf.com/reader/full/sbi-po-2013-preparation-guide 110/169

SBI PO 2013 Preparation Guide Page 110 

www.jagranjosh.com Page

110 

SBI PO 2013 Preparation Guide 

3.  The average run of a cricket player of 10 innings was 25. How many runs must he make in his

next inning so that his average becomes 30 :

(a)  25

(b) 30

(c) 

80(d) 100

(e)  None of these

4.  The mean of 36 observations was 25. It was found later that an observation 24 was wrongly

taken as 42. The corrected new mean is :

(a)  24

(b) 24.5

(c)  25

(d) 25.5

(e)  None of these

5.  The average weight of A, B and C is 50 kg. If the average weight of A and B be 45 kg and that

of B and C be 54 kg, then the weight of B is :

(a)  45

(b) 48

(c)  50

(d) 55

(e)  None of these

Answers

1.  (a) 2.  (d) 3.  (c) 4.  (b) 5.  (b)

Answer Hints

1.  (a) Let the number be x. Then

∴ x = 9.

2.  (d) We can’t determine the average because there is no information about no. of girls and

no. of boys.

3.  (c) His total runs after 10 innings = 25 × 10 = 250 and his total score after 11th

inning should

be 11 × 30 = 330 to make his average 30. Therefore he requires another 330  – 250 = 80

runs.

Page 111: Sbi Po 2013 Preparation Guide

7/30/2019 Sbi Po 2013 Preparation Guide

http://slidepdf.com/reader/full/sbi-po-2013-preparation-guide 111/169

SBI PO 2013 Preparation Guide Page 111 

www.jagranjosh.com Page

111 

SBI PO 2013 Preparation Guide 

4.  (b) The wrongly taken figure is 42 for 24. It means, in the corrected average total value will

decrease by (42 – 24) = 18. Therefore the corrected average will reduce by = 0.5.

5.  (b) the total weight of A, B and C is

A + B + C = 50 × 3 = 150 ……(1) 

Similarly, A + B = 45 × 2 = 90 …….(2) 

And B + C = 54 × 2 = 108 ……….(3) 

By (2) + (3) – (1)

A + B + B + C – A – B – C = 108 + 90 – 150

∴B = 48.

PERCENTAGE

Introduction: In this unit, we will study about the concepts of Percentage and the questions

based on the topic. The concepts of percentage are very helpful in solving various kinds of 

questions on Data Interpretation and Analysis.

Percentage: The term “per cent” means “for every hundred”. A fraction whose denominators

is 100 is called a percentage and the numerator of the fraction is called the rate per cent. It is

denoted by the symbol %.

Here x % = . For example 10% = .

Similarly, fraction can be changed in the form of percentage when we multiply them by 100.

For Example . 

To decrease a number by a given %:

Multiple the numbers by the factor

To find the % increase of a number:

Page 112: Sbi Po 2013 Preparation Guide

7/30/2019 Sbi Po 2013 Preparation Guide

http://slidepdf.com/reader/full/sbi-po-2013-preparation-guide 112/169

SBI PO 2013 Preparation Guide Page 112 

www.jagranjosh.com Page

112 

SBI PO 2013 Preparation Guide 

% increase =

To find the % decrease of a number:

% decrease =

Practice Example

1.  In an election between two candidates, 75% of the voters casted their votes, out of which 2%

of the votes were declared invalid. A candidate got 9261 votes which were 75% of the total

valid votes. Find the total number of votes enrolled in that election.

(a)  15000

(b) 16000

(c)  16800(d) 17000

(e)  None of these

2.  If 50% of (x – y) = 30% of (x + y), then what percent of x is y?

(a)  25%

(b) 50%

(c)  75%

(d) 100%

(e)  None of these

3.  The salary of a person was reduced by 10%. By what percent should his reduced salary be

raised so as to bring it at par with his original salary?

(a)  10 %

(b) (c)  12 %

(d) 13 %

(e)  None of these

4.  A single discount equivalent to a discount series 10%, 20% and 10% is

(a)  40 %

(b) 35.2 %

(c)  30 %

(d) 25.8 %

Page 113: Sbi Po 2013 Preparation Guide

7/30/2019 Sbi Po 2013 Preparation Guide

http://slidepdf.com/reader/full/sbi-po-2013-preparation-guide 113/169

SBI PO 2013 Preparation Guide Page 113 

www.jagranjosh.com Page

113 

SBI PO 2013 Preparation Guide 

(e)  None of these

5.  A single discount equivalent to a discount series 50% + 30% + 20% is

(a)  100 %

(b) 80 %

(c)  75 %

(d) 72 %

(e)  None of these

Answers

1.  (c) 2.  (a) 3.  (b) 4.  (b) 5.  (d)

Answer Hints

1.  (c) Let the total enrolled votes are x.

Then, Number of votes cast = 75 % of x. Valid votes = 98 % of (75 % of x).

∴ 75 % 0f [98 % of (75 % of x)] = 9261.

2.  (a) here 50% of (x – y) = 30% of (x + y)   10(x – y) = 6(x + y)

10x – 6x = 6y + 10y 4x = 16y x = y 25 % of x = y. because .

4.  (b) Let we have 100 initially first 10 % discount = 100 × = 10.

The second discount 20 % = 90 × = 18.

The third discount 10 % = 72 × = 7.2.

Hence total discount = 10 + 18 + 7.2 = 35.2.

Page 114: Sbi Po 2013 Preparation Guide

7/30/2019 Sbi Po 2013 Preparation Guide

http://slidepdf.com/reader/full/sbi-po-2013-preparation-guide 114/169

SBI PO 2013 Preparation Guide Page 114 

www.jagranjosh.com Page

114 

SBI PO 2013 Preparation Guide 

Page 115: Sbi Po 2013 Preparation Guide

7/30/2019 Sbi Po 2013 Preparation Guide

http://slidepdf.com/reader/full/sbi-po-2013-preparation-guide 115/169

SBI PO 2013 Preparation Guide Page 115 

www.jagranjosh.com Page

115 

SBI PO 2013 Preparation Guide 

ENGLISH LANGUAGE

ANTONYMS

Introduction: In this unit we will study about antonyms. The questions about antonyms are

generally asked in SBI PO exams. The SBI PO exam will contain about three to five questions

based on synonyms.

Antonyms: The word comes from ancient Greek words ‘anti’ and ‘onoma’ where ‘anti’ means

‘opposite’ and ‘onoma’ means ‘name’. The literal meaning of antonyms is the opposite name

which means the word which has opposite meaning. For example; Day – Night, Long – Short, Up

 – Down, Small – Large, etc

Example Questions

Q. Find the opposite word Concur.

(a) Disagree

(b) Disappear

(c)  Disarrange

(d) Discourage

(e) None of these

Answer: The synonyms for Concur are agree, cooperate, combine etc. Therefore the antonym

for ‘agree’ is ‘disagree’. Hence right answer is (a). 

Note: for the preparation of the Antonyms questions, the students are required to have a good

practice of the topic and English reading habit. These two things will help you to be better

prepared for the exam. When you are solving the questions of antonyms try to figure out your

synonymous words which come first in your mind for the question word then look at options

for all possible antonyms of your question word and the synonymous words. By this way you

will find the most opposite word for the question word. 

Practice Examples

Direction: In the given questions, choose the word opposite in meaning to the given word and

mark it in the Answer Sheet. 

1.  Condensation

(a)  Abridgment

(b) Broadening

Page 116: Sbi Po 2013 Preparation Guide

7/30/2019 Sbi Po 2013 Preparation Guide

http://slidepdf.com/reader/full/sbi-po-2013-preparation-guide 116/169

SBI PO 2013 Preparation Guide Page 116 

www.jagranjosh.com Page

116 

SBI PO 2013 Preparation Guide 

(c)  Compression

(d) Concentration

(e)  None of these

2.  Condemn

(a)  Praise

(b) Censure

(c)  Disapproval

(d) Doom

(e)  None of these

3.  Encroach

(a)  Creep

(b)  Infringe

(c) 

Intrude(d) Recede

(e)  None of these

4.  Endeavour

(a)  Aim

(b) Try

(c)  Abandon

(d) Effort

(e)  None of these

5.  Ferocious

(a)  Barbarous

(b) Bloody

(c)  Brutal

(d) Humane

(e)  None of these

6.  Fastidious

(a)  Censorious

(b) Critical

(c)  Dainty

(d) Careless

(e)  None of these

7.  Greasy

(a)  Lean

Page 117: Sbi Po 2013 Preparation Guide

7/30/2019 Sbi Po 2013 Preparation Guide

http://slidepdf.com/reader/full/sbi-po-2013-preparation-guide 117/169

SBI PO 2013 Preparation Guide Page 117 

www.jagranjosh.com Page

117 

SBI PO 2013 Preparation Guide 

(b) Oily

(c)  Fatty

(d) Sebaceous

(e)  None of these

8.  Hallow

(a)  Consecrate

(b) Desecrate

(c)  Dedicate

(d) Devote

(e)  None of these

9.  Harmonize

(a)  Accord

(b) Differ

(c)  Agree

(d) Cohere

(e)  None of these

10. Hazard

(a)  Adventure

(b) Risk

(c)  Secure

(d) Peril

(e)  None of these

11. Herculean

(a)  Athletic

(b) Colossal

(c)  Feeble

(d) Strong

(e)  None of these

12. Hypothesis

(a)  Proof 

(b) Conjecture

(c)  Supposition

(d) Theory

(e)  None of these

13. Impulsive

Page 118: Sbi Po 2013 Preparation Guide

7/30/2019 Sbi Po 2013 Preparation Guide

http://slidepdf.com/reader/full/sbi-po-2013-preparation-guide 118/169

SBI PO 2013 Preparation Guide Page 118 

www.jagranjosh.com Page

118 

SBI PO 2013 Preparation Guide 

(a)  Deliberate

(b) Careless

(c)  Hasty

(d)  Impetuous

(e) None of these

14. Incomplete

(a)  Absolute

(b) Defective

(c)  Faulty

(d) Partial

(e)  None of these

15. Incurable

(a) Hopeless

(b)  Irrecoverable

(c)  Remediable

(d) Cureless

(e)  None of these

16. Judicious

(a)  Cautious

(b) Considerate

(c)  Cool

(d) Rash

(e)  None of these

17. Knot

(a)  Complication

(b) Simplicity

(c)  Difficulty

(d) Entanglement

(e)  None of these

18. Lament

(a)  Complain

(b) Moan

(c)  Cry

(d) Rejoice

(e)  None of these

Page 119: Sbi Po 2013 Preparation Guide

7/30/2019 Sbi Po 2013 Preparation Guide

http://slidepdf.com/reader/full/sbi-po-2013-preparation-guide 119/169

SBI PO 2013 Preparation Guide Page 119 

www.jagranjosh.com Page

119 

SBI PO 2013 Preparation Guide 

19. Legend

(a)  Caption

(b) Fact

(c)  Fiction

(d) Myth

(e)  None of these

20. Malpractice

(a)  Dereliction

(b) Evil

(c)  Misbehavior

(d) Right

(e)  None of these

Answers1.  (b)

2.  (a)

3.  (d)

4.  (c)

5.  (d)

6.  (d)

7.  (a)

8.  (b)

9.  (b)

10. (c)

11. (c)

12. (a)

13. (a)

14. (a)

15. (c)

16. (d)

17. (b)

18. (d)

19. (b)

20. (d)

SYNONYMS

Introduction: In this unit we will study about synonyms. The questions about synonyms are

generally asked in SBI PO exams. The SBI PO exam will contain about three to five questions

based on synonyms.

Synonyms: The word comes from ancient Greek words ‘syn’ and ‘onoma’ where ‘syn’ means

‘with’ and ‘onoma’ means ‘name’. Synonyms are the word which has similar meanings. 

Two words are said to be synonymous when they have similar meanings. The words caste and

class is synonymous because both the words have similar meaning.

Example Questions

Q. Find the word which synonymous to Deny.(a)  Regain

(b) Refuse

(c)  Repair

(d) Reduce

(e)  None of these

Page 120: Sbi Po 2013 Preparation Guide

7/30/2019 Sbi Po 2013 Preparation Guide

http://slidepdf.com/reader/full/sbi-po-2013-preparation-guide 120/169

SBI PO 2013 Preparation Guide Page 120 

www.jagranjosh.com Page

120 

SBI PO 2013 Preparation Guide 

Answer: The synonyms for Deny are Contradict, Refuse, Reject etc. here we have Refuse an

option which means the right answer is (b).

Note: for the preparation of the synonyms questions the students are required to have a good

practice of the topic and English reading habit. These two things will help you to be better

prepared for the exam. When you are solving the questions of synonyms try to figure out your

synonymous words for the question word then match with the options.

Practice Examples

Directions: In the given questions out of the four alternatives, choose the one which best

expresses the meaning of the given word. 

1.  Deposit

(a)  Degrade

(b) Dethrone

(c)  Place

(d) Removal

(e)  None of these

2.  Depart

(a)  Decamp

(b) Divest

(c)  Display

(d) Degrade

(e)  None of these

3.  Portion

(a)  Gallery

(b) Fragment

(c)  Site

(d) Case

(e)  None of these

4.  Exact(a)  True

(b) Pattern

(c)  Elusive

(d) Close

(e)  None of these

Page 121: Sbi Po 2013 Preparation Guide

7/30/2019 Sbi Po 2013 Preparation Guide

http://slidepdf.com/reader/full/sbi-po-2013-preparation-guide 121/169

SBI PO 2013 Preparation Guide Page 121 

www.jagranjosh.com Page

121 

SBI PO 2013 Preparation Guide 

5.  Indignant

(a)  Bend

(b) Faulty

(c)  Gratified

(d) Angry

(e)  None of these

6.  Incur

(a)  Elude

(b) Enhance

(c)  Mount

(d) Acquire

(e)  None of these

7. 

Caustic(a)  Acrid

(b) Mild

(c)  Cavil

(d) Clue

(e)  None of these

8.  Cavity

(a)  Cave

(b) Hollow

(c)  Class

(d) Origin

(e)  None of these

9.  Charge

(a)  Crack

(b) Right

(c)  Accuse

(d) Claim

(e)  None of these

10. Catastrophic

(a)  Callous

(b) Dull

(c)  Calamitous

(d) Continuous

Page 122: Sbi Po 2013 Preparation Guide

7/30/2019 Sbi Po 2013 Preparation Guide

http://slidepdf.com/reader/full/sbi-po-2013-preparation-guide 122/169

SBI PO 2013 Preparation Guide Page 122 

www.jagranjosh.com Page

122 

SBI PO 2013 Preparation Guide 

(e)  None of these

11. Discretion

(a)  Rashness

(b) Careless

(c)  Carefulness

(d) Disgrace

(e)  None of these

12. Imperative

(a)  Optional

(b) Authoritative

(c)  Elective

(d) Voluntary

(e) None of these

13. Opportunity

(a)  Chance

(b) Omission

(c)  Artisan

(d) Vital

(e)  None of these

14. Numerate

(a)  Compute(b) Denude

(c)  Nudge

(d) Account

(e)  None of these

15. Noise

(a)  Quiet

(b) Blare

(c)  Placid

(d) Peaceful

(e)  None of these

16. Nimble

(a)  Clumsy

(b) Lump

(c)  Silent

Page 123: Sbi Po 2013 Preparation Guide

7/30/2019 Sbi Po 2013 Preparation Guide

http://slidepdf.com/reader/full/sbi-po-2013-preparation-guide 123/169

SBI PO 2013 Preparation Guide Page 123 

www.jagranjosh.com Page

123 

SBI PO 2013 Preparation Guide 

(d) Agile

(e)  None of these

17. Nerve

(a)  Courage

(b) Weakness

(c)  Fragile

(d) Lame

(e)  None of these

18. Notch

(a)  Designate

(b) Beck

(c)  Cut

(d) Rule

(e)  None of these

19. Proclaim

(a)  Conceal

(b) Cover

(c)  Disguise

(d) Announce

(e)  None of these

20. Profane(a)  Hallow

(b) Debase

(c)  Dedicate

(d) Consecrate

(e)  None of these

Answers

1.  (c)

2.  (a)

3.  (b)

4.  (a)

5.  (d)

6.  (d)

7.  (a)

8.  (b)

9.  (c)

10. (c)

11. (c)

12. (b)

13. (a)

14. (a)

15. (b)

16. (d)

17. (a)

18. (c)

19. (d)

20. (c)

FILL IN THE BLANKS

Page 124: Sbi Po 2013 Preparation Guide

7/30/2019 Sbi Po 2013 Preparation Guide

http://slidepdf.com/reader/full/sbi-po-2013-preparation-guide 124/169

SBI PO 2013 Preparation Guide Page 124 

www.jagranjosh.com Page

124 

SBI PO 2013 Preparation Guide 

Introduction: In the SBI PO exam, there can be questions on Fill in the Blanks to assess the

candidate’s knowledge of English grammar and composition. The exam can consist of five

questions on the Fill in the blanks.

Here, we will practice few questions on the Fill in the Blanks type.

Practice Questions

Direction: Fill in the Blanks with the most suitable alternative.

1.  The terrorists………………………..a bomb in the local bus. 

(a)  Planted

(b) Hid

(c)  Any

(d) Explode

(e)  Stole

2.  Every citizen is expected to……………………………the interests of his country. 

(a)  Protect

(b) Guarantee

(c)  Increase

(d) Grow

(e)  Obey

3.  She says her dress cost her Rs 500. It doesn’t look……………………it. 

(a)  Less

(b) Worth

(c)  In

(d) For

(e)  About

4.  Pulse Polio is a drive……………………….to eradicate polio from all over the country? 

(a)  Tinted

(b) Began

(c)  Launched

(d) Parked(e)  Aim

5.  A…………………………of cars was made available to the minister and his party?

(a)  Pack

(b) Group

Page 125: Sbi Po 2013 Preparation Guide

7/30/2019 Sbi Po 2013 Preparation Guide

http://slidepdf.com/reader/full/sbi-po-2013-preparation-guide 125/169

SBI PO 2013 Preparation Guide Page 125 

www.jagranjosh.com Page

125 

SBI PO 2013 Preparation Guide 

(c)  Number

(d) Fleet

(e)  Horde

6.  Shikha could not get………………….the examination. 

(a)  Over

(b) Through

(c)  Along

(d)  Into

(e)  Up

7.  The magic show was the prime attraction for the children and adults………………………….. 

(a)  Alike

(b) Together

(c)  All(d) Either

(e)  Altogether

8.  It is ……………………………accepted that Jayadeva was born in the village of Kenduli.

(a)  Wholesome

(b) Widely

(c)  Firmly

(d) Openly

(e)  Fairly

9.  Abdul is senior to Habib………………………six years in this institute. 

(a)  Above

(b) Over

(c)  By

(d) For

(e)  From

10. Ramesh was to appear…………………..the interview for the post of Pub lic Relations Officers.

(a)  For

(b)  Into

(c)  Before

(d) With

(e)  At

11. I was speaking to him on phone when the line……………………cut off.

Page 126: Sbi Po 2013 Preparation Guide

7/30/2019 Sbi Po 2013 Preparation Guide

http://slidepdf.com/reader/full/sbi-po-2013-preparation-guide 126/169

SBI PO 2013 Preparation Guide Page 126 

www.jagranjosh.com Page

126 

SBI PO 2013 Preparation Guide 

(a)  Got

(b) Gets

(c)  Gotten

(d) Getting

(e) Get

12. I do not………………………of his conduct although he is my own brother. 

(a)  Tolerate

(b) Agree

(c)  Excuse

(d) Approve

(e)  Writer

13. Rakesh promised me that he would be……………………a letter after reaching home? 

(a)  Wrote(b) Written

(c)  Write

(d) Writing

(e)  Writer

14. The judge found him guilty and he was sentenced…………………death. 

(a)  To

(b) Upon

(c)  For

(d) Till

(e)  As

15. Satish is the strongest…………………all the boys in our school. 

(a)  On

(b) Between

(c)  Of 

(d) To

(e)  Before

16. The teacher asked Priya to go home when when she started…………………..of stomach pain. 

(a)  Complaining

(b) Knowing

(c)  Blaming

(d) Feeling

Page 127: Sbi Po 2013 Preparation Guide

7/30/2019 Sbi Po 2013 Preparation Guide

http://slidepdf.com/reader/full/sbi-po-2013-preparation-guide 127/169

SBI PO 2013 Preparation Guide Page 127 

www.jagranjosh.com Page

127 

SBI PO 2013 Preparation Guide 

(e)  Developing

17. Anuradha’s voice is …………………………..sweet as Lata’s. 

(a)  More

(b) Far

(c)  As

(d) Very

(e)  So

18. The child has been……………………sick very often for the last three months.

(a)  Felt

(b) Falling

(c)  Falls

(d) Fallen

(e)  Fall

19. The surgeon advised the patient to…………………………an operation. 

(a)  Undergo

(b) Underpass

(c)  Underset

(d) Undertake

(e)  Underlie

20. It was in consultation…………………………the workers union that the management arrived at

this decision.(a)  Over

(b) To

(c)  About

(d) By

(e)  With

21. She…………………..in slandering people. 

(a)  Enjoys

(b) Participates

(c)  Indulges

(d) Laughed

(e)  Angered

22. Many are………………………..under the burden of poverty. 

(a)  Buried

Page 128: Sbi Po 2013 Preparation Guide

7/30/2019 Sbi Po 2013 Preparation Guide

http://slidepdf.com/reader/full/sbi-po-2013-preparation-guide 128/169

SBI PO 2013 Preparation Guide Page 128 

www.jagranjosh.com Page

128 

SBI PO 2013 Preparation Guide 

(b) Pushed

(c)  Thrown

(d) Reeling

(e)  Pressed

23. We want a law……………………..dowry. 

(a)  For

(b) About

(c)  On

(d) Against

(e)  To

24. The curfew was……………..yesterday.

(a)  Implemented

(b) Taken(c)  Told

(d)  Imposed

(e)  Lift

25. He brought oranges……………………..the dozen. 

(a)  In

(b) By

(c)  For

(d) At

(e)  To

26. …………………..he woke up, he had missed the station. 

(a)  Where

(b) After

(c)  While

(d) As

(e)  When

27. I wanted to know what had………………..beneath it. 

(a)  Bed

(b) Lain

(c)  Laid

(d) Lying

(e)  Laying

Page 129: Sbi Po 2013 Preparation Guide

7/30/2019 Sbi Po 2013 Preparation Guide

http://slidepdf.com/reader/full/sbi-po-2013-preparation-guide 129/169

SBI PO 2013 Preparation Guide Page 129 

www.jagranjosh.com Page

129 

SBI PO 2013 Preparation Guide 

28. The principal…………………………the boys to go home. 

(a)  Let

(b) Deems

(c)  Allowed

(d) Want

(e)  send

29. The couple did not…………………………the rival company. 

(a)  Hike

(b) Hack

(c)  Excel

(d) Represent

(e)  Sold

30. This has……………………..many myths. (a)  Broken

(b) Shatter

(c)  Fell

(d) Finished

(e)  Stopped

Answer:-

1.  (a)

2.  (a)3.  (b)

4.  (c)

5.  (d)

6.  (b)

7.  (a)

8.  (b)9.  (c)

10. (a)

11. (a)

12. (d)

13. (a)

14. (d)15. (c)

16. (a)

17. (c)

18. (d)

19. (a)

20. (e)21. (c)

22. (d)

23. (d)

24. (d)

25. (b)

26. (e)27. (b)

28. (c)

29. (d)

30. (b)

ERROR DETECTION

Introduction: In the SBI PO exam, there can be questions on Error Detection to assess the

candidate’s knowledge of English grammar and composition. The exam can consist of five

questions on the Error Detection.

Here, we will practice few questions on the Error Detection.

Practice Questions

Page 130: Sbi Po 2013 Preparation Guide

7/30/2019 Sbi Po 2013 Preparation Guide

http://slidepdf.com/reader/full/sbi-po-2013-preparation-guide 130/169

SBI PO 2013 Preparation Guide Page 130 

www.jagranjosh.com Page

130 

SBI PO 2013 Preparation Guide 

Direction: In each sentence below, four words have been printed in Bold Type. One of these

may be wrongly spelt or inappropriate in the context (if any) and is the answer. If all four words

are correctly spelt or appropriate in the context, then the answer is ‘(e)’ i.e. no error.

1.  They questioned him and me unendingly about our trip to Singapore and the Purchase we

(a) (b) (c) (d)Had made. No Error

(e)

2.  The Passerbyes seemed not to be affected by the gory sight of the crushed dog. No Error

(a) (b) (c) (d) (e)

3.  Mrs Bose, napping on the sofa, woke up started and enquired what happened. No Error

(a) (b) (c) (d) (e)

4. The lady approached me timidly and sat down besides me fidgeting nervously. No Error 

(a) (b) (c) (d) (e)

5.  The M.P. was noticeably upset by the manner in which the Minister conducted himself.

(a) (b) (c) (d)

No error.

(e)

6.  Him not agreeing to the proposal outlined by me is baffling. No Error

(a) (b) (c) (d) (e)

7.  Whenever I felt sorrowful I use to turn towards God. No Error

(a) (b) (c) (d) (e)

8.  The carpenter uses the saw to saw wood and planks. No Error

(a) (b) (c) (d) (e)

9.  I have been ordered to lay in bed and do nothing. No Error 

(a) (b) (c) (d) (e)

10. I did this just for your sake and her but have to account for it. No Error

(a) (b) (c) (d) (e)

11. Is there any further reason you give for your strange behavior towards my sister. No error (a) (b) (c) (d) (e)

12. The posts were advertised by the Planning Commison in all the local dailies. No Error

(a) (b) (c) (d) (e)

13. I am utterly exhausted and much annoyed with the turn of events against me. No Error

Page 131: Sbi Po 2013 Preparation Guide

7/30/2019 Sbi Po 2013 Preparation Guide

http://slidepdf.com/reader/full/sbi-po-2013-preparation-guide 131/169

SBI PO 2013 Preparation Guide Page 131 

www.jagranjosh.com Page

131 

SBI PO 2013 Preparation Guide 

(a) (b) (c) (d) (e)

14. His niece is too much tall for her age, isn’t she? No Error

(a) (b) (c) (d) (e)

15. The old woman hugged her grandsons warmly with tear in her eyes. No Error

(a) (b) (c) (d) (e)

16. This train takes perfectly ten hours to reach Calcutta. No Error

(a) (b) (c) (d) (e)

17. His parents were informed of his serious illness and of the surgory performed on him. No

Error 

(a) (b) (c) (d) (e)

18. The appointment will be made initally for period of three years. No Error

(a) (b) (c) (d) (e)

19. When Satish heard those humiliation words he became furious. No Error 

(a) (b) (c) (d) (e)

20. Rajendra and Ashu came very late lost night. No Error

(a) (b) (c) (d) (e)

21. Fluency in English language and last experience in the field would be desirable. No Error

(a) (b) (c) (d) (e)

22. The fact, as brought out by all the newspaper, are that global tele-communication market is 

(a) (b) (c)

Not very global. No Error

(d) (e)

23. Water does not few upward to the mountains but downward to the sea. No Error

(a) (b) (c) (d) (e)

24. It is written in the book of God that he who has plenty shall be asked to help others. No

Errors

(a) (b) (c) (d) (e)

25. He is competent and brilliant at his work, but utterly unpretentious. No Error

(a) (b) (c) (d) (e)

26. He needs not work So hard lest he should fall ill. No Error

(a) (b) (c) (d) (e)

Page 132: Sbi Po 2013 Preparation Guide

7/30/2019 Sbi Po 2013 Preparation Guide

http://slidepdf.com/reader/full/sbi-po-2013-preparation-guide 132/169

SBI PO 2013 Preparation Guide Page 132 

www.jagranjosh.com Page

132 

SBI PO 2013 Preparation Guide 

27. The problem seems to be complicated but the real fact is that it is’nt. Is it? No Error 

(a) (b) (c) (d) (e)

28. The amount that I have paid for it is lesser than what you have paid. No Error

(a) (b) (c) (d) (e)

29. These boxers dominated the ring with heroic display of their talent for boxing. No Error

(a) (b) (c) (d) (e)

30. I who had missed the final round yesterday am very upset. No Error 

(a) (b) (c) (d) (e)

Answers:-

1.  (b)

2.  (a)

3.  (b)

4.  (c)

5.  (b)

6.  (a)

7.  (c)

8.  (a)

9.  (b)

10. (c)

11. (e)

12. (c)

13. (a)

14. (b)

15. (d)

16. (b)

17. (c)

18. (c)

19. (b)

20. (c)

21. (c)

22. (a)

23. (b)

24. (b)

25. (b)

26. (a)

27. (b)

28. (c)

29. (e)

30. (c)

IDIOMS & PHRASES

Introduction: In the SBI PO exam, there can be questions of Idioms & Phrases to assess thecandidate’s knowledge of English grammar and composition. The exam can consist of five

questions on the Idioms & Phrases.

Here, we will practice few questions on the Idioms & Phrases.

Practice Questions

Direction: Find the word/phrases given below each of the sentences should replace the

word/phrase printed in bold letter to make the sentence grammatically correct. If the sentence

is correct as it is and no correction is required, then mark (e) as your answer. 

1.  The government announced much tax concessions for the income tax payers.

(a)  A little

(b) Some

(c)  Any

(d) Heaps of 

(e)  No correction required

Page 133: Sbi Po 2013 Preparation Guide

7/30/2019 Sbi Po 2013 Preparation Guide

http://slidepdf.com/reader/full/sbi-po-2013-preparation-guide 133/169

SBI PO 2013 Preparation Guide Page 133 

www.jagranjosh.com Page

133 

SBI PO 2013 Preparation Guide 

2.  Had you gone to Calcutta, do not forget to visit the Metro Rail.

(a)  Had you go

(b)  If you had gone

(c)  Going

(d) If you go

(e)  No Correction Required

3.  Swimming cannot be learnt unless one will get down to water.

(a)  So long as one gets down

(b) Until one gets down

(c)  Unless one gets down

(d) Until one will get down

(e)  No Correction Required

4.  Shabnam asked Laila how did she join Excellent Bank Coaching.(a)  How she has joined

(b) How she joined

(c)  How had she joined

(d) That how she joined

(e)  No correction required

5.  People are voluntarily helping the riot victims.

(a)  Spontaneously

(b) Purposefully

(c)  Naturally

(d)  Intentionally

(e)  No Correction Required

6.  You are requested to kindly help me.

(a)  To help

(b) Kindly to help

(c)  To please help

(d) Not to help

(e)  No Correction required

7.  Ramesh even could not solve the sum.

(a)  Ramesh and I both

(b) Only Ramesh

(c)  Ramesh but

Page 134: Sbi Po 2013 Preparation Guide

7/30/2019 Sbi Po 2013 Preparation Guide

http://slidepdf.com/reader/full/sbi-po-2013-preparation-guide 134/169

SBI PO 2013 Preparation Guide Page 134 

www.jagranjosh.com Page

134 

SBI PO 2013 Preparation Guide 

(d) Ramesh only

(e)  No Correction

8.  A body of volunteers have been organized for the relief work.

(a)  Are also

(b) Could have

(c)  Are being

(d)  Is being

(e)  No Correction required

9.  She is in danger to lose her balance.

(a)  To loose

(b) Of lose

(c)  Of losing

(d) To lose off (e)  No Correction Required

10. He is good friend of his and I. 

(a)  His and me

(b) Him and yours

(c)  Hers and mine

(d) Me and his

(e)  No Correction Required

11. This is the only book which I want.(a)  That I want

(b)  I wanted

(c)  I had wanted

(d) Which I have wanted

(e)  No Correction Required

12. The room is rather too dark.

(a)  Much too

(b) Too

(c)  Rather

(d) Too much

(e)  No correction required

13. Either of these are good.

(a)  Either

Page 135: Sbi Po 2013 Preparation Guide

7/30/2019 Sbi Po 2013 Preparation Guide

http://slidepdf.com/reader/full/sbi-po-2013-preparation-guide 135/169

SBI PO 2013 Preparation Guide Page 135 

www.jagranjosh.com Page

135 

SBI PO 2013 Preparation Guide 

(b) Both

(c)  Any of these

(d) None

(e)  No correction required

14. He walks as if to be drunk.

(a)  He were

(b) Though

(c)  Like

(d) As

(e)  No correction required

15. Nothing than hard work can help you.

(a)  Nothing else than

(b) No other things than(c)  But

(d) Nothing but

(e)  No correction required

16. He is a sales-representative in a company.

(a)  Sales-representative

(b) Selling-representative

(c)  Sell-representative

(d) Sail-representative

(e)  No correction required

17. It is hard for me re-adjust in life abroad.

(a)  With life

(b) To life

(c)  By life

(d) Life

(e)  No correction required

18. The drought was followed by months of famine.

(a)  By many months

(b) By a months of 

(c)  By

(d) By severe months

(e)  No correction required

Page 136: Sbi Po 2013 Preparation Guide

7/30/2019 Sbi Po 2013 Preparation Guide

http://slidepdf.com/reader/full/sbi-po-2013-preparation-guide 136/169

SBI PO 2013 Preparation Guide Page 136 

www.jagranjosh.com Page

136 

SBI PO 2013 Preparation Guide 

19. There are no raises in the salary.

(a)  Is any rise

(b) Are no rise

(c)  Is no rise

(d) Are no bonuses

(e)  No correction required

20. The bell was wrung late.

(a)  Is wrung

(b)  Is rung

(c)  Has rung

(d) Was rung

(e)  No correction required

21. The aggregate income of the trader was enormous.(a)  Sum total

(b) Maximum earning

(c)  Minimum penalty

(d) Daily sale

(e)  No correction required

22. The passenger was on his own.

(a)  Clonely

(b) By one self 

(c)  With his family

(d) Having his own ticket

(e)  By himself 

23. I have read only the first three chapters of this novel.

(a)  The three

(b) The whole

(c)  Most of the

(d) Three first

(e)  No correction required

24. He is cunning and rogue fellow.

(a)  A rogue man

(b) An honest man

(c)  Crafty fellow

Page 137: Sbi Po 2013 Preparation Guide

7/30/2019 Sbi Po 2013 Preparation Guide

http://slidepdf.com/reader/full/sbi-po-2013-preparation-guide 137/169

SBI PO 2013 Preparation Guide Page 137 

www.jagranjosh.com Page

137 

SBI PO 2013 Preparation Guide 

(d) Fast trader

(e)  No correction required

25. It is better to do something than sit idle.

(a)  Sitting idle

(b) Be idle

(c)  To sit idle

(d) Not to do so

(e)  No correction required

26. He was chosen as captain. 

(a)  Elected as

(b) Nominated as

(c)  Not chosen as

(d) Elected(e)  No correction required

27. Both of us did not attend school.

(a)  Both the two boys

(b) We all

(c)  Any of us

(d) All of us

(e)  The two of us

28. He is not such a fool.(a)  A

(b) That a

(c)  Much of a

(d) Quite a

(e)  No correction required

29. You will have to return my belonging whenever I ask for them.

(a)  Should have to

(b) Will have

(c)  Would have

(d) Will be

(e)  No correction required

30. I am prepared to lie down my life for the sake of the nation.

(a)  Preparing to lie

Page 138: Sbi Po 2013 Preparation Guide

7/30/2019 Sbi Po 2013 Preparation Guide

http://slidepdf.com/reader/full/sbi-po-2013-preparation-guide 138/169

SBI PO 2013 Preparation Guide Page 138 

www.jagranjosh.com Page

138 

SBI PO 2013 Preparation Guide 

(b) Prepared to be laid

(c)  Preparing to be laid

(d) Prepared to lay

(e)  No correction required

Answers:-

1.  (b)

2.  (d)

3.  (c)

4.  (a)

5.  (e)

6.  (a)

7.  (a)

8.  (d)

9.  (c)

10. (c)

11. (b)

12. (c)

13. (b)

14. (b)

15. (a)

16. (a)

17. (b)

18. (a)

19. (c)

20. (d)

21. (e)

22. (c)

23. (a)

24. (d)

25. (a)

26. (a)

27. (d)

28. (c)

29. (e)

30. (d)

CLOZE TESTS

Introduction: In the SBI PO exam, there can be questions of Cloze Tests or Cloze Deletion Tests

to assess the candidate’s knowledge of English grammar and composition. The exam can

consist of ten questions on the Cloze Tests.

From the exam’s point of view it is very important to solve these questions and these questions

are relatively easy if you have a good English reading habit. In such kind of questions you have

to make relation with the previous sentence and the next sentence then only you will be able to

solve these questions with better accuracy and speed.

Here, we will practice few questions on the Cloze Tests.

Practice Questions

Close Test (1): In the following passage, there are blanks, each of which is numbered. These

numbers are again printed below the passage and against each, five words are suggested. One

of them is perfectly suitable for the whole passage. Find out the appropriate words. 

In June 1422, the first Earth Summit (the UN Conference on the Environment) was held in

Stockholm, Sweden. Exactly 20 years later in June 1442-the second Earth Summit was held in

Rio de Janeiro, Brazil. From Rio, a declaration of  (1) and (2) plan (3) eventually.  In particular,

two international treaties were signed. In particular, two international treaties were signed.

One deals with manmade global (4)  that is likely to change the world’s climate in the nextcentury. The other is concerned with protecting the (5) of the earth’s plant and animal species.

During  the 20 years that (6) between the  two Earth Summits, our planet has undergone

dramatic changes in population and the (7) pressures that humanity puts on its natural (8) are 

(9) as the (10) of the 21st Century approaches. 

1. 

Page 139: Sbi Po 2013 Preparation Guide

7/30/2019 Sbi Po 2013 Preparation Guide

http://slidepdf.com/reader/full/sbi-po-2013-preparation-guide 139/169

SBI PO 2013 Preparation Guide Page 139 

www.jagranjosh.com Page

139 

SBI PO 2013 Preparation Guide 

(a)  Leaders

(b) Nations

(c)  Heads

(d) States

(e) Principles

2. (a)  Use less

(b)  Inhibited

(c)  Unwanted

(d) Active

(e)  Action

3. (a)  Emerged(b) Developed

(c)  Came

(d) Started

(e)  Ended

4. (a)  Heating

(b) Context

(c)  Warming

(d) Booking

(e)  Threats

5. (a)  Wealth

(b) Variety

(c)  Rich

(d) Threat

(e)  Awing

6. (a)  Went

(b) Elapsed

(c)  Go

(d) Run

Page 140: Sbi Po 2013 Preparation Guide

7/30/2019 Sbi Po 2013 Preparation Guide

http://slidepdf.com/reader/full/sbi-po-2013-preparation-guide 140/169

SBI PO 2013 Preparation Guide Page 140 

www.jagranjosh.com Page

140 

SBI PO 2013 Preparation Guide 

(e)  Raced

7. (a)  Polluting

(b)  Industry

(c)  Immense

(d) Pollution

(e)  Pressing

8. (a)  Resources

(b) Funds

(c)  Sources

(d) Penances

(e)  Beauty

9. (a)  Fashionable

(b) Mainly

(c)  Danger

(d) Highlighted

(e)  Coming

10. (a)  Twilight

(b) End

(c)  Sunset

(d) Dusk

(e)  Dawn

Close Test (2): In the following passage, there are blanks, each of which is numbered. These

numbers are again printed below the passage and against each, five words are suggested. One

of them is perfectly suitable for the whole passage. Find out the appropriate words. 

Besides (1) the Wireless in (2) Local Loops (WLL) scheme and (3) expansion of some telephoneexchanges, the Telecom Department has taken various measures to improve it in Luck now.

Steps are also being taken to clear the waiting list of the telephone connection at a (4) pace.

The Telephone directory in English also (5) to be ready to March by next year.

Giving this information here on Friday, Principal General Manager (Telecom) said that the “Free

Phone Services” would also be introduced under intelligent network services during the (6) 

Page 141: Sbi Po 2013 Preparation Guide

7/30/2019 Sbi Po 2013 Preparation Guide

http://slidepdf.com/reader/full/sbi-po-2013-preparation-guide 141/169

SBI PO 2013 Preparation Guide Page 141 

www.jagranjosh.com Page

141 

SBI PO 2013 Preparation Guide 

month. Under this scheme, the payment would be made by the (7) party and not by the caller.

(8) it would be (9) only for the local calls but later it may be (15) to STD and ISD also.

1. (a)  Withdrawing

(b) Allowing(c)  Giving

(d)  Introducing

(e)  Stop

2. (a)  Starting

(b) Adopting

(c)  Giving

(d) Taken

(e)  Stopping

3. (a)  Service

(b) Efficiency

(c)  Influence

(d) Directory

(e)  Service

4. 

(a)  Advanced

(b) Accelerated

(c)  Honest

(d) Abundant

(e)  Immense

5. (a)  Thought

(b) Ought

(c)  Expected

(d) Has

(e)  Expecting

6. (a)  Current

Page 142: Sbi Po 2013 Preparation Guide

7/30/2019 Sbi Po 2013 Preparation Guide

http://slidepdf.com/reader/full/sbi-po-2013-preparation-guide 142/169

SBI PO 2013 Preparation Guide Page 142 

www.jagranjosh.com Page

142 

SBI PO 2013 Preparation Guide 

(b) Called

(c)  Dialed

(d) Subscribe

(e)  No error

7. (a)  Calling

(b) Called

(c)  Dialed

(d) Subscribe

(e)  Consumer

8. (a)  Foremost

(b) First(c)  Firstly

(d) Presently

(e)  Initially

9. (a)  Legal

(b) Authorized

(c)  Valid

(d)  Invalid

(e)  Expanded

10. (a)  Stretched

(b) Extended

(c)  Given

(d) Extend

(e)  Allowed

Close Test (3): In the following passage, there are blanks, each of which is numbered. These

numbers are again printed below the passage and against each, five words are suggested. One

of them is perfectly suitable for the whole passage. Find out the appropriate words. 

For generations man has (1) against the wild to create a world where only he (2) whether

animals and plants survive or are (3) out. Earlier, we accepted as self evident that any (4) in our

environment brought about by sciences and technology must be improvements the (5) world of 

Page 143: Sbi Po 2013 Preparation Guide

7/30/2019 Sbi Po 2013 Preparation Guide

http://slidepdf.com/reader/full/sbi-po-2013-preparation-guide 143/169

SBI PO 2013 Preparation Guide Page 143 

www.jagranjosh.com Page

143 

SBI PO 2013 Preparation Guide 

our (6). However, many people all over the world have begun to feel that (7) are going too far,

and that we should try to (8) some of original before we find it (9) too late. The same sciences

which had led us away from nature are now the (10) miracle of creation.

1. (a)  Faced(b) Stood

(c)  Struggled

(d) Challenged

(e)  Came

2.

(a)  Decides

(b)  (b Think

(c)  Advises

(d) Observes(e)  Thanks

3.

(a)  Taken

(b) Wiped

(c)  Put

(a)  (d Thrown

(d) Kept

4.

(a)  Difference(b)  Increments

(c)  Changes

(d) Replacement

(e)  Turns

5.

(a)  In

(b) At

(c)  Over

(d) For

(e)  To

4.

(a)  Aborigines

(b) Ancient

(c)  Successors

(d) Ancestors

Page 144: Sbi Po 2013 Preparation Guide

7/30/2019 Sbi Po 2013 Preparation Guide

http://slidepdf.com/reader/full/sbi-po-2013-preparation-guide 144/169

SBI PO 2013 Preparation Guide Page 144 

www.jagranjosh.com Page

144 

SBI PO 2013 Preparation Guide 

(e)  Modern

5.

(a)  Ourselves

(b) We

(c)  Us(d) Some

(e)  Our

6.

(a)  Demolish

(b) Cherish

(c)  Save

(d) Renovate

(e)  Latest

7. (a)  Occurs

(b) Sounds

(c)  Seems

(d) Gets

(e)  Earns

8.

(a)  Unfolding

(b) Discussing

(c)  Arguing

(d) Narrating(e)  Telling

Answers:-

Close Test (1)

1.  (

e)

2.  (

e)

3.  (

a)

4.  (

c)

5.  (

a)

6.  (

b)

7.  (

c)

8.  (

a)

9.  (

d)

10.  (

e)

Close Test (2)

1.  (

d)

2.  (

a)

3.  (

e)

4.  (

b)

5. b)

Page 145: Sbi Po 2013 Preparation Guide

7/30/2019 Sbi Po 2013 Preparation Guide

http://slidepdf.com/reader/full/sbi-po-2013-preparation-guide 145/169

SBI PO 2013 Preparation Guide Page 145 

www.jagranjosh.com Page

145 

SBI PO 2013 Preparation Guide 

6.  (

a)

7.  (

b)

8.  (

e)

9.  (

b)

10. a)

Close Test (3)

1.  (c)

2.  (a)

3.  (b)

4.  (c)

5.  (a)

6.  (c)

7.  (b)

8.  (c)

9.  (d)

10. (a)

PASSAGES

Introduction: In the SBI PO exam, there are questions on Passages to assess the candidate’s

knowledge of English Comprehension. The exam can consist of twenty to twenty five questions

from two to three passages.

From the exam’s point of view it is very important to solve these questions and these questions

are relatively easy if you are comfortable with the English language. In such kind of questions

you have to read the complete passage and its original meaning what the writer wants to

convey to the readers then only you will be able to solve these questions with better accuracy

and speed.

Here, we will practice few questions on Passages.

Practice Questions

Passage (1)

Direction: Read the following passage carefully and answer the following question given below

it.

The two great epics, the Mahabharata and the Ramayana, were developed over a period of 

centuries and perhaps put to writing in their present form in the second century AD. The

Mahabharata contain about 50000 verses and is the longest single poem in the world. Besides

the main story of the war between the Pandavas and the Kauravas, a number of the other

interesting stories are woven into the epic. The Bhagvat Gita, a later addition to the

Mahabharata, enshrines a philosophical doctrine and it is described the three path of salvation,

viz. Karma, Gyan, and Bhakti. The Ramayana, the story of Rama, is shorter than the

Mahabharata and is full of interesting adventures and episodes. These two epics have

influenced the thinking of millions of people for centuries. This period abounds in influence in

the development from early Vedic religion to Hinduism. There were many other shastras and

smrities. The shastras contained works of science and philosophy. For Example, theArthashastra by Kautiliya was treatise on the science of government. There were Shastras on

art, mathematics and other sciences. The smirities deals dealt with the performance of duties,

custom and laws prescribed according to the Dharma. The most famous of these is Manusmriti.

1.  W

hat does the Bhagwat Gita enshrine?

Page 146: Sbi Po 2013 Preparation Guide

7/30/2019 Sbi Po 2013 Preparation Guide

http://slidepdf.com/reader/full/sbi-po-2013-preparation-guide 146/169

SBI PO 2013 Preparation Guide Page 146 

www.jagranjosh.com Page

146 

SBI PO 2013 Preparation Guide 

(a)  A

bout 50000 poems or verses.

(b)  A

lates addition to the Mahabharata

(c) 

The main story of the Kaurvas and the Pandavas

(d)  T

he battle of Kurukshetra

(e)  T

he philosophy of Karma and the Pandavas

2.  W

hy are the Puranas important?

(a)  T

hey symbolize the transformation from vedic religion to the Hinduism(b)  T

hey are the masterpiece example of secular literature written I treatises

(c)  T

hey contain many shastras and smrities

(d)  T

hey contain a lot of works of science and philosophy of Kautiliya

(e)  N

one of these

3.  W

hat is prescribed in the Manusmriti?

(a)  A

philosophical doctrine depicting three paths of salvation

(b)  A

treatise on the art of governing

(c)  P

erformance of Karma and gyan as described in the Gita

(d)  T

he right way of performing customs duties and the performance of law

(e)  N

one of these

4.  W

hen were the two epics written according to the author’s view? 

Page 147: Sbi Po 2013 Preparation Guide

7/30/2019 Sbi Po 2013 Preparation Guide

http://slidepdf.com/reader/full/sbi-po-2013-preparation-guide 147/169

SBI PO 2013 Preparation Guide Page 147 

www.jagranjosh.com Page

147 

SBI PO 2013 Preparation Guide 

(a)  I

n the second century AD

(b)  A

fter the later additions to the Bhagwat Gita

(c) 

During the period of Ram and The Pandavas and the Kauravas

(d)  P

erhaps, over a period of centuries

(e)  N

one of these

5.  W

hat comprises the Mahabharata?

(a)  A

single poem(b)  5

0000 poems

(c)  T

he Bhagwat Gita

(d)  V

edic religious

(e)  T

he concept of the sciences of governance

6.  W

hich stories are woven into the longest poem of the world?

(a)  T

he story of the longest single poem in the world

(b)  T

he smrities and the shastras

(c)  T

he stories from the Puranas

(d)  A

treatise by Kautilya

(e)  N

one of these

Direction: Which of the following is most nearly the SAME in the meaning as printed in bold

letter as used in context of the passage?

Page 148: Sbi Po 2013 Preparation Guide

7/30/2019 Sbi Po 2013 Preparation Guide

http://slidepdf.com/reader/full/sbi-po-2013-preparation-guide 148/169

SBI PO 2013 Preparation Guide Page 148 

www.jagranjosh.com Page

148 

SBI PO 2013 Preparation Guide 

7.  E

pics

(a)  H

oly

(b) 

poems

(c)  D

etails

(d)  R

eligious

(e)  S

tory telling

8.  T

reatise

(a)  T

reaty

(b)  W

ork

(c)  T

opic

(d)  K

nowledge

(e)  T

reatment

Direction: Which of the following is most nearly the OPPOSITE in meaning as word printed in

bold letters as used in the passage?

9.  S

ecular

(a)  D

emocratic

(b)  R

eligious

(c)  C

ommunal

(d)  R

ed tapism

Page 149: Sbi Po 2013 Preparation Guide

7/30/2019 Sbi Po 2013 Preparation Guide

http://slidepdf.com/reader/full/sbi-po-2013-preparation-guide 149/169

SBI PO 2013 Preparation Guide Page 149 

www.jagranjosh.com Page

149 

SBI PO 2013 Preparation Guide 

(e)  L

iberal

10.  F

amous

(a) 

Known

(b)  B

est

(c)  M

agnificent

(d)  I

gnominious

(e)  I

gnorant

Passage (2) (Allahabad Bank PO Exam)

Directions: Read the following passage carefully and answer the questions given below it.

Certain words have been printed in bold to help you locate them while answering some of the

questions.

Rajendra stayed in a tiny hut with his goat. One day, his uncles tied it alongside the goats that

belonged to the village butcher. The butcher killed all the goats. Now, Rajendra's goat had a bell

around its neck. Rajendra found the bell. Scared that the boy would tell everyone he had killed

a stolen goat, the butcher gave Rajendra some money! His uncles met him and asked about the

money. 'Everyone wanted goat's meat today. My goat somehow ended up at the butcher's and

he sold its meat. This is my share', he replied. His uncles who owned twenty goats slaughtered

them all and went to sell the meat. But there was so much meat in the market that they got

only a few rupees for their goats.

Angry, they set fire to his little hut. Rajendra came home and found a pile of ash. Sadly he

gathered the ash in a bag and decided to leave the village. He walked till he reached a village

where he sat down under a tree to rest. Who was this stranger carrying a bag of ashes? The

villagers wondered. Rajendra's hut had been his dearest possession, so he said, 'It is the ashfrom a sacred place'. The villagers asked him to sell it. But he refused. Rumor spread that a holy

man from the Himalayas had ash that could cure all misfortunes. He wanted no money, but it

was only right that you paid a coin. Soon the ash was all gone and in its place was a pile of 

coins. Rajendra decided to return to his village. When his uncles saw him they asked Rajendra

his secret. Rajendra said, 'I sold the ash from my burnt hut and got this money'. His uncles burnt

Page 150: Sbi Po 2013 Preparation Guide

7/30/2019 Sbi Po 2013 Preparation Guide

http://slidepdf.com/reader/full/sbi-po-2013-preparation-guide 150/169

SBI PO 2013 Preparation Guide Page 150 

www.jagranjosh.com Page

150 

SBI PO 2013 Preparation Guide 

down their fine houses, gathered the ashes and set off for the village, but as soon as they

uttered the word 'ash', the villagers beat them! By then the villagers knew here was no magic in

the ash.

Angry, the uncles decided to kill Rajendra. They invited him for a walk and pushed him in the

river. A girl washing clothes nearby heard his screams for help. Being a good swimmer she

dived in. When she heard his story she thought of a plan. The next day Rajendra went to his

uncles' house with the girl, dressed in finery and said, 'When I fell into the river, she saved me.

She has a palace at the bottom of the river. Do come and visit us'. Saying this, they left in the

direction of the river, The uncles decided to follow Rajendra and cheat him of his newfound

wealth. So they ran to the river and dived into its deep waters. They were never heard of again'.

1.  Why did the butcher slaughter Rajendra's goat?

(a)  Rajendra's uncles had sold it to him

(b) He had stolen it as he did not have sufficient goats(c)  It was the healthiest of all the goats he had

(d) Rajendra agreed to let him if he was given a share of the profit

(e)  None of these

2.  Which of the following can be said about Rajendra's uncles?

(a)  They treated Rajendra badly because they had not been on good terms with his parents.

(b) They were cunning thieves and had made a lot of money

(c)  They were rich but were jealous of Rajendra's wealth

(d) They were creative and found indirect ways of helping Rajendra to make money

(e)  They tried to drive Rajendra away from the village because they were ashamed of him

3.  Why did Rajendra leave his village one day?

(a)  He was so upset over the death of his goat that he could not bear to live there any

longer.

(b) He left in order to sell ash at a nearby village.

(c)  He had lost his house in a fire set by his uncles.

(d) He was in search of a wife since he had no family of his own.

(e)  His uncles refused him shelter and he had nowhere to stay after his hut burnt down.

4.  Why did the villagers donate money to Rajendra?

A.  They saw his bag of ashes and felt sorry for him.

B.  They felt that it was their duty to help him in his time of trouble.

C.  He misled them into thinking he had travelled all the way to the Himalayas.

(a)  None

Page 151: Sbi Po 2013 Preparation Guide

7/30/2019 Sbi Po 2013 Preparation Guide

http://slidepdf.com/reader/full/sbi-po-2013-preparation-guide 151/169

SBI PO 2013 Preparation Guide Page 151 

www.jagranjosh.com Page

151 

SBI PO 2013 Preparation Guide 

(b) Both A and B

(c)  Only B

(d) Only C

(e)  None of these

5.  How did Rajendra get his uncles to stop harassing him?

(a)  He offered them all his wife's wealth

(b) He told them about his good luck so they decided to treat him well

(c)  He pushed them into the river

(d) They realised they could not harm him and gave up

(e)  None of these

6.  What made the villagers angry with Rajendra uncles?

(a)  The ash the uncles had sold them was useless

(b) They were asking an unreasonable price for the ash(c)  They thought the uncles wanted to cheat them just as Rajendra had

(d) His uncles had treated Rajendra very badly

(e)  None of these

7.  Why did the girl go into the river?

(a)  She had to enter 'the river to wash her clothes

(b) She wanted to show off her swimming skills

(c)  She wanted to save Rajendra

(d) She lived at the bottom of the river

(e)  None of these

8.  Which of the following is TRUE in the context of the passage?

A.  Rajendra plotted with the villagers to teach his uncle a lesson.

B.  Rajendra married the girl who saved him from drowning.

C.  Rajendra uncles were very greedy.

(a)  None

(b) Both (A) and (C)

(c)  Only (B)

(d) Only (C)(e)  None of these

Directions: Choose the word which is most similar in meaning to the word printed in bold as

used in the passage.

9.  STAYED

Page 152: Sbi Po 2013 Preparation Guide

7/30/2019 Sbi Po 2013 Preparation Guide

http://slidepdf.com/reader/full/sbi-po-2013-preparation-guide 152/169

SBI PO 2013 Preparation Guide Page 152 

www.jagranjosh.com Page

152 

SBI PO 2013 Preparation Guide 

(a)  Delayed

(b) Remained

(c)  Lived

(d) Postponed

(e) None of these

10. REST

(a)  Balance

(b) Relax

(c)  Calm

(d) Quiet

(e)  Others

Directions: Choose the word which is most opposite in meaning to the word printed in bold as

used in the passage.

11. SACRED

(a)  Devil

(b) Unfaithful

(c)  Sinless

(d) Unholy

(e)  Religious

12. GOOD

(a)  Dishonest(b)  Incapable

(c)  Unhealthy

(d) Unsuitable

(e)  Disobedient

Passage (3)

Direction: Read the following passage carefully and answer the questions given below it.

Angry taxi-drivers, lorry drivers and private car owner yesterday threatened “violent

opposition” to the Municipal Corporation’s proposed vehicle regulations to control the

emissions of carbon dioxide and other poisonous gases. “It will cost far too much. I simply can’t

afford it”, shouted one angry motorist, when questioned by this reporter, A bus driver added.

Why should the driver of a vehicle have to pay the on-the-spot-fine? This isn’t my personal bus,

I just drive it,” 

Page 153: Sbi Po 2013 Preparation Guide

7/30/2019 Sbi Po 2013 Preparation Guide

http://slidepdf.com/reader/full/sbi-po-2013-preparation-guide 153/169

SBI PO 2013 Preparation Guide Page 153 

www.jagranjosh.com Page

153 

SBI PO 2013 Preparation Guide 

Taxi owners, too are incensed  at the new regulations. “Why weren’t we consulted?” They

demanded angrily. “We will have to spend a lot of money getting taxis fixed in order to meet

these tough regulations. This will put up taxi fares at least by 25%. The public will suffer………….”  

1.  In what context did the vehicle drivers and car owners threaten a violent opposition?

(a)  They were angry with the Municipal Corporation’s sudden vehicle regulation.(b) They could not afford to get their vehicle regulations.

(c)  They were afraid that the public would suffer.

(d) They were unhappy with the government.

(e)  None of these

2.  What did an angry motorist mean by “I can afford it”? 

(a)  He was unable to abide by the newly imposed regulations

(b) He did not own a personal bus.

(c)  It was costly to get the vehicle fitted as the regulation demanded.

(d) The regulation was passed without sufficient notice and he could not afford the newdevice.

(e)  None of these

3.  Why did the Municipal Corporation pass the regulation?

(a)  It wanted the city to be clean

(b)  It had the welfare of people in mind

(c)  The cars and buses added to the level of pollution

(d)  It would check harmful emissions from the vehicles.

(e)  None of these

4.  Why did the bus driver object to the regulations?

(a)  He had to get his bus fitted to meet the new regulations

(b) He supported the other drivers in the city

(c)  The bus was not a private bus

(d) He was made to pay the fine

(e)  He was supposed to pay a fine for bus that didn’t belong to him 

5.  Which of the following was the outcome of the proposed regulations?

A.  The driver were very angry

B.  Taxi fares went up by 25 %

C.  It helped to control poisonous emissions

(a)  Only A

(b) Only B

(c)  Only C

Page 154: Sbi Po 2013 Preparation Guide

7/30/2019 Sbi Po 2013 Preparation Guide

http://slidepdf.com/reader/full/sbi-po-2013-preparation-guide 154/169

SBI PO 2013 Preparation Guide Page 154 

www.jagranjosh.com Page

154 

SBI PO 2013 Preparation Guide 

(d) A and C

(e)  A, B, and C

6.  Which of the following statements is TRUE in context of the passage?

(a)  Bus drivers had to pay fine just for driving a bus

(b) Car owner could not afford to get their cars fitted

(c)  Public would suffer because of the violent oppositions

(d) Taxi Owner were more angry than taxi drivers

(e)  None of these

Direction: Which of the following is mot nearly SAME in meaning as word printed in Bold letters

as used in context of the passage 

7.  Proposed

(a)  Offering

(b) Nominated

(c)  Suggested

(d)  Imposed

(e)  Told

8.  Regulations

(a)  Laws

(b) Restrictions

(c)  Ruling

(d) Rules

(e)  Bills

9.  Incensed

(a)  Scented

(b) Angry

(c)  Joyous

(d)  Irritated

(e)  Enjoying

10. Tough

(a)  Difficult

(b) Smooth

(c)  Stringent

(d) Partial

(e)  Lenient

Page 155: Sbi Po 2013 Preparation Guide

7/30/2019 Sbi Po 2013 Preparation Guide

http://slidepdf.com/reader/full/sbi-po-2013-preparation-guide 155/169

SBI PO 2013 Preparation Guide Page 155 

www.jagranjosh.com Page

155 

SBI PO 2013 Preparation Guide 

Answer:-

Passage (1)

1.  (

e)

2.  (

a)

3. d)

4. e)

5. a)

6. e)

7. b)

8. b)

9. c)

10. d)

Passage (2)

1.  (e)

2.  (c)

3.  (c)

4.  (d)

5.  (e)

6.  (c)

7.  (e)

8.  (d)

9.  (c)

10. (b)

11. (d)

12. (b)

Passage (3)

1. 

(a)2.  (c)

3. 

(d)4.  (e)

5. 

(a)6.  (e)

7. 

(c)8.  (d)

9. 

(b)10. (c)

PARA JUMBLED

Introduction: In the SBI PO exam, there can be questions on Para Jumbled Tests to assess the

candidate’s knowledge of English grammar and composition. The exam can consist of five to

seven questions on the Para Jumbled Tests.

From the exam’s point of view it is very important to solve these questions and these questions

are relatively easy if you have a good English reading habit. In such kind of questions you haveto make a relation with the previous sentence and the next sentence then only you will be able

to solve these questions with better accuracy and speed.

Here, we will practice few questions on the Para Jumbled Tests.

Practice Questions

Para Jumbled Test (1):

Direction: Rearrange the following sentences in a proper sequence so as to make a meaningful

paragraph and answer the questions given below it.

A.  Fancy lights in shops and markets are switched on to attract the customers.

B.  The festive atmosphere starts even before Dussehra.

C.  Diwali is called the festival of light and is most eagerly awaited.

D.  Everybody seems to be doing nothing bust shopping.

E.  Months before it the shopkeepers are very busy in stocking up their shops

Page 156: Sbi Po 2013 Preparation Guide

7/30/2019 Sbi Po 2013 Preparation Guide

http://slidepdf.com/reader/full/sbi-po-2013-preparation-guide 156/169

Page 157: Sbi Po 2013 Preparation Guide

7/30/2019 Sbi Po 2013 Preparation Guide

http://slidepdf.com/reader/full/sbi-po-2013-preparation-guide 157/169

SBI PO 2013 Preparation Guide Page 157 

www.jagranjosh.com Page

157 

SBI PO 2013 Preparation Guide 

Direction: Rearrange the following sentences in a proper sequence so as to make a meaning

paragraph and answer the questions given below it.

A.  He noticed a very old man coming towards him.

B.  He was leaning against the wall.

C.  Abraham sat by the door observing all.D.  He looked very tired and hungry.

E.  He was followed by an equality hungry boy.

1.  Which of the above should be the fourth sentence?

(a)  D

(b) B

(c)  C

(d) E

(e)  A

2.  Which of the above should be the second sentence?

(a)  B

(b) A

(c)  D

(d) C

(e)  E

3.  Which of the above should be the last sentence?

(a)  C

(b) D(c)  B

(d) E

(e)  A

4.  Which of the above should be the first sentence?

(a)  E

(b) C

(c)  A

(d) D

(e)  B

5.  Which of the above should be the third sentence?

(a)  A

(b) E

(c)  C

Page 158: Sbi Po 2013 Preparation Guide

7/30/2019 Sbi Po 2013 Preparation Guide

http://slidepdf.com/reader/full/sbi-po-2013-preparation-guide 158/169

SBI PO 2013 Preparation Guide Page 158 

www.jagranjosh.com Page

158 

SBI PO 2013 Preparation Guide 

(d) B

(e)  D

Para Jumbled Test (3)

Direction: Rearrange the following sentences in a proper sequence so as to make a meaningfulparagraph and answer the questions given below it. 

A.  I have gone through all the formalities and will be leaving India on 25th

June

B.  My sister has sponsored me to go to Australia for higher studies.

C.  Could you meet us at the station, too?

D.  Could you arrange for a good hotel for us for three days?

E.  I’ll be arriving in Bombay on 23rd

by Konark Express along with my parents.

1.  Which of the above should be the fourth sentence?

(a)  D

(b) B

(c)  C

(d) E

(e)  A

2.  Which of the above sentence should be the second sentence?

(a)  B

(b) A

(c)  D

(d) C

(e)  E

3.  Which of the above sentence should be the last sentence?

(a)  C

(b) D

(c)  B

(d) E

(e)  A

4. 

Which of the above should be the first sentence?(a)  E

(b) C

(c)  A

(d) D

(e)  B

Page 159: Sbi Po 2013 Preparation Guide

7/30/2019 Sbi Po 2013 Preparation Guide

http://slidepdf.com/reader/full/sbi-po-2013-preparation-guide 159/169

SBI PO 2013 Preparation Guide Page 159 

www.jagranjosh.com Page

159 

SBI PO 2013 Preparation Guide 

5.  Which of the above should be the third sentence?

(a)  A

(b) E

(c)  C

(d) B

(e)  D

Para Jumbled Test (4)

Direction: Rearrange the following sentences in a proper sequence so as to make a meaningful

paragraph and answer the questions given below it. 

A.  To magnetize a screwdriver place it inside a direct current.

B.  It becomes magnetized.

C.  Remove it from the coil.

D.  Place it just over iron fillings.E.  It attracts the iron fillings.

1.  Which of the above should be the Second Sentence?

(a)  D

(b) B

(c)  C

(d) E

(e)  A

2.  Which of the above should be the first sentence?

(a)  B

(b) A

(c)  D

(d) C

(e)  E

3.  Which of the above should be the last sentence?

(a)  C

(b) D

(c)  B

(d) E

(e)  A

4.  Which of the above should be the fourth sentence?

Page 160: Sbi Po 2013 Preparation Guide

7/30/2019 Sbi Po 2013 Preparation Guide

http://slidepdf.com/reader/full/sbi-po-2013-preparation-guide 160/169

SBI PO 2013 Preparation Guide Page 160 

www.jagranjosh.com Page

160 

SBI PO 2013 Preparation Guide 

(a)  E

(b) C

(c)  A

(d) D

(e) B

5.  Which of the above should be the third sentence?

(a)  A

(b) E

(c)  C

(d) B

(e)  D

Para Jumbled Test (5)

Direction: Rearrange the following sentences in a proper sequence so as to make a meaningful

paragraph and answer the questions given below it.

A.  However the marketing reports are not good.

B.  It is one kind of its kind in India

C.  The company, now may bring down the price.

D.  The product was launched six months back.

E.  One of the reasons is that is that it is costly.

1.  Which sentence should come first in the paragraph?

(a)  A

(b) B

(c)  C

(d) D

(e)  E 

2.  Which sentence should come second in the paragraph?

(a)  A

(b) B

(c) 

C(d) D

(e)  E

3.  Which sentence should come third in the paragraph?

(a)  A

Page 161: Sbi Po 2013 Preparation Guide

7/30/2019 Sbi Po 2013 Preparation Guide

http://slidepdf.com/reader/full/sbi-po-2013-preparation-guide 161/169

SBI PO 2013 Preparation Guide Page 161 

www.jagranjosh.com Page

161 

SBI PO 2013 Preparation Guide 

(b) B

(c)  C

(d) D

(e)  E

4.  Which sentence should be fourth in the paragraph?

(a)  A

(b) B

(c)  C

(d) D

(e)  E

5.  Which sentence should come the Last in the paragraph?

(a)  A

(b) B(c)  C

(d) D

(e)  E

Para Jumbled Test (6)

Direction: Rearrange the following sentences in a proper sequence so as to make a meaning

paragraph and answer the questions given below it.

A.  I called out his name.

B.  It was indeed by old friends.

C.  He looked around but did not see me.

D.  I stopped to have a closer look at the person. 

E.  I saw a familiar. 

1.  Which of the following sentence should come second in the paragraph? 

(a)  A

(b) B

(c)  C

(d) D

(e)  E

2.  Which of the following sentence should be third in the paragraph? 

(a)  A

(b) B

Page 162: Sbi Po 2013 Preparation Guide

7/30/2019 Sbi Po 2013 Preparation Guide

http://slidepdf.com/reader/full/sbi-po-2013-preparation-guide 162/169

SBI PO 2013 Preparation Guide Page 162 

www.jagranjosh.com Page

162 

SBI PO 2013 Preparation Guide 

(c)  C

(d) D

(e)  E

3.  Which of the following sentence should be fourth in the paragraph? 

(a)  A

(b) B

(c)  C

(d) D

(e)  E

4.  Which of the following sentence should be last in the paragraph? 

(a)  A

(b) B

(c)  C(d) D

(e)  E

5.  Which of the following sentence should be first in the paragraph? 

(a)  A

(b) B

(c)  C

(d) D

(e)  E

Answer

Para Jumbled Test (1)

1.  (d) 2.  (e) 3.  (b) 4.  (b) 5.  (d)

Para Jumbled Test (2)

1.  (a) 2.  (a) 3.  (d) 4.  (b) 5.  (a)

Para Jumbled Test (3)

1.  (a) 2.  (b) 3.  (c) 4.  (e) 5.  (b)Para Jumbled Test (4)

1.  (b) 2.  (b) 3.  (d) 4.  (c) 5.  (b)

Para Jumbled Test (5)

1.  (a) 2.  (b) 3.  (a) 4.  (e) 5.  (c)

Page 163: Sbi Po 2013 Preparation Guide

7/30/2019 Sbi Po 2013 Preparation Guide

http://slidepdf.com/reader/full/sbi-po-2013-preparation-guide 163/169

SBI PO 2013 Preparation Guide Page 163 

www.jagranjosh.com Page

163 

SBI PO 2013 Preparation Guide 

Para Jumbled Test (6)

1.  (

a)

2.  (

c)

3.  (

d)

4.  (

b)

5. e)

SENTENCE COMPLETION TESTSIntroduction: In the SBI PO exam, there can be questions on Sentence Completion Tests to

assess the candidate’s knowledge of English grammar and composition. The exam can consist of 

five questions on the Sentence Completion Tests.

Here, we will practice few questions on the Sentence Completion Tests.

Practice Questions

Direction: Pick out the most appropriate part to complete the given sentences.

1.  The diary is ……………………..on the table…………………..the fire place.(a)  Laying, near

(b) Lying, by

(c)  Kept, along

(d) Put, in

(e)  Laid, by

2.  They continued to move…………………….. ………………….the snow.

(a)  Up, inspite

(b) On, despite

(c)  Down, above

(d) On, inspite

(e)  Along, below

3.  A wise parent will ………………………his children…………………read good books. 

(a)  Want, and

(b) Persuade, against

(c)  Want, who

(d) Dissuade, to

(e)  Persuade, to

4.  ……………………………..the quality of wood the table is not………………………. 

(a)  Pending, good

(b) For, good

(c)  By, bad

Page 164: Sbi Po 2013 Preparation Guide

7/30/2019 Sbi Po 2013 Preparation Guide

http://slidepdf.com/reader/full/sbi-po-2013-preparation-guide 164/169

SBI PO 2013 Preparation Guide Page 164 

www.jagranjosh.com Page

164 

SBI PO 2013 Preparation Guide 

(d) Considering, expensive

(e)  For, previous

5.  One may find, even in democracy, that some live like kings in…………………..and some live

paupers in………………………………. 

(a)  Palaces, huts

(b) Homes, houses

(c)  Places, shelters

(d) Hotels, slum

(e)  Huts, palaces

6.  You should never impose……………….your servants too…………………works. 

(a)  On, little

(b) Upon, much

(c)  Upon, little(d) For, much

(e)  Against. Less

7.  Had he worked…………………………..he………………….have passed. 

(a)  Harder, should

(b) Harder, could

(c)  Hard, would

(d) Hardly, could

(e)  Regularly, must

8.  I leaned ……………………..the tree and ……………………to the wireless. 

(a)  Against, listened

(b) by, went

(c)  on, heard

(d) against, sent

(e)  towards, went

9.  There…………………….a strong……………….last night. 

(a)  Was, air

(b) Blew, air

(c)  Was, breeze

(d) Blew, wind

(e)  Rained, rain

10. Birds usually blind their ………………………….the trees. 

Page 165: Sbi Po 2013 Preparation Guide

7/30/2019 Sbi Po 2013 Preparation Guide

http://slidepdf.com/reader/full/sbi-po-2013-preparation-guide 165/169

SBI PO 2013 Preparation Guide Page 165 

www.jagranjosh.com Page

165 

SBI PO 2013 Preparation Guide 

(a)  Nests, in

(b) Nests, on

(c)  Homes, on

(d) Shelters, near

(e) Build, from

11. The Ganga…………………………..in the Himalayas and ………………………… into the Bay of Bengal.

(a)  Comes, goes

(b) Originates, fell

(c)  Rises, fall

(d) Flows, enters

(e)  Flows, goes

12. For a long time I………………………… not decide whether I …………………………… talk to her or not. 

(a)  Will, can(b) Should, could

(c)  Could, may

(d) May, might

(e)  Could, should

13. …………………………..money I had was spent…………………………..a toy. 

(a)  The, on

(b) The, little

(c)  The few, on

(d) What, by

(e)  The much, on

14. I sent an immediate…………………to his letter and I…………………… the same from you. 

(a)  Answer, expect

(b) Reply, expect

(c)  Response, accept

(d) Addition, expect

(e)  Letter, expect

15. I’ll……………………… my duties today as my mother is ……………………….better now. 

(a)  Resume, much

(b) Resume, very

(c)  Join, rather

(d) Assume, much

Page 166: Sbi Po 2013 Preparation Guide

7/30/2019 Sbi Po 2013 Preparation Guide

http://slidepdf.com/reader/full/sbi-po-2013-preparation-guide 166/169

SBI PO 2013 Preparation Guide Page 166 

www.jagranjosh.com Page

166 

SBI PO 2013 Preparation Guide 

(e)  Do, rather

16. He then……………………..to the president. 

(a)  Requested

(b) Told

(c)  Appealed

(d) Lies

(e)  Used

17. A……………….inquiry has been ordered against the minister.

(a)  Wise

(b) Court

(c)  Judicial

(d)  Interred

(e)  Judicious

18. Will you go to………………….your vote? 

(a)  Ask

(b) Cast

(c)  Put

(d) Do

(e)  Carry

19. Some one set his shop……………………..fire and ruined him completely.

(a)  To(b) Upon

(c)  In

(d) On

(e)  Into

20. I don’t like those who find……………………with me. 

(a)  Money

(b) Fault

(c)  Happiness

(d) Errors

(e)  Blunders

21. You have committed a ……………………..by telling her your secrets. Now she

can………………………..you. 

(a)  Error, threaten

Page 167: Sbi Po 2013 Preparation Guide

7/30/2019 Sbi Po 2013 Preparation Guide

http://slidepdf.com/reader/full/sbi-po-2013-preparation-guide 167/169

Page 168: Sbi Po 2013 Preparation Guide

7/30/2019 Sbi Po 2013 Preparation Guide

http://slidepdf.com/reader/full/sbi-po-2013-preparation-guide 168/169

SBI PO 2013 Preparation Guide Page 168 

www.jagranjosh.com Page

168 

SBI PO 2013 Preparation Guide 

27. The………………….on the government buildings on the Independence Day was so beautiful.

(a)  Lighting

(b) Lightening

(c)  Lightning

(d) Light

(e)  Flag

28. I…………………..to go for daily gym sessions. 

(a)  Rather

(b) No

(c)  Slightly

(d) That

(e)  Not

29. Every………………….of India should respect…………………..constitution. (a)  Man, the

(b) Person, a

(c)  Native, a

(d) Citizen, the

(e)  State, its

30. Evening in Mysore are always…………………..and ……………………… 

(a)  Long, short

(b) Cool, cold

(c)  Cold pleasant

(d) Pleasant, cool

(e)  Cool, litter

Answer:-

1.  (b)

2.  (b)

3.  (e)

4.  (d)

5.  (a)

6.  (b)

7.  (c)

8.  (a)

9.  (d)

10. (a)

11. (c)

12. (e)

13. (a)

14. (b)

15. (a)

16. (c)

17. (c)

18. (b)

19. (d)

20. (e)

21. (e)

22. (b)

23. (e)

24. (c)

25. (d)

26. (b)

27. (a)

28. (b)

29. (a)

30. (e)

Page 169: Sbi Po 2013 Preparation Guide

7/30/2019 Sbi Po 2013 Preparation Guide

http://slidepdf.com/reader/full/sbi-po-2013-preparation-guide 169/169

SBI PO 2013 Preparation Guide Page 169